CAPSTONE ATI COMPREHENSIVE PREDICTOR 2023-2024 TEST BANK/RN ATI CAPSTONE COMPREHENSIVE PREDICTOR TEST BANK 2000+ REAL EXAM QUESTIONS AND CORRECT ANSWERS

What can be delegated to Assistive personnel (AP)?

– ADLs – bathing – grooming – dressing – ambulating – feeding (w/o swallow precautions) – positioning – bed making – specimen collection – I&O – VS (stable clients

A nurse on a med surge unit has recieved change of shift report and will care for 4 clients. Which of the following clients needs will the nurse assign to an AP?
A. Feeding a client who was admitted 24 hours ago with aspiration pneumonia
B. Reinforcing teaching with a client who is learning to walk with a quad cane
C. Reapplying a condom catheter for a client who has urinary incontinence
D. Applying a sterile dressing to a pressure ulcer

C

A nurse is delegating the ambulation of a client who had knee arthroplasty 5 days ago to an AP. Which of the following info should the nurse share with the AP?
Select all:
A. the roommate is up independently
B. The client ambulates with his slippers on over his antiembolic stockings
C. The client uses a front wheeled walker when ambulating
D. The client had pain meds 30 minutes ago
E. The client is allergic to codeine
F. the client ate 50 % of his breakfast this morning

B
C
D

An RN is making assignments for client care to an LPN at the beginning of the shift. Which of the following assignments should the LPN question?
A. Assisting a client who is 24 hr postop to use an incentive spirometer
B. Collecting a clean catch urine specimen from a client who was admitted on the previous shift
C. providing nasopharyngeal suctioning for a client who has pneumonia
D. Replacing the cartridge and tubing on a PCA pump

D

A nurse is preparing an inservice program about delegation. Which of the following elements should she identify when presenting the 5 rights of delegation. Select all:
A. Right client
B. Right supervision/evaluation
C. Right direction/communication
D. Right time
E. Right circumstances

B
C
E

A nurse manager of a med surge unit is assigning care responsibilities for the oncoming shift. A client is waiting transfer back to the unit from the PACU following thoracic surgery. To which staff member should the nurse assign the client?
A. Charge nurse
B. RN
C. LVN
D. AP

B

What is the study of conduct and character?

Ethics

What are the values and beliefs that guide behavior and decision making?

Morals

What is the right to make ones own personal decisions, even tho those decisions might not be in the persons best interest

Autonomy

What are positive actions to help others

Beneficience

What is an agreement to keep promises

Fidelity

What is fairness in care delivery and use of resources

Justice

What is avoidance of harm or injury

Non-maleficence

A nurse is caring for a client who decides not to have surgery despite significant blockages in his coronary arteries. The nurse understands that this clients choice is an example of what principles?
A. Fidelity
B. Autonomy
C. Justice
D. Nonmalificience

A

A nurse offers pain meds to a client who is postop prior to ambulation. The nurse understands that this aspect of care delivery is an example of which of the following ethical principles?
A. Fidelity
B. Autonomy
C. Justice
D. Beneficience

D

A nurse is instructing a group of nursing students about the responsibilities involved with organ donation and procurement. When the nurse explains that all clients waiting for a kidney transplant have to meet the same qualifications, the students should understand that this aspect of care delivery is an example of which ethical principle
A. Fidelity
B. Autonomy
C. Justice
D. Nonmaleficence

C

A nurse questions a med prescription as too extreme and light of the clients advanced age and unstable status. The nurse understands that this action is an example of which ethical principle
A. Fidelity
B. Autonomy
C. Justice
D. Nonmalificence

D

Which of the following situations can be identified as an ethical dilemma?
A. A nurse on a med surge unit demonstrates signs of chemical impairment
B. A nurse over hears another nurse telling an older adult client that if he doesnt stay in bed she will restrain him
C. A family has conflicting feelings about the initiation of enteral tube feedings of their father who is terminally ill
D. A client who is terminally ill hesitates to name her spouse on her durable power of attorney form

C

Most managers can be categorized as

authoritative, democratic, and laissez faire

makes decisions of the group
motivates by coercion
communication occurs down the chain of command
Work output by the staff is usually high-good for crisis situations and bureaucratic settings

Authoritative

includes the group when decisions are made
Motivates by supporting star achievements
Communication occurs up and down the chain of command
Work output by staff is usually of good quality-good when cooperation and collaboration is necessary

Democratic

makes very few decisions and does little planning
motivation is largely the responsibility of individuals staff members
Communication occurs up and down the chain of command and between group members
Work output is low unless an informal leader evolves from the group
*the use of any of these styles may be appropriate depending on the situation

Laissez faire

The nurse should consider the hierarchy of human needs when prioritizing interventions, which are?

– Physiological needs first (oxygen, shelter, food)
– Safety & security needs (physical safety)
– Love and belonging
– Self esteem
– Self actualization

The ABC framework identifies, in order, the three basic needs for sustaining life

Airway
Breathing
Circulation

Nurses must follow what code of standards in delegating and assigning tasks

ANA codes of standards

What values would a nurse possess to be a client advocate?

– caring
– autonomy
– respect
– empowerment

What do the nurse need to keep in mind about the client when being their advocate?

Client’s religion & culture

When should planning discharge process begin?
a. at time of admission
b. 2 days after client is admitted
c. whenever the nurse has the time to do planning
d. when the physician has the discharge order

A

What is an interdisciplinary team?

A group of health care professionals from different disciplines

Fill in the blank:
1. _______ is used by interdisciplinary team to make health care decisions about clients with multiple problems. 2. ________, which may take place at team meetings, allows the achievement of results that the participants would be incapable of accomplishing if working alone.

1 & 2 = collaboration

What is the nurse’s contribution to an interdisciplinary team?

– knowledge of nursing care & its management
– a holistic understanding of the client, her/his healthcare needs & healthcare systems.

A four-month-old infant is admitted to the pediatric intensive care unit
with a temperature of 105°F (40.5 °C). The infant is irritable, and the nurse
observes nuchal rigidity. Which assessment finding would indicate an
increase in intracranial pressure?
1. Positive Babinski.
2. High-pitched cry.
3. Bulging posterior fontanelle.
4. Pinpoint pupils.

2

A client is receiving total parenteral nutrition (TPN). To determine the
client’s tolerance of this treatment, the nurse should assess for which of the
following?
1. A significant increase in pulse rate.
2. A decrease in diastolic blood pressure.
3. Temperature in excess of 98.6°F (37°C).
4. Urine output of at least 30 cc per hour.

4

The client is exhibiting symptoms of myxedema. The nursing
assessment should reveal
1. increased pulse rate.
2. decreased temperature.
3. fine tremors.
4. increased radioactive iodine uptake level.

2

A nonstress test is scheduled for a client at 34-weeks gestation who
developed hypertension, periorbital edema, and proteinuria. Which of the
following nursing actions should be included in the care plan in order to
BEST prepare the client for the diagnostic test?
1. Start an intravenous line for an oxytocin infusion.
2. Obtain a signed consent prior to the procedure.
3. Instruct client to push a button when she feels fetal movement.
4. Attach a spiral electrode to the fetal head.

3

Which of the following nursing interventions is MOST important for a
45-year-old woman with rheumatoid arthritis?
1. Provide support to flexed joints with pillows and pads.
2. Position her on her abdomen several times a day.
3. Massage the inflamed joints with creams and oils.
4. Assist her with heat application and ROM exercises.

4

The nurse is caring for a young adult admitted to the hospital with a
severe head injury. The nurse should position the patient
1. with his neck in a midline position and the head of the bed elevated 30°.
2. side-lying with his head extended and the bed flat.
3. in high Fowler’s position with his head maintained in a neutral position.
4. in semi-Fowler’s position with his head turned to the side.

1

The nurse is teaching a 40-year-old man diagnosed with a lower motor
neuron disorder to perform intermittent self-catheterization at home. The
nurse should instruct the client to
1. use a new sterile catheter each time he performs a catheterization.
2. perform the Valsalva maneuver(holding breath and bearing down) before doing the catheterization.
3. perform the catheterization procedure every 8 hours.
4. limit his fluid intake to reduce the number of times a catheterization is needed.

2

A client is being discharged with sublingual nitroglycerin (Nitrostat).
The client should be cautioned by the nurse to
1. take the medication five minutes after the pain has started.
2. stop taking the medication if a stinging sensation is absent.
3. take the medication on an empty stomach.
4. avoid abrupt changes in posture.

4

A 38-year-old woman is returned to her room after a subtotal
thyroidectomy for treatment of hyperthyroidism. Which of the following, if
found by the nurse at the patient’s bedside, is nonessential?
1. Potassium chloride for IV administration.
2. Calcium gluconate for IV administration.
3. Tracheostomy set-up.
4. Suction equipment.

1

A nurse recognizes that an initial positive outcome of treatment for a
victim of sexual abuse by one parent would be that the client
1. acknowledges willing participation in an incestuous relationship.
2. reestablishes a trusting relationship with his/her other parent.
3. verbalizes that s/he is not responsible for the sexual abuse.
4. describes feelings of anxiety when speaking about sexual abuse.

3

An adolescent client is ordered to take tetracycline HCL (Achromycin)
250 mg PO bid. Which of the following instructions should be given to this
client by the nurse?
1. “Take the medication on a full stomach, or with a glass of milk.”
2. “Wear sunscreen and a hat when outdoors.”
3. “Continue taking the medication until you feel better.”
4. “Avoid the use of soaps or detergents for two weeks.”

2

After a client develops left-sided hemiparesis from a cerebral vascular
accident (CVA), there is a decrease in muscle tone. Which of the following
nursing diagnoses would be a priority to include in his care plan?
1. Alteration in mobility related to paralysis.
2. Alteration in skin integrity related to decrease in tissue oxygenation.
3. Alteration in skin integrity related to immobility.
4. Alteration in communication related to decrease in thought processes

2

A client has a history of oliguria, hypertension, and peripheral edema.
Current lab values are: BUN -25, K+ -4.0 mEq/L. Which nutrient should be
restricted in the client’s diet?
1. Protein.
2. Fats.
3. Carbohydrates.
4. Magnesium.

1

An extremely agitated client is receiving haloperidol (Haldol) IM every
30 minutes while in the psychiatric emergency room. The MOST important
nursing intervention is to
1. monitor vital signs, especially blood pressure, every 30 minutes.
2. remain at the client’s side to provide reassurance.
3. tell the client the name of the medication and its effects.
4. monitor the anticholinergic effects of the medication.

1

The nurse is caring for clients in the skilled nursing facility. Which of the
following clients require the nurse’s IMMEDIATE attention?
1. A client admitted for a cerebral vascular accident (CVA) whose prescription for
warfarin (Coumadin) expired two days ago.
2. A client in pain who was receiving morphine in an acute care institution and was
transferred with a prescription for acetaminophen with codeine.
3. A client who has dysuria and foul-smelling, cloudy, dark amber urine.
4. An immunosuppressed client who has not received an influenza immunization.

1

The nurse is observing care given to a client experiencing severe to
panic levels of anxiety. The nurse would intervene in which of the following
situations?
1. The staff maintains a calm manner when interacting with the client.
2. The staff attends to client’s physical needs as necessary.
3. The staff helps the client identify thoughts or feelings that occurred prior to the
onset of the anxiety.
4. The staff assesses the client’s need for medication or seclusion if other
interventions have failed to reduce anxiety.

3

A 69-year-old client is undergoing his second exchange of intermittent
peritoneal dialysis (IPD). Which of the following would require an
intervention by the nurse?
1. The client complains of pain during the inflow of the dialysate.
2. The client complains of constipation.
3. The dialysate outflow is cloudy.
4. There is blood-tinged fluid around the intra-abdominal catheter.

3

The clinic nurse is performing diet teaching with a 67-year-old client
with acute gout. The nurse should teach the client to limit his intake of
1. red meat and shellfish.
2. cottage cheese and ice cream.
3. fruit juices and milk.
4. fresh fruits and uncooked vegetables.

1

A client is scheduled for a left lower lobectomy. The physician has
ordered diazepam (Valium) 2 mg IM for anxiety. The nurse would determine
that the medication is appropriate if the client displays which of the
following symptoms?
1. Agitation and decreased level of consciousness.
2. Lethargy and decreased respiratory rate.
3. Restlessness and increased heart rate.
4. Hostility and increased blood pressure.

3

A 59-year-old woman with bipolar disorder is receiving haloperidol
(Haldol) 2 mg PO tid. She tells the nurse, “Milk is coming out of my
breasts.” Which of the following responses by the nurse is BEST?
1. “You are seeing things that aren’t real.”
2. “Why don’t we go make some fudge.”
3. “You are experiencing a side effect of Haldol.”
4. “I’ll contact your physician to change your medication.”

3

The physician orders ranitidine hydrochloride (Zantac) 150 mg PO qd for
a client. The nurse should advise the client the BEST time to take this
medication is
1. before breakfast.
2. with dinner.
3. with food.
4. at hs.

4

. If a client develops cor pulmonale (right-sided heart failure), the nurse
would expect to observe
1. increasing respiratory difficulty seen with exertion.
2. cough productive of a large amount of thick, yellow mucus.
3. peripheral edema and anorexia.
4. twitching of extremities.

3

The nurse is performing triage on a group of clients in the emergency
department. Which of the following clients should the nurse see FIRST?
1. A 12-year-old oozing blood from a laceration of the left thumb due to cut on a
rusty metal can.
2. A 19-year-old with a fever of 103.8°F (39.8°C) who is able to identify her sister
but not the place
and time.
3. A 49-year-old with a compound fracture of the right leg who is complaining of
severe pain.
4. A 65-year-old with a flushed face, dry mucous membranes, and a blood sugar of
470 mg/dL.

2

The nurse in the outpatient clinic teaches a client with a sprained right
ankle to walk with a cane. What behavior, if demonstrated by the client,
would indicate that teaching was effective?
1. The client advances the cane 18 inches in front of her foot with each step.
2. The client holds the cane in her left hand.
3. The client advances her right leg, then her left leg, and then the cane.
4. The client holds the cane with her elbow flexed 60°.

2

A client returns to his room following a myelogram. The nursing care
plan should include which of the following?
1. Encourage oral fluid intake.
2. Maintain the prone position for 12 hours.
3. Encourage the client to ambulate after the procedure.
4. Evaluate the client’s distal pulses on the affected side.

1

The nurse is caring for a patient following an appendectomy. The patient
takes a deep breath, coughs, and then winces in pain. Which of the
following statements, if made by the nurse to the patient, is BEST?
1. “Take three deep breaths, hold your incision, and then cough.”
2. “That was good. Do that again and soon it won’t hurt as much.”
3. “It won’t hurt as much if you hold your incision when you cough.”
4. “Take another deep breath, hold it, and then cough deeply

1

A young woman is transferred to a psychiatric crisis unit with a
diagnosis of a dissociative disorder. The nurse knows which of the following
comments by the client is MOST indicative of this disorder?
1. “I keep having recurring nightmares.”
2. “I have a headache and my stomach has bothered me for a week.”
3. “I always check the door locks three times before I leave home.”
4. “I don’t know who I am and I don’t know where I live.”

4

A 23-year-old man is admitted with a subdural hematoma and cerebral
edema after a motorcycle accident. Which of the following symptoms should
the nurse expect to see INITIALLY?
1. Unequal and dilated pupils.
2. Decerebrate posturing.
3. Grand mal seizures.
4. Decreased level of consciousness.

4

. The nursing team includes two RNs, one LPN/LVN, and one nursing
assistant. The nurse should consider the assignments appropriate if the
nursing assistant is assigned to care for
1. a client with Alzheimer’s requiring assistance with feeding.
2. a client with osteoporosis complaining of burning on urination.
3. a client with scleroderma receiving a tube feeding.
4. a client with cancer who has Cheyne-Stokes respirations.

1

An elderly client is returned to her room after an open reduction and
internal fixation of the left femoral head after a fracture. It is MOST
important for the nursing care plan to include that the client
1. eat a high-protein, low-residue diet.
2. lie on her unoperated side.
3. exercise her arms and legs.
4. cough and deep breathe.

4

Which of the following is a correctly stated nursing diagnosis for a client
with abruptio placentae?
1. Infection related to obstetrical trauma.
2. Potential for fetal injury related to abruptio placentae.
3. Potential alteration in tissue perfusion related to depletion of fibrinogen.
4. Fluid volume deficit related to bleeding.

4

An 8-year-old client is returned to the recovery room after a
bronchoscopy. The nurse should position the client
1. in semi-Fowler’s position.
2. prone, with the head turned to the side.
3. with the head of the bed elevated 45° and the neck extended.
4. supine, with the head in the midline position.

1

Which of the following assessment findings would indicate to the nurse
the need for more sedation in a client who is withdrawing from alcohol
dependence?
1. Steadily increasing vital signs.
2. Mild tremors and irritability.
3. Decreased respirations and disorientation.
4. Stomach distress and inability to sleep.

1

The home care nurse is instructing a client recently diagnosed with
tuberculosis. It is MOST important for the nurse to include which of the
following as a part of the teaching plan?
1. During the first two weeks of treatment, the client should cover his mouth and
nose when he coughs or sneezes.
2. It is necessary for the client to wear a mask at all times to prevent transmission of
the disease.
3. The family should support the client to help reduce feeling of low self-esteem and
isolation.
4. The client will be required to take prescribed medication for a duration of 6-9
months.

4

The nurse’s INITIAL priority when managing a physically assaultive
client is to
1. restrict the client to the room.
2. place the client under one-to-one supervision.
3. restore the client’s self-control and prevent further loss of control.
4. clear the immediate area of other clients to prevent harm.

3

A client with newly diagnosed type I diabetes mellitus is being seen by
the home health nurse. The physician orders include: 1,200-calorie ADA
diet, 15 units of NPH insulin before breakfast, and check blood sugar qid.
When the nurse visits the client at 5 PM, the nurse observes the man
performing a blood sugar analysis. The result is 50 mg/dL. The nurse would
expect the client to be
1. confused with cold, clammy skin and a pulse of 110.
2. lethargic with hot, dry skin and rapid, deep respirations.
3. alert and cooperative with a BP of 130/80 and respirations of 12.
4. short of breath, with distended neck veins and a bounding pulse of 96.

1

The nurse is supervising the staff providing care for an 18-month-old
hospitalized with hepatitis A. The nurse determines that the staff’s care is
appropriate if which of the following is observed?
1. The child is placed in a private room.
2. The staff removes a toy from the child’s bed and takes it to the nurse’s station.
3. The staff offers the child french fries and a vanilla milkshake for a midafternoon
snack.
4. The staff uses standard precautions.

1

When using restraints for an agitated/aggressive patient, which of the
following statements should NOT influence the nurse’s actions during this
intervention?
1. The restraints/seclusion policies set forth by the institution.
2. The patient’s competence.
3. The patient’s voluntary/involuntary status.
4. The patient’s nursing care plan.

3

The nurse is caring for an 80-year-old client with Parkinson’s disease.
Which of the following nursing goals is MOST realistic and appropriate in
planning care for this client?
1. Return the client to usual activities of daily living.
2. Maintain optimal function within the client’s limitations.
3. Prepare the client for a peaceful and dignified death.
4. Arrest progression of the disease process in the client.

2

A client with a peptic ulcer had a partial gastrectomy and vagotomy
(Billroth I). In planning the discharge teaching, the client should be
cautioned by the nurse about which of the following?
1. Sit up for at least 30 minutes after eating.
2. Avoid fluids between meals.
3. Increase the intake of high-carbohydrate foods.
4. Avoid eating large meals that are high in simple sugars and liquids.

4

A nurse is caring for a 37-year-old woman with metastatic ovarian
cancer admitted for nausea and vomiting. The physician orders total
parenteral nutrition (TPN), a nutritional consult, and diet recall. Which of
the following is the BEST indication that the patient’s nutritional status has
improved after 4 days?
1. The patient eats most of the food served to her.
2. The patient has gained 1 pound since admission.
3. The patient’s albumin level is 4.0mg/dL.
4. The patient’s hemoglobin is 8.5g/dL.

3

A 23-year-old woman at 32-weeks gestation is seen in the outpatient
clinic. Which of the following findings, if assessed by the nurse, would
indicate a possible complication?
1. The client’s urine test is positive for glucose and acetone.
2. The client has 1+ pedal edema in both feet at the end of the day.
3. The client complains of an increase in vaginal discharge.
4. The client says she feels pressure against her diaphragm when the baby moves.

1

After abdominal surgery, a client has a nasogastric tube attached to low
suctioning. The client becomes nauseated, and the nurse observes a
decrease in the flow of gastric secretions. Which of the following nursing
interventions would be MOST appropriate?
1. Irrigate the nasogastric tube with distilled water.
2. Aspirate the gastric contents with a syringe.
3. Administer an antiemetic medicine.
4. Insert a new nasogastric tube.

2

After sustaining a closed head injury and numerous lacerations and
abrasions to the face and neck, a five-year-old child is admitted to the
emergency room. The client is unconscious and has minimal response to
noxious stimuli. Which of the following assessments, if observed by the
nurse three hours after admission, should be reported to the physician?
1. The client has slight edema of the eyelids.
2. There is clear fluid draining from the client’s right ear.
3. There is some bleeding from the child’s lacerations.
4. The client withdraws in response to painful stimuli.

2

The nurse is caring for a manic client in the seclusion room, and it is
time for lunch. It is MOST appropriate for the nurse to take which of the
following actions?
1. Take the client to the dining room with 1:1 supervision.
2. Inform the client he may go to the dining room when he controls his behavior.
3. Hold the meal until the client is able to come out of seclusion.
4. Serve the meal to the client in the seclusion room.

4

A client is given morphine 6 mg IV push for postoperative pain.
Following administration of this drug, the nurse observes the following:
pulse 68, respirations 8, BP 100/68, client sleeping quietly. Which of the
following nursing actions is MOST appropriate?
1. Allow the client to sleep undisturbed.
2. Administer oxygen via facemask or nasal prongs.
3. Administer naloxone (Narcan).
4. Place epinephrine 1:1,000 at the bedside.

3

What type of infectious diseases are required to be reported to the health department?

– severe cases of Staphylococcus aureus infections including methicillin-resistant Staphylococcus aureus (MRSA)

What is the process of taking a telephone order from a provider?

Patient name, drug, dose, route, frequency
read back for accuracy

A nurse is caring for a client who has tuberculosis. Which of the following actions should the nurse take? SATA

a) Place the client in a negative pressure room
b) wear gloves when assisting the client with oral care
c) limit each visitor to 2 hr increments
d) wear a surgical mask when providing care
e) Use antimicrobial sanitizer for hand hygiene

A
B
E

A charge nurse is discussing the responsibility of nurses carig for clients who have C. difficile. Which of the following information should the nurse include in the teaching?

a) Assign the client to a room with a negative air-flow system
b) Use alcohol-based hand sanitizer when leaving the clients room
c) clean contaminated surfaces in the clients room with a phenol solution
d) have family members wear a gown and gloves when visiting

D

A nurse is caring for a client receiving IV fluids. During a routine check, the nurse determines that the client has developed phlebitis and removes the IV catheter. Which of the following actions should the nurse take next?

a) place a warm compress over the IV site
b) record the findings in the client’s chart
c) notify the client’s primary care provider
d) prepare to insert a new IV catheter

A

A nurse is caring for a client who has dementia. Which of the following interventions should the nurse take to minimize the risk for injury for this client?

a) use a bed exit alarm system
b) raise 4 side rails while client is in bed
c) apply one soft wrist restraint
d) dim the lights in the client’s room

A

A nurse is implementing a plan of care for a client who is at risk for falls. Which of the following is an appropriate nursing action?

a) implement a regular toileting schedule
b) encourage the client to wear athletic socks when ambulating
c) place all 4 bed rails in the upright position
c) require a family member to remain at the bedside

A

Which of the following techniques should the nurse use when performing nasotracheal suctioning for a client?

a) insert the suction catheter while the client is swallowing
b) apply intermittent suction when withdrawing the catheter
c) place the catheter in a location that is clean and dry for later use
d) hold the suction catheter with the clean, non-dominant hand

B

A nurse is caring for a client following an acute myocardial infarction. The client is concerned that providing self-care will be difficult due to extreme fatigue. Which of the following strategies should the nurse implement to promote the client’s independence?

a) request an occupational therapy consult to determine the need for assistive devices
b) assign assistive personnel to perform self-care tasks for client
c) instruct the client to focus on gradually resuming self-care tasks
d) ask the client if a family member is available to assist with his care

C

A nurse is reviewing the medical records of a client who has a pressure ulcer. Which of the following is an expected finding?

a) serum albumin level of 3 g/dL
b) HDL level of 90 mg/dL
c) Norton scale score of 18
d) Braden scale score of 20

A

A nurse is caring for a client who needs a 24-hr urine collection initiated. Which of the following client statements indicates an understanding of the procedure?

a) “I had a bowel movement, but I was able to save the urine”
b) “I have a specimen in the bathroom from about 30 minutes ago”
c) “I flushed what I urinated at 7 am and have saved the rest since”
d) “I drink a lot, so I will fill up the bottle and complete the test quickly”

C

A nurse is caring for a client who has an NG tube that is to be irrigated every 8 hr. Which of the following should be used to irrigate the tube in order to maintain fluid and electrolyte balance?

a) tap water
b) sterile water
c) 0.9% sodium chloride
d) 0.45% sodium chloride

C

A nurse is reinforcing teaching regarding the use of a cane to a client who has left-leg weakness. Which of the following should the nurse include in the teaching?

a) use the cane on the weak side of the body
b) advance the cane and the atrong leg simultaneously
c) maintain two points of support on the floor
d) advance the cane 30 to 45 cm (12-18 in) with each step

C

Which of the following should indicate to a nurse the need to suction a client’s tracheostomy?

a) irritability
b) hypotension
c) flushing
d) bradycardia

A

A nurse is caring for a client who has a prescription for wound irrigation. Which of the following actions should the nurse take?

a) wear sterile gloves when removing the old dressing
b) warm the irrigation solution to 40.5C (105F)
c) cleanse the wound from the center outwards
d) use a 20 mL syringe to irrigate the wound

C

A nurse is providing teaching about a clear liquid diet. Which of the following should the nurse instruct the client to avoid?

a) lemon-lime sports drinks
b) ginger ale
c) black coffee
d) orange sherbet

D

A nurse is caring for a client who is having difficulty voiding following the removal in an indwelling urinary catheter. Which of the following interventions should the nurse take?

a) assess for bladder distention after 6 hr
b) encourage the client to use a bed pan in the supine position
c) restrict the clients intake of oral fluids
d) pour warm water over the clients perineum

D

When caring for the client diagnosed with delirium, which condition is the most important for the nurse to investigate?
1. Cancer of any kind.
2. Impaired hearing.
3. Prescription drug intoxication.
4. Heart failure.

3

Which of the following is essential when caring for a client who is experiencing delirium?
1. Controlling behavioral symptoms with low-dose psychotropics.
2. Identifying the underlying causative condition or illness. 3. Manipulating the environment to increase orientation.
4. Decreasing or discontinuing all previously prescribed medications.

2

Which of the following is a realistic short-term goal to be accomplished in 2 to 3 days for a client with delirium?
1. Explain the experience of having delirium.
2. Resume a normal sleep-wake cycle.
3. Regain orientation to time and place.
4. Establish normal bowel and bladder function.

3

A client diagnosed with dementia wanders the halls of the locked nursing unit during the day. To ensure the client’s safety while walking in the halls, the nurse should do which of the following?
1. Administer PRN haloperidol (Haldol) to decrease the need to walk.
2. Assess the client’s gait for steadiness.
3. Restrain the client in a geriatric chair.
4. Administer PRN lorazepam (Ativan) to provide sedation.

2

During a home visit to an elderly client with mild dementia, the client’s daughter reports that she has one major problem with her mother. She says, “She sleeps most of the day and is up most of the night. I can’t get a decent night’s sleep anymore.” Which suggestions should the nurse make to the daughter? Select all that apply.
1. Ask the client’s physician for a strong sleep medicine. 2. Establish a set routine for rising, hygiene, meals, short rest periods, and bedtime.
3. Engage the client in simple, brief exercises or a short walk when she gets drowsy during the day.
4. Promote relaxation before bedtime with a warm bath or relaxing music.
5. Have the daughter encourage the use of caffeinated beverages during the day to keep her mother awake.

2
3
4

The physician orders risperidone (Risperdal) for a client with Alzheimer’s disease. The nurse anticipates administering this medication to help decrease which of the following behaviors?
1. Sleep disturbances.
2. Concomitant depression.
3. Agitation and assaultiveness.
4. Confusion and withdrawal.

3

The nurse is making a home visit with a client diagnosed with Alzheimer’s disease. The client recently started on lorazepam (Ativan) due to increased anxiety. The nurse is cautioning the family about the use of lorazepam (Ativan). The nurse should instruct the family to report which of the following significant side effects to the health care provider?
1. Paradoxical excitement.
2. Headache.
3. Slowing of reflexes.
4. Fatigue.

1

When providing family education for those who have a relative with Alzheimer’s disease about minimizing stress, which of the following suggestions is most relevant?
1. Allow the client to go to bed four to five times during the day.
2. Test the cognitive functioning of the client several times a day.
3. Provide reality orientation even if the memory loss is severe.
4. Maintain consistency in environment, routine, and caregivers

4

What are some ways to identify a patient before giving a medication?

The Joint Commission requires 2 client identifiers be used when administering medications.
– clients name
– assigned identification number
– telephone number
– birth date or other personal-specific identifiers. Bar code scanners may be used to identify clients

What are some things to teach about home safety with elderly patients?

– Removing items that could cause the client to trip, such as throw rugs and loose carpets
– Placing electrical cords and extension cords that against a wall behind furniture
– Making sure that steps and sidewalks are in good repair
– Placing grab bars near the toilet and in the tub or shower and installing a stool riser
– Using a non-skid mat in the tub or shower
– Placing a shower chair in the shower
– Ensuring that lighting is adequate both inside and outside of the home

A nurse is providing discharge instructions to a client who has a prescription for the use of oxygen in
his home. Which of the following should the nurse teach the client about using oxygen safely in his
home? (Select all that apply.)
A. Family members who smoke must be at least 10 ft from the client when oxygen is in use.
B. Nail polish should not be used near a client who is receiving oxygen.
C. A “No Smoking” sign should be placed on the front door.
D. Cotton bedding and clothing should be replaced with items made from wool.
E. A fire extinguisher should be readily available in the home.

B
C
E

A nurse is providing home safety instructions to a group of older adult clients. Match the safety risk
with the appropriate instruction.
____ Passive smoking
____ Carbon monoxide poisoning
____ Food poisoning
A. Have water heaters inspected on an annual
basis.
B. Cook all meat at an appropriate temperature.
C. Avoid enclosed areas with others who may be
smoking.

C
A
B

When performing nasotracheal suctioning what technique should be used?

Sterile asepsis bc the trachea is considered sterile and prevents infections

A nurse educator is presenting a module on basic first aid for newly licensed home health nurses. The nurse educator evaluates the teaching as effective when the newly licensed nurse states the client who has heat stroke will have which of the following?
A. Hypotension
B. Bradycardia
C. Clammy skin
D. Bradypnea

A

What do you do when a client has a seizure

– lower to bed/floor
– protect head, move nearby furniture, provide privacy, – – put on side with head flexed slightly forward, and loosen clothing to prevent injury
-in event of seizure, stay with client and call for help
-admin meds as ordered
-note duration of seizure and sequence and type of movement

seclusion and restraints

-must be ordered
-should be ordered for the shortest duration necessary and only if less restrictive measures are not sufficient
-a client may voluntarily request temp seclusion
-restraints can be physical or chemical
-if used, frequency of client assessments in regards to food, fluid, comfort, and safety should be performed and documented every 15-30 min

What position is good to use for a patient who is at high risk for a pressure ulcer

30 degree lateral position is recommended for clients at risk for pressure ulcers

health promotion (injury prevention-suffocation): infant (birth-1 yr)

-avoid plastic bags
-keep balloons out of reach
-ensure crib mattress fits snugly
-ensure crib slats are no more than 6 cm (2.4 in) apart
-remove crib mobiles and gyms by 4-5 months
-do not use pillows in crib
-place infant on back for sleep
-keep toys with small parts out of reach
-remove drawstrings from jackets and other clothing

hypotension is classified with a reading below normal;

systolic < 90 mm Hg; can be a result of fluid depletion, heart failure, or vasodilation

What temperature should pork be cooked at

160 degrees

What is the safest way to thaw out frozen foods

In the refrigerator

What are the precautions for vancomycin resistant enterococcus

Standard precautions including hand washing and gloving should be followed

What does a newborns poop look like

If your baby is exclusively breastfed, her poop will be yellow or slightly green and have a mushy or creamy consistency

What is appropriate for an adolescent in the hospital?

Puzzles and books

What is the proper nutrition during pregnancy

– Folic acid is important for pregnancy, as it can help to prevent birth defects known as neural tube defects, including spina bifida
– green leafy vegetables and brown rice

What should be avoided during pregnancy

Do not take vitamin A supplements, or any supplements containing vitamin A (retinol), as too much could harm your baby

What is the most appropriate method for contraception for an adolescent

IUD or implant

If a patient has anorexia nervosa and works out constantly

Allow them to workout and continue their regimen

What medications can be taken to help with smoking cessation

Bupropion hydrochloride is a medicine for depression, but it also helps people quit smoking. Brand names include Zyban®, Wellbutrin®, Wellbutrin SR® and Wellbutrin XL® but this medication is also available as a generic. Varenicline (chantix)

What are the five stages of grief

denial
anger
bargaining
depression
acceptance

discrete and applies the letting go of an object or person before the loss as in the case of terminal illness
individuals have the opportunity to greet before the actual loss

anticipatory grief

involves difficult progression through the expected stages of the grieving process
grief work is prolonged and manifestations more severe
client may develop suicidal ideation, intense feelings of guilt and lowered self-esteem
somatic complaints persist for an extended period of time

dysfunctional grief

Signs for meningococcemia

Vomiting, febrile, petechial rash
(unstable)

Levothyroxine effects

Used to restore client’s metabolic rate
* Toxic effects = heat intolerance, Tachycardia, Weight loss, Hypertension

Multiple Sclerosis Patient

Mitoxantrone SE’s Mitoxantrone IV every 3 months (chemo drug)
* Report Sore Throat
(greatest risk for client is severe infection due to myelosuppression from mitoxantrone)
* Vomiting = causes dehydration
* Hair Loss = emotional distress
* Amenorrhea = emotional distress

Malnourished COPD patients

(1) Limit liquid intake at meal times
(2) Consume foods w/ protein (like eggs)
(3) Maintain an upright position (High Fowler’s position) to promote ventilation
(4) Use milk instead of water when making soup

Which grief process is it when Client exhibits increased anxiety + may project anger toward self + others
“I don’t deserve to die, this isn’t fair”

Anger stage

Which Grief Process when Client acknowledges the impending loss while remaining hopeful
“If I could just make it through this, I’d never smoke again”

Bargaining Stage

How should you respond when client wants to discontinue dialysis

“What has changed to make you decide this?”
= Seek clarification from client to establish mutual understanding while staying therapeutic

What should the nurse do when one member of a support group expresses anger repeatedly?

Focus more on the group members who have a positive outlook
(Speak to group member privately to uncover source of anger)

What immunizations are CONTRAINDICATED for pregnant women + which SHOULD be given?

Contraindicated = Herpes Zoster + Varicella + MMR (measles, mumps, rubella)

Should give = TDaP (Tetanus, Diphtheria, Pertussis)

Long term effects of NSAIDS (Ibuprofen)

Gastric Ulcerations, perforations, hemorrhage, hypertension

Alcohol Use Manifestations of Withdrawal

Body burns 0.5 oz of alcohol per hour
* Withdrawal appears within 4-12 hours
* Irritability + Tremors + Anxiety
* Nausea + Vomiting + HA
* Diaphoresis
* Sleep Disturbances
* TACHYCARDIA + HTN

Use Benzodiazepines = tx
Diazepam (Valium), lorazepam (Ativan), and chlordiazepoxide (Librium)

When does Discharge planning begin?

At Admission

Case Management nursing involves:

*Decreasing cost by improving client outcomes
* Providing education to optimize health participation
* Advocating for services + client’s rights

What is bipolar disorder?

Bipolar disorder is a mood disorder with recurrent episodes of depression and mania.

What comorbidities may be observed with a patient who is bipolar?

Substance use disorder (experiences more rapid cycling), anxiety disorders, eating disorders, ADHD.

What therapy will be useful for patients with bipolar?

Electroconvulsive therapy for the patient who is suicidal or rapid cycling who HAS taken Lithium and has proven ineffective. Used to subdue manic behavior.

What kind of medications are indicated for abstinence maintenance of alcohol?

Disulfiram (Antabuse), Naltrexone (Vivitrol), Acamprosate (Campral)

Teaching points for naltrexone (Vivitrol)?

Take with meals to supress GI distress. Monthly IM injections should be suggested for patients who have difficulty to adhering to the medication regimen.

A nurse is caring for a client who underwent a subtotal gastrectomy. To manage dumping syndrome, the nurse should advise the client to:
a) restrict fluid intake to 1 qt (1,000 ml)/day.
b) drink liquids only between meals.
c) don’t drink liquids 2 hours before meals.
d) drink liquids only with meals.

B

A patient who has undergone colostomy surgery is experiencing constipation. Which of the following interventions should a nurse consider for such a patient?
a) Instruct the patient to keep a record of food intake
b) Instruct the patient to avoid prune or apple juice
c) Suggest fluid intake of at least 2 L per day
d) Assist the patient regarding the correct diet or to minimize food intake

C

A client is admitted with a diagnosis of acute appendicitis. When assessing the abdomen, the nurse would expect to find rebound tenderness at which location?
a) Left lower quadrant
b) Left upper quadrant
c) Right upper quadrant
d) Right lower quadrant

D

Which outcome indicates effective client teaching to prevent constipation?
a) The client reports engaging in a regular exercise regimen.
b) The client limits water intake to three glasses per day.
c) The client verbalizes consumption of low-fiber foods.
d) The client maintains a sedentary lifestyle.

A

Patients diagnosed with esophageal varices are at risk for hemorrhagic shock. Which of the following is a sign of potential hypovolemia?
a) Hypotension
b) Bradycardia
c) Warm moist skin
d) Polyuria

A

The nurse is assessing a client with a bleeding gastric ulcer. When examining the client’s stool, which of the following characteristics would the nurse be most likely to find?
a) Green color and texture
b) Black and tarry appearance
c) Clay-like quality
d) Bright red blood in stool

B

After teaching a group of students about the various organs of the upper gastrointestinal tract and possible disorders, the instructor determines that the teaching was successful when the students identify which of the following structures as possibly being affected?
a) Large intestine
b) Ileum
c) Stomach
d) Liver

C

A nurse is caring for a client with active upper GI bleeding. What is the appropriate diet for this client during the first 24 hours after admission?
a) Skim milk
b) Nothing by mouth
c) Regular diet
d) Clear liquids

B

Bladder retraining for the treatment of urge incontinence:

• Use timed voidings to increase intervals between voidings/decrease voiding frequency.
• Perform pelvic floor (Kegel) exercises.
• Perform relaxation techniques.
• Offer undergarments while the client is retraining.
• Teach the client not to ignore the urge to void.
• Provide positive reinforcement as client maintains continence.
• Eliminate or decrease caffeine drinks.
• Take diuretics in the morning.

what are normal creatinine levels?
what are normal BUN levels?

0.8-1.4 mg/dL
8-25 mg/dL

What are total serum protein values (normals)

6-8 g/dL

Describe pre-albumin

this is the best tool for evaluating nutrition. it has a half-life of 2 days which is much shorter than albumin so it is much more accurate. (albumin’s half-life is 2-3 weeks)

what is normal pre-albumin values?
what are normal serum levels of magnesium ?
what is a normal potassium serum level?

17-40 mg/dL
1.5-2.5 mEq/L (less than 1.5 is considered hypomagnesemia)
3.5-5.0 mEq/L (less than 3.5 is considered hypokalemia)

what are good sources of folic acid?

Excellent sources of folate include romaine lettuce, spinach, asparagus, turnip greens, mustard greens, calf’s liver, parsley, collard greens, broccoli, cauliflower, beets, chicken liver and lentils.

Sources of potassium

beans, spinach, potatoes, dried apricots, acorn squash, yogurt, salmon, avocados, mushrooms and bananas

what is important about the diet of someone taking ACE inhibitors?

can result in high potassium levels. Limit potassium intake (beans, spinach, potatoes, dried apricots, acorn squash, yogurt, salmon, avocados, mushrooms and bananas)

Taking Coumadin. Which foods should the client limit?

Foods containing Vitamin K. Dark leafy greens (spinach), brussel sprouts, broccoli, asparagus, cabbage, pickels, prunes

what is a normal hematocrit level in a female?
What are normal Hgb values (female)?
what are normal values for WBCs?

37-48% (male is 42-52%)
12-16 g/dL (male 13-17)
4500-11,000 / uL

what foods should you avoid if you have diverticulitis?

avoid hard-to-digest foods such as nuts, corn, popcorn, and seeds, for fear that these foods would get stuck in the diverticula and lead to inflammation. (Eat foods high in fiber)

When taking MAOI’s, limit your consumption of

thyramine–it can cause elevated BP. This is found in “aged” products such as aged cheeses (swiss), cured meats (pepperoni/salomi), sauerkraut, soy sauce…Examples of MAOI’s are: Isocarboxazid (Marplan), Phenelzine (Nardil), Selogilive, Emsam, Eldepryl, Zelapar…

At what age does bone loss begin with osteoporotis
what are normal Calcium levels?

at age 35 (women)
8.6-10 mg/dL

A positive Chvosteks sign is found in a patient. The nurse would anticipate IV administration of

calcium gluconate (because hypocalcemia causes Chvostek’s sign)

What are the S/S of lithium toxicity?
(depakote for bipolar disorder)

fine hand tremors, mild GI upset, slurred speech and muscle weakness

a nurse is obtaining a medication history from a client who is to start a new prescription for warfarin ( Coumadin) . which of the following over the counter medication should the nurse instruct the client to avoid

Aspirin

a nurse responsible for a client receiving a antihypertensive medication is to

teach the client to change position slowly to avoid dizziness or fainting

a client should receive a dose of flumazenil ( romazicon) to treat symptoms of

benzodiazepine overdose

a nurse is reinforcing teaching to a client who is prescribed diazepam tor anxiety of the following statement indicated the client understand the teaching

I will tell my doctor before I stop taking the medication

a nurse is reinforcing teaching to a client who is starting amitriptyline ( Elavil) for treatment of depression which of the following should the nurse include

1. change position slowly to minimize dizziness
2. chewing sugarless gum to prevent dry mouth

a client who is start taking lithium carbonate month ago tell the nurse she has just begun taking multiply daily doses of ibuprofen ( motrin) for tension headache. should the client avoid ibuprofen. why or why not ?

what , if any is the appropriate action for the nurse to take NSAIDS such as ibuprofen increase the renal reabsorption of lithium carbonate , possibly leading to lithium carbonate toxicity . therefor this client would avoid NSAIDS . the nurse should notify the provider of client headache and ibuprofen us

a client has prescription for valproic ( Depakote) which of the following laboratory value should the nurse anticipate monitor for the client taking this medication

thrombocytes, amylase count and liver function test

alcohol withdrawal
heroin withdrawal
nicotine withdrawal
alcohol abstinence
opioid over dose

chlordiazeproxide( Librium)
methadone( dolophine)
bupropion ( wellbutrin)
disulfiram ( antabuse)
naloxone (narcan)

a client who has parkinson’s disease is prescribed levodopa/carbidopa ( sinemet) and pramipexole ( Mirapex) for which of the following should the nurse monitor this client

orthostatic hypotension

a nurse is preparing to care for a client in the surgical unit who will be receiving lorazapam ( ativan IV) . for what adverse effect should the nurse monitor this client

the nurse should monitor the client respiratory depression

a client has a new prescription for spironilactone ( aldactone ) which of the following laboratory value should the nurse recognized as a reason to withhold the morning dose of the medication and notify the provider

serum potassium 5.2

a nurse is caring for a client who prescribed daily dose of both digoxin ( llanoxin ) and furosemide ( Lasix) . the client potassium level 3.2 mEq/L for which of the following medication interaction is the client at risk

Toxic level of digoxin

a nurse is reinforcing a teaching on a client who has a prescription for verapamil ( calan) which of the following statement by the client indicated need further teaching

i should decrease the amount of calcium in my diet while taking the medication

A nurse is caring for an older adult client who ahs a new prescription for digoxin and takes multiple other medications. Concurrent use of which of the following medications places the client at risk for digoxin toxicity?

* Verapamil (Calan)

Adverse effect of Verapamil

Avoid grapefruit juice

Interaction of diuretics and ACE inhibitors

excessive reduction in blood pressure and symptomatic hypotension or hyperkalemia

What can prevent MI, stroke, or death in high-risk patients

Ramipril

What to monitor for when taking enoxaparin (lovenox)

Hyperkalemia

Cases of headache, hemorrhagic anemia, eosinophilia, alopecia, hepatocellular and cholestatic liver injury reported

What are the therapeutic effects of protamine

Antidote to severe heparin overdose + Reversal of heparin administered during procedures

How to prevent adverse effects of oxycodone

can cause respiratory depression.

What is the nursing intervention and/or client education ? Monitor vital signs.
› Stop opioids for respiratory rate less than 12/min, and notify the provider.
› Have naloxone and resuscitation equipment available.
› Avoid use of opioids with CNS depressant medications (barbiturates,
benzodiazepines, consumption of alcohol).

opioid agonists can cause Constipation

What is the nursing intervention and/or client education ?

Advise the client to increase fluid/fiber intake and physical activity.
› Administer a stimulant laxative such as bisacodyl (Dulcolax) to counteract
decreased bowel motility, or a stool softener such as docusate sodium (Colace)
to prevent constipation.

Adverse effects of ferrous sulfate

constipation;
upset stomach;
black or dark-colored stools; or.
temporary staining of the teeth.

Baclofen (Lioresal) therapeutic outcome:

Decrease the frequency and severity of muscle spasms (MS).

What is the difference between respiratory acidosis and respiratory alkalosis?

Acidosis refers to an excess of acid in the blood that causes the pH to fall below 7.35, and alkalosis refers to an excess of base in the blood that causes the pH to rise above 7.45.

Bowel elimination how to get a specimen collection

Collect stool specimens for serial fecal occult blood (guaiac) testing 3 times from 3 different defecations. Stool samples should come from fresh stools that are not contaminated with water or urine.

Identifying manifestations of transient ischemic attacks

symptoms r/t afffected area. Rapid onset of weakness, numbness, aphasia, visual field cuts. 1-2 clusters before stroke.

Musculoskeletal congenital disorders

Monitor skin for breakdown areas and prevent pressure sores.

The nurse caring for a child in Buck’s skin traction will keep the:

Child pulled up in bed

Where should the cath bag be placed when urinary catheterization

Make sure the catheter bag/system is at a level below the client’s bladder to avoid reflux.

What are the signs and symptoms of fluid volume deficit

loss of total body Na. Causes include vomiting, excessive sweating, diarrhea, burns, diuretic use, and kidney failure. Clinical features include diminished skin turgor, dry mucous membranes, tachycardia, and orthostatic hypotension.

What is the nursing action for dehiscence

Cover with a sterile towel moistened with sterile saline; Have patient flex knees slightly and put in Fowler’s .

A nurse is caring for a client following an electroconvulsive therapy. What nursing action?

Continuously reorient client after ECT

A nurse is caring for a client who is taking multiple medications about the possible interactions. Which of the following members of the interdisciplinary team should the nurse make a referral to?

Advance practice nurse

A nurse manage is taking measures to increase cost effective. In which of the following client situations should the nurse manager intervene?

A nurse uses sterile gauze pads to place a dressing on a surgical wound.

A nurse is contributing to the plan of care for client who has ascites due to cirrhosis. Which of the following interventions should the nurse recommend including in the plan?

Measure the client’s abdominal girth daily

A nurse in an urgent care clinic is caring for a client who reports recently using methylenedioxy-methamphetamine. Which of the following findings should the nurse expect?

Hallucinations

A nurse preparing a sterile field to perform a dressing change for a client’s leg wound. Which of the following actions should the nurse take?

Place sterile objects at least 2.5 cm from the edge of the sterile field.

A nurse is reinforcing teaching with a client who has a new rx for transdermal nitroglycerin patches. Which of the following statements by the client indicates an understanding of the teaching?

I will place the patch on a hairless area of skin

A nurse is reinforcing with a new mother on facility security measures. Which of the following statements by the mother indicates an understanding of the teaching?

I will have an ID band that matches the one my baby wears

• A nurse begins to bathe a newly admitted client who reports that she has not had anything to eat that day, the nurse interrupts the bath and obtains a healthy meal for the client. This action by the nurse is an example of which of the following?

Boundary crossing

A nurse enters the room of a school aged child and finds him on the floor experiencing a tonic colonic seizure, which of the following actions should the nurse take?

Place a pillow under the client’s head

A nurse is providing support to the family of a school aged child who has a new diagnosis of a terminal illness, which of the following actions should the nurse take?

Provide the family with time to talk openly

A nurse is checking for gastric residual volume on the client who is scheduled to receive an intermittent enteral feeding. The nurse finds that the client has a GRV greater than 250 mLs on two consecutive measurements, which of the following actions should the nurse take?

Hold the feeding and recheck the GRV in one hour

A nurse in a clinic is reviewing the medical record of a toddler and notes that he previously had a severe allergic reaction to a gelatin, which of the following immunizations should the nurse identify as unsafe for this client to receive?

Measles Mumps and Rubella (MMR)

A nurse is reinforcing teaching with a client who has a prescription for antibiotic therapy. The client tells the nurse that he always experiences diarrhea when taking antibiotics, which of the following food choices should the nurse recommend to lessen the occurrence of diarrhea?

Yogurt

A home health nurse is caring for a client who has Alzheimer’s disease. the client’s son is concerned about his mother becoming frustrated. which of the following interventions should the nurse include?

make a schedule for daily tasks

A nurse is caring for an adolescent client who has bulimia nervosa, which of the following actions should the nurse take?

Observe the client during and after meals

A nurse is participating in a performance improvement program which of the following actions should the nurse take to evaluate the effectiveness of the program?

perform chart audits

A nurse is talking to a client who refuses a blood transfusion for religious reasons. which of the following responses should the nurse make?

you have the right to change your mind

A nurse is assisting with staff education about hand hygiene. which of the following should the nurse include in the teaching?

wash your hands with soap and water for 20 seconds

A nurse is assisting with a community health course for family members of clients who half Alzheimer’s disease. which of the following should the nurse include?

limit the number of choices for the client

• A nurse in a provider’s office is collecting data from an 18-month-old toddler which of the following should the nurse identify as a potential indication of child mal-treatment?

the toddler is overly affectionate towards the nurse

A nurse is contributing to the plan of care for a client who has ascites due to chromosols, which of the following should the nurse recommend including in the plan?

measure the client’s abdominal girth

A nurse is caring for a client who is experiencing alcohol withdrawal the nurse should prepare to administer which of the following medications?

diazepam

A nurse is preparing to delegate care to assistive personnel. Which of the following information should the nurse verify prior to delegation?

The AP’s job description

A nurse is caring for a client who has paranoid schizophrenia and believes that she is being followed by the FBI agents who are pretending to be psychiatric staff. Which of the following responses should the nurse make?

What makes you think the staff is following you?

A nurse is reinforcing teaching with a client who has GERD and a prescription for ranitidine. Which of the following statements by the client indicates an understanding of the teaching?

I should take this medication in the morning and at bedtime

A nurse is reinforcing teaching with new parents about car seat safety. Which of the following instructions should the nurse include?

Position the car seat at 90-degree angle

A nurse is collecting data from a client who is 2 days postpartum. The client tells the nurse she cannot afford to pay for baby formula. The nurse should refer the client to which of the following members of the intra-professional team?

Case manager

A community health center is assisting in the development of a brochure about hypertension. Which of the following action should the nurse take?

explain medical terminology using basic one syllable words

A community health nurse is assisting with the development of a pamphlet regarding choking hazard for toddlers. Which of the following foods should the nurse include?

grapes

A nurse is caring for a who has a new diagnosis of diabetes mellitus and is refusing to learn how to self-administer insulin. Which of the following responses should the nurse make?

“I’d like to hear your thoughts about giving yourself this medication”

A nurse is implementing a bladder training program for a client who had a stroke. Which of the following interventions should the nurse make?

check for residual after voiding

A nurse is preparing change of shift report on a client who is 2 days postoperative following a total knee arthroscopy (or arthroplasty). Which of the following information about the client should the nurse include in the report?

time of last pain medication

A nurse is caring for a client who is 2 days following an above-the-knee amputation. Which of the
following actions should the nurse take to promote progression to independence and mobility for
the client?

Encourage the client to use the overhead trapeze

. A nurse at a pediatric clinic is checking the vital signs of a 2 month old infant prior to
administering immunization. The nurse should recognize respiratory rate is 30/min. This indicates
that the respiratory rate is?

🍰 Within the expected range

A nurse in a clinic is collecting data with a client who had a vaginal birth 6 weeks ago. Which of
the following findings should the nurse expect?

🍰 Reported Lochia Alba

A nurse is reinforcing discharge teaching with a family of a client who has dependent personality
disorder. What action should the nurse take?

🍰 Encourage the client to be assertive

A charge nurse in the long-term care facility notices the smell of alcohol from one of the nurse’s
breath. Which oc the following actions should the nurse take first?

🍰 Remove the nurse from the client care area

A nurse is supervising an assistive personnel. Which of the following actions by the AP indicates
an effective use of client care materials?

🍰 Wears clean gloves when assisting client with oral care

A nurse is preparing to administer spironolactone and lisinopril to a client who reports anxiety,
dyspnea, and weakness of the leg. Which of the following actions should the nurse take first?

🍰 Withhold the prescribed medication

A nurse is assisting with the care of a client who is receiving chemotherapy and radiation for
advanced breast cancer. The client states, “I’m

🍰 “Tell me more about what you are thinking.”

A nurse is RT with a client who is 35 weeks gestation who will undergo a contraction stress test.
WOTF information should the nurse include in the teaching?

🍰 You will lie flat on your back for the test

A nurse is collecting data from a client who represents the facility and who reports vomiting.
WOTF findings indicate that the client is experiencing Fluid Volume deficit?

🍰 Orthostatic hypotension

A nurse is reinforcing dietary teaching for a client who is 37 weeks gestation and has gestational
hypertension. WOTF foods should the nurse recommend? (SATA)

– raw apple
– cooked carrots

A client who has cancer is being discharged to home with hospice services. The client has a
prescription for oxycodone for pain control. WOTF medications should the nurse remind the client
to take regularly to prevent a common adverse effect of this medication?

🍰 Docusate Sodium (stool softener)

A nurse is caring for an adolescent client who has been hospitalized for several weeks. WOTF
actions should the nurse take relative to the client’s developmental stage?

Arrange for uninterrupted visitations with friends

A nurse is RT with a parent of a preschool age child who has a new diagnosis of varicella. WOTF
statements indicate an understanding of the teaching?

🍰 I will apply an antipruritic cream to my child’s skin

A nurse is caring for a client who is experiencing urge incontinence and limited dexterity. WOTF
actions should the nurse take?

🍰 Establishing urinary habit program

A nurse is reinforcing teaching with a client about intermittent catheterization to measure residual
urine. WOTF information should the nurse include in the teaching?

You will need to urinate before

A nurse at a long-term care facility is reviewing the PoC of a client who has a prescription for
mitten restraints. WOTF tasks does the nurse assign to the assistive personnel?

🍰 Assist with ROM of the hand

A nurse is RT with a client who has primary open-angle glaucoma and a new prescription for
Timolol eye drops. WOTF statements by the client indicates an understanding of the teaching?

I should check my heart rate while taking this medication

A nurse is obtaining a health history from a client who wants to take oral contraceptives. WOTF
findings is the nurse’s priority?

smoking

A home health nurse is preparing to set up a continuous passive motion machine for a client who
is post-op following a total knee replacement. WOTF actions should the nurse respond to take
first?

Inspect the CPM machine and electrical cords for possible damage

A nurse is preparing to administer morphine 30mg to a client who reports pain. Available
morphine is 20mg/mL

1.5 mL

A nurse is RT about hand hygiene to a newly licensed nurse. WOTF information should the nurse
include in her teaching?

🍰 Interlace the fingers while rubbing hands together

A nurse is reinforcing dietary teaching with a client who experienced a myocardial infarction. The
nurse should encourage the client to limit WOTF?

🍰 Saturated fats

A nurse is RT with the family of a client who is terminally ill about the grief process. WOTF
information should the nurse include in the teaching?

🍰 Anger towards the health care staff is expected

A nurse is collecting data from a client who has diabetic Ketoacidosis. WOTF should the nurse
expect?

fruity breath

A nurse in the long term facility is caring for an older adult client who has a history of
hypertension. WOTF findings should the nurse recognize as an indicative sign of a Transient
Ischemic Attack?

🍰 Pain radiating down the left arm

28. A nurse is planning care for a client who recently attempted suicide. WOTF actions should the
nurse plan to take?

Ensure the client swallows each dose of medicine

29. A nurse is caring for a client who has upper gastrointestinal bleed. WOTF actions should the
nurse take first?

🍰 Check bowel sounds

30. A nurse is assisting with the admission of an adolescent client who is suspected of having
bacterial meningitis.

🍰 Nuchal rigidity

31. A nurse is transcribing a prescription from the provider. The prescription reads Gentamicin 2 gtt
OD q4h. WOTF information should the nurse clarify with the provider?

dosage

33. A nurse is reinforcing teaching with a parent of a child who has a new prescription for ferrous
sulfate in liquid form. WOTF statements by the parent indicates an understanding of the
teaching?

🍰 I will give the iron through a straw

34. A nurse is collecting data from a client who has cholecystitis. WOTF findings should the nurse
expect?

🍰 Pain in the right upper abdomen

35. A nurse is caring for a 3 year old child at a well-child visit. The parents report that their son has
been playing with imaginary friends. WOTF responses should the nurse make?

🍰 This is a common behavior for children at this age

36. A nurse is assisting with the physical examination on a 7-year-old child. WOTF options should the
nurse take?

🍰 Explain the purpose of the equipment used

37. A nurse is delegating client care assignments. WOTF tasks should the nurse assign to the
assistive personnel?

🍰 Apply an antiembolic stocking for a client who is 2 days post-op

38. A nurse is caring for a client who had a femoral popliteal bypass graft 2 days ago. When
monitoring peripheral pulses, the nurse is unable to locate the pulses in the affected leg. WOTF
actions should the nurse take?

🍰 Notify the charge nurse about the findings

39. A nurse is caring for a client who has multiple sclerosis and has a new prescription for baclofen.
WOTF indicates that the medication is having its desired effect?

🍰 Improved dexterity

40. A nurse is assisting with the prenatal examination of a client who is at 8 weeks gestation. The
nurse notices a purplish color of the vagina

🍰 Chadwick’s sign

42. A charge nurse is observing a newly licensed nurse perform suction on a tracheostomy. Which of
the following should the charge nurse intervene?

suctioning for 30 seconds

44. A nurse is reinforcing discharge teaching to the family of the client who has Alzheimer’s Disease.
The client has wandering behaviour and the family is concerned about safety at home. Which of
the following instructions should the nurse include?

🍰 Replace door knobs with ones containing simple locks

45. A nurse is reinforcing teaching with a mother who is breastfeeding her 3 week old newborn.
Which of the following instructions should the nurse include in the teaching?

🍰 Feed the newborn every 2-3 hours

46. A nurse is collecting data from a child during a well child visit. Which of the following actions
should the nurse take when checking the child’s oculomotor function?

🍰 Pupils are equal and react to light.

47. A nurse is caring for a client who delivered by C-section 1 day ago. The client requests non
pharmacological interventions to manage pain when changing position. Which of the following
responses should the nurse make?

🍰 Use breathing techniques when changing positions

48. A nurse is caring for a client who has a new diagnosis of heart failure and has a new prescription
for digoxin. The nurse should monitor which of the following for an adverse effect?

🍰 Visual Changes

49. A nurse is caring for a client who has anorexia nervosa. At home the client exercises frequently
each day. Which of the following interventions should the nurse implement?

throughout the day

50. A nurse is reinforcing teaching to a client who is to start taking ferrous sulfate elixir. Which of the
following statements shows client understanding of teaching?

I will mix medication with water

51. A charge nurse at Emergency Department recommends that a client should be discharged in
response to mass casualty disaster. Which of the following clients should the nurse recommend?

🍰 A client who is 3 days post-op of knee arthroplasty and requires knee rehabilitation

52. A nurse is caring for a mother who is breastfeeding her newborn 1 week after delivery. Which of
the following types of stools from the newborn should the nurse expect?

seedy

53. A nurse is reinforcing teaching about foot care of a client who has Diabetes. Which of the
following statements indicates an understanding of the teaching?

🍰 I should shake out my shoes before I put them on

54. A charge nurse is discussing about confidentiality of a client; about sharing the client’s medical
record. Which of the following people should the nurse identify as an appropriate person to share
client’s information with?

social worker

55. A nurse is assisting to monitor a client who has preeclampsia and is receiving an infusion of
Magnesium Sulfate. Which of the following findings should the nurse expect to stop the infusion
and report to the charge nurse?

🍰 Diaphoresis

56. A nurse is providing skin care for a 14-year-old adolescent who is in skeletal traction for a
fractured femur. Which of the following actions should the nurse take? Select all that apply.

🍰 Check skin every 2 hours
🍰 Cleanse skin with soap-free agents
🍰 Use a draw sheet to move client

57. A nurse is caring for a client who has prescription for Warfarin. What laboratory values should the
nurse check?

INR

58. A nurse in a mental health facility is observing two clients playing cards in the day room. One of
the clients becomes agitated and throws the cards on the ground. Which of the following actions
should the nurse take first?

🍰 Place the client into a quiet area

59. A nurse is reviewing the medical record for a child who is scheduled for a Varicella Immunization.
Which of the following findings should the nurse recognize as a contraindication.

🍰 Chemotherapy

60. A nurse is reinforcing teaching about sterile technique with a newly licensed nurse. The nurse
should include in the teaching. Which of the following procedures requires sterile technique?

🍰 Suctioning Tracheostomy

61. A nurse is reinforcing teaching to a client who has Hypertension about dietary measures to
control blood pressure. Which of the following food selection shows an understanding of
teaching?

🍰 Baked chicken

62. A nurse is caring for a client following an electroconvulsive therapy. What action should the nurse
monitor?

🍰 15 minutes disorientation

63. A nurse is reinforcing teaching with a client who is planning on becoming pregnant about the
dietary intake need to reduce the risk of neural tube defect. Which of the following foods is a
priority to include in the teaching?

cooked spinach

64. A nurse is caring for a client who has new prescription for ferrous sulfate. Which of the following
findings should the nurse monitor the client?

🍰 Gastrointestinal Distress

65. A nurse is providing care for a client who has cystic fibrosis. Which of the following effects is a
therapeutic effect of administering pancreatic enzymes?

🍰 Improved respiratory function

66. A nurse is reinforcing discharge teaching with a parent of a newborn. Which of the following
statements indicates an understanding of the teaching?

🍰 “I will fold the diaper just below my baby’s cord.”

67. A nurse is preparing to administer acetaminophen 50 mg PO to an infant. Available is 80 mg/0.8
mL. How many mL should the nurse administer to the infant?

0.5mL

68. A nurse is caring for a client who is in Buck’s traction. Which of the following actions should the
nurse take?

🍰 Ensure the weights are hanging freely

69. A nurse is caring for a client who is receiving continuous bladder irrigation following a
Transurethral resection of the prostate. Which of the following findings should the nurse report
immediately to the provider?

🍰 The nurse observes continuous bright, red urinary output

70. A nurse is collecting data from an adolescent client who is experiencing sexual abuse. Which of
the following shows that the client is using the defense mechanism of suppression?

🍰 “I drink beer as often as I can so I don’t have to think about it.”

72. A nurse is caring for a client who is taking multiple medications about the possible interactions.
Which of the following members of the interdisciplinary team should the nurse make a referral to?

🍰 Advance practice nurse

73. A nurse on the postpartum unit is reinforcing teaching with a client whose baby is 1 day old about
the development of sensory behavior. Which of the following statements by the client indicates an
understanding of teaching?

🍰 “My baby can hear my voice when I talk to him.”

74. A nurse is caring for a client who has an indwelling catheter for a urinary drainage. Which of the
following actions should the nurse take?

Coil the tube on the bed above the drainage bag.

75. A nurse is on a mental health unit is caring for a client who expresses anxiety. The nurse
promises to walk with the client everyday. Which of the following ethical principles is the nurse
displaying?

🍰 Fidelity

76. A charge nurse is monitoring a group of assistive personnel regarding the use of gloves in contact
isolation. Which of the following actions by the AP should the charge nurse intervene?

🍰 Remove gloves last when taking off PPE

78. A nurse is caring for a client who has an AV Shunt in his right arm. Which of the following actions
should the nurse take?

🍰 Check bruit over the shunt on a regular basis

79. A nurse is collecting data from a client who is 12 hour post-op following intestinal surgery. Which
of the following findings should the nurse report to the charge nurse prior to client ambulation?

🍰 Oral temperature of 99.7F

80. A nurse is collecting data from a client who is in his third trimester pregnancy, having her routine
prenatal visit. The client reports she is dizzy,clammy, and becomes pale lying down. Which of the
following actions should the nurse instruct the client to take?

🍰 Lie on her left side

81. A nurse is assisting a client move up in bed. Which of the following action should the nurse take?

🍰 Raise the head of the bed

82. A nurse is reinforcing teaching from a client who has a Urinary Tract Infection and a prescription
for antibiotic. Which of the following instructions should the nurse include in teaching?

🍰 Empty bladder every 4 hours.

83. A nurse is performing nasopharyngeal suctioning on an adult client. Which of the following
techniques should the nurse use?

🍰 Wait one minute between suctioning

84. A nurse is caring for a client who has a follow up visit for bipolar disorder and a new prescription
for Lithium Therapy. Which of the following indicates Lithium Toxicity?

🍰 Dysrhythmias

85. A nurse is reinforcing teaching for a 36 year old client who is 16 weeks gestation and who is
scheduled for amniocentesis. What is the purpose of amniocentesis?

🍰 To determine chromosomal abnormalities

86. A nurse in a long term care facility has just received change of shift. Which of the following clients
should the nurse attend to first?

🍰 Client who has a temperature of 101.4 and appears confused

87. A nurse is collecting data from a parent of a school age child who is allergic to bee venom. What
should the nurse administer?

🍰 Epinephrine (muscle relaxant)

88. A nurse is caring for a client who has increased ICP following a closed head injury. Which of the
following actions should the nurse take?

🍰 Elevate the Head of the Bed 30 degrees

89. A nurse is planning a health promotion program for high school students. Which of the following
about cigarette smoking would be the most appropriate to get the students to stop smoking?

🍰 Smoking cause unattractive stain on the teeth and hands

90. A nurse is preparing to administer intermittent enteral feeding to a client who has a small dour
NGT. Which of the following actions should the nurse plan to take?

🍰 elevated the head of bed to 45 degree

91. A nurse is caring for a preschool age child who recently experienced the death of a parent. Which
of the following should the nurse expect?

🍰 States death is punishment for bad behavior.

93. A nurse is contributing to the plan of care to a client who has delirium. Which of the following
interventions should the nurse recommend?

🍰 Assist the client to the bathroom every 2 hours.

94. A home health nurse is reinforcing teaching with a client about safe food handling to prevent food
poisoning. Which of the following instructions should the nurse include in the teaching?

🍰 Allow leftover food to cool before refrigerating

95. A nurse on a hospice unit is caring for a client that has cancer and is in the active phase of dying.
Which of the following findings indicates intervention from the nurse?

🍰 Assistive Personnel is encouraging intake of fluids

96. A nurse is caring for an adolescent who had a child and is requesting the most reliable method of
birth control. Which of the following should the nurse recommend?

🍰 Injectable progesterone

97. A client is collecting nutritional data from a group of adult clients. Which of the following clients
should the nurse recommend an interprofessional conference with a dietitian?

🍰 BMI of 32

98. A nurse is assisting a client to ambulate when the client begins to have a generalized seizure.
What action should the nurse take? (I don’t know the answer for this one)

🍰 Place on the side

99. A nurse is caring for a client who is 34 weeks gestation. Which of the following statements is the
nurse priority to report to the provider?

🍰 “Acetaminophen is not working on my persistent headaches”

What is the use/effect of protamine

🍰 Decreasing clotting time

101. What does it mean when a client is pacing back?

🍰 Client returns to the room

102. What is the adverse effect of the postoperative opioid medication?

Urinary retention

103. What is the adverse effect of methylphenidate?

🍰 Decreased appetite

104. What must be done when the nurse is not sure of calculating the medication dosage
correctly?

🍰 Call another nurse to verify the calculation

105. What should be done when the nurse sees a mid-abdominal incision?

🍰 Reinforce Dressing

106. What is the adverse effect of Enoxaparin?

🍰 Epistaxis

107. What progress must be seen in a four year old child?

🍰 Eat vegetables

108. What is the adverse effect of furosemide?

🍰 Hypokalemia

109. What safety measures must be implemented for a 12 month old infant?

🍰 Turn pot handles inward

110. What action must be done when the nurse finds out that there is a gastric residual prior to
providing bolus feeding?

🍰 Withhold the bolus feeding and check again in an hour

111. A nurse is caring for a Asian client who is admitted 2 days ago. Which of the following
practices should the nurse must be mindful of?

🍰 Avoid eye contact

112. When a nurse is caring for a client who has a terminal cancer, this should be followed strictly.

🍰 Treatment times

113. Which of the following tasks the nurse can delegate to the Assistive Personnel to perform?

Catheter care

114. A nurse is caring for a client who is terminally ill, sees the patient’s partner crying and
seemed to be upset. What action should the nurse take?

🍰 Encourage the partner to express feelings

115. What should be the action taken when a client is suspected with the manifestations of Bulimia
Nervosa?

🍰 Observe the condition and manifestations

116. The nurse from the morning shift is turning over the reports to the afternoon shift. Which of
the following clients must be seen first?

🍰 A client who is diagnosed of Guillain-Barre syndrome

117. Withhold doxycycline if the client is manifesting this symptom.

🍰 Crackles

118. What intervention must be implemented for a client who has kidney failure with peripheral
edema?

Decrease water intake

119. 4 months infant?

🍰 Cool mist vaporizer

120. The client has attempted to manage 6 month history of severe joint pain in both knees and
hip joints with over-the-counter-medications. The client states these have not completely resolved
the pain. The HCP orders Diclofenac Sodium. When reinforcing teaching about this medication, it
is important that the client understands which concern?

🍰 Do not take with Aspirin

121. The LPN/LVN cares for the client after a plaster cast has been applied to the left leg due to a
fractured femur. The LPN/ LVN should take which action?

🍰 Turn the client every 2 hours

123. The nursing assistive personnel approaches the LPN/LVN and says, “You need to help my
client with morning care because I am not going to be able to get it done.” The LPN/LVN identifies
this is an example of which type of delegation?

🍰 Reverse delegation

124. The client diagnosed with colitis, type 1 diabetes, and cholelithiasis describes experiencing
pain when chewing and relates the food doesn’t have any flavor. Which response is most
appropriate by the LPN/LVN?

🍰 “It is normal as a person ages to have osteoarthritis in the jaw and the loss of taste
buds.”

125. Which statement by the LPN/LVN to the client who is receiving dextroamphetamine
(Dexedrine) for daytime sleepiness and fatigue, is the most important?

🍰 “Call the healthcare provider’s office immediately if your heart seems to be
skipping beats.”

126. The LPN/LVN identifies the nursing assistive personnel is using appropriate body mechanics
if which finding is observed?

🍰 The AP keeps the head erect when lifting a heavy object.

127. The client has a nasogastric (NG) and T-tube is placed following an open (classic)
cholecystectomy. The LPN/LVN understands the purpose of the T-tube is which statement>

🍰 To ensure patency of the common bile duct.

128. The LPN/LVN determines teaching is effective if the client makes which statement about the
proper use of nitroglycerin sublingual tablets?

🍰 “I should take 1 nitroglycerin tablet and repeat the dose in 5 minutes if the chest
pain is unrelieved.”

129. The client with chronic bronchitis says to the LPN/LVN “I do okay until my wife starts
smoking, then my bronchitis gets bad again.” Which statement by the LPN/LVN would be most
appropriate?

🍰 “It is very important for your medical condition that your wife not smoke.”

130. The client is in active labor and says, “Help me! I have to push! I have to push now!” Which is
the best response?

🍰 ” Take a deep breath and bear down like you are having a bowel movement.”

131. The LPN/LVN obtains vital signs from the client: oral temperature of 97.6F (36.4C), radial
pulse of 80 bpm, Respiration rate is 28, and B/P 110/60. Which action should the LPN/LVN take
next?

🍰 Report the vital signs to the registered nurse in charge.

132. The client diagnosed with cancer of the pancreas reports mild to moderate pain within the
past week. Based on this, the LPN/LVN should recommend which to the client?

🍰 Continue with non-opioid analgesics as your were doing.

133. The LPN/LVN cares for the 4 year old child with cast on his left arm due to a fractured wrist.
The child states “The cast feels funny over my wrist bone”. Which is the best response from the
LPN/LVN?

🍰 “I will talk with the health care provider.”

134. The nurse cares for the client who just underwent through an abdominal surgery. To increase
the client’s comfort level, it is most important for the nurse to place the client in which position?

🍰 Supine with knees flexed

135. Which explanation about a thoracentesis by the nurse would be most appropriate?

🍰 “The HCP will be removing fluid from your pleural space.”

136. The client who is having frequent diarrhea is diagnosed with methicillin-resistant
staphylococcus aureus (MRSA) and is placed on contact precautions. The nurse should intervene
if the nurse observes the AP performing which action?

🍰 The AP takes the blood pressure cuff out of the client’s room

137. The nurse finds the client unresponsive on the floor, what should the nurse take first?

🍰 Check respirations and pulse

The nurse knows that a modifiable risk factor related to reducing the possibility of a second
myocardial infarction would be which factor?

🍰 Change of diet to reduce fat intake

The nurse cares for the client who returned from a traditional cholecystectomy 45 minutes
ago. Since that time, the client’s NG tube has been draining 75-100 mL per hour and the fluid
appears to be slightly blood tinged. The nurse must take which action first?

🍰 Report the condition to the RN immediately

The nurse begins to prepare the deceased body for viewing by the family due to arrive in 20
minutes. The nurse is paged to assist a client to eat breakfast and to help another with morning
care and transfer to the chair. Which action by the nurse is most appropriate?

🍰 Delegate the clients requiring help with breakfast and morning care to the nursing
assistive personnel and continue to prepare the body

The nurse observes the client falling out of a chair; which action should the nurse take first?

🍰 Stay with the client and ask the AP to get the RN

The nurse should offer the client which food for a clear liquid diet?

🍰 ½ cup of strawberry gelatin

The nurse observes an AP place a hot water bottle directly on the skin of an elderly patient.
Which of the following action should the nurse take first?

🍰 Immediately remove the hot water bottle

The client insist that the nurse remove her vest restraint “immediately”. What action by the
nurse is best?

🍰 Check the health care provider’s order to determine the criteria for removing the
restraint.

. The nurse knows that the treatment for pleurisy is which implementation?

🍰 An anti inflammatory medication

The nurse works with the client taking phenytoin for seizure prevention. Which instruction is
the most essential for the nurse to reinforce regarding this medication?

🍰 Notify the health care provider immediately if a skin rash develops

The RN mentions that the client diagnosed with borderline personality disorder uses the
defense mechanism of splitting. The nurse recognizes which presentation exemplifies splitting?

🍰 Views other as either all good or all bad

The elderly client is admitted with severe substernal chest pain, tachypnea and tachycardia.
The client’s skin is cool and clammy. The client becomes listless and has decrease in level of
consciousness. The nurse correctly identifies the cause of the client’s symptoms as which
process?

🍰 Cardiogenic shock

The nurse cares for the client diagnosed with a tension pneumothorax. The client asks the
nurse, “which is the most serious complication?”. Which is the best response by the nurse?

🍰 Mediastinal shift

The nurse identifies which position is the best for the client immediately post liver biopsy?

🍰 Right side-lying

Following a tornado, the nurse assists a disaster team by gathering data for victims who are
not physically injured. Which interview question is the best?

🍰 Please share with me what help you need at this time.

. The client diagnosed with asthma currently takes beclomethasone dipropionate. The nurse is
aware that the most serious side effect of this medication are which findings?

🍰 Bronchospasm and angioedema

The nurse cares for an elderly client diagnosed with type 2 DM and partial-thickness burns of
unknown origin on the lower legs and feet. Which reason if stated by the nurse is most likely
cause of the burns?

🍰 Diabetes not being managed appropriately

. The client reports a sad mood, loss of pleasure in usually pleasurable things, and an inability
to sleep through the night. No manic episodes or psychotic behavior is noted in the outpatient
clinic. The nurse correctly understands that which diagnosis matches this client’s presentation?

🍰 Major depression

For three weeks, the client with a spinal cord injury at C7 and has had intensive sessions with
PT TID. the nurse determines the client is making expected progress if which finding is observed?

🍰 The client is able to independently complete ADL

The HCP orders alprazolam 0.5 mg tablet PO TID. the available tablet contains 0.25 mg. How
many tablets should the nurse give?

6 tabs

The client states “I just haven’t felt the best lately, I don’t know what it is, I am not feeling like
my usual self.” Based on the client’s hx of two previous MI and HF, which question should the
nurse ask?

🍰 Have you had any swelling in your feet or lower legs?

The middle aged adult reports fatigue, insomnia, and difficulty focusing on tasks. The client
has three teenagers in the home and is the primary caregiver for the client’s elderly mother with
significant health problems. The nurse knows that one of the difficulties in this situation is what?

🍰 Being a member of sandwich generation

The 3 month old infant was brought to the emergency room with a dx of SIDS. which question
is the most appropriate for the nurse to ask?

🍰 What did the baby look like when you found him/her

The nurse suctions the client’s trache, what is the most important action?

🍰 Apply suction no longer than 10 seconds

The nurse instructs the AP to complete a bath for the client. The AP claims that they do not
have enough time. What action should the nurse take?

🍰 Report to supervisor

Which instruction is essential for the nurse to give the client who has venous
thromboembolism of the left leg?

🍰 Bed rest must be maintained until the anticoagulant therapy is begun

163. Client has COPD and pulmonary edema. Action

🍰 Raise the head of the bed

A nurse Is collecting data from a client who received oxytocin 10 units IM 30 min ago
for excessive vaginal bleeding. Which of the following findings should the nurse expect?

– Client report of uterine cramping

A nurse is assisting with discharge planning for a group of clients. Which of the
following clients should the nurse recommend for a home care referral.

An older adult client who has heart failure and lives alone

A nurse is collecting data from the caregiver of a client who has Alzheimer’s disease.
The caregiver reports that client has difficulty sleeping at night and wonder throughout
the house. Which of the following interventions for the nurse recommend.

Encourage the client to take frequent walks during the day

A charge nurse on a mental health unit is supervising a newly licensed nurse. For
which of the following actions by the newly licensed nurse should the supervising nurse
intervene?

Tells a client he will lose his phone privileges if he does not take his medication

A nurse is reinforcing dietary teaching with a client who is at risk for cardiovascular
disease. Which of the following statements by the client indicates an understanding of
the teaching?

– “I have unsalted pretzels for a snack”

A nurse is reviewing the home meds of a client who recently had TIA to begin taking
clopidogrel. The nurse should instruct the client that which of the following OTC meds
interacts adversely w/ clopidogrel?

Naproxen

A nurse in the providers office is collecting data from a client who has psoriasis.
Which of the following statements by the client should the nurse report to the provider?

I try not to look at the scales on my body

A nurse is receiving info about advance directives w/ a newly admitted client. Which
of the following statements by the client indicates an understanding of the information?

– Advance directives include a living wil

A nurse is reinforcing teaching with the support person of a client who is in the first
stage of labor. Which of the following instructions should the nurse include regarding
effleurage?

Gently stroke her abdomen during contractions

A nurse is collecting data from a client who just received his first dose of
sulfasalazine to treat ulcerative colitis. Which of the following findings should the nurse
identify as an indication of an allergic reaction to this medication.

– Dyspnea

A nurse is caring for a client who recently gave birth to her first child. The newborn is
crying and the client states, “I can’t seem to do anything right. What should I do?. Which
of the following responses should the nurse make?

Let me show you how to swaddle and cuddle him, then you try.

A charge nurse in a long term care facility is developing a performance improvement
plan for an AP. Which of the following actions should the nurse take when developing
the plan? (SATA

– Set a specic time frame for meeting performance goals✅
– Ask the nurse supervisor to review the plan✅
– Include the performance standard that the AP should meet

A charge nurse is observing a newly licensed nurse perform suctioning for a client
who has a tracheostomy. For which of the following actions by the nurse should the
charge nurse intervene?

– Suction for 30 seconds

A nurse is reinforcing teaching with a client who has arthritis. Which of the following
instructions the nurse include in the teaching?

Apply ice to the inflamed joint

A nurse is contributing to the plan of care for an older adult client. Which of the
following physiological changes should the nurse recommend considering when
administering medication?

– Decreased kidney function

A nurse is caring for a client who has terminal cancer. Which of the following actions
should the nurse take to promote the client’s autonomy?

Allow the client to choose treatment times

A nurse is assisting in the plan of care for a client who has end stage amyotrophic
lateral sclerosis and has developed pneumonia. Which of the following actions should
the nurse take?

– verify the status of the client’s advanced directives

A nurse is collecting data from a client in an outpatient clinic and notices extensive
bruising on clients arm. The nurse suspects the client ** partner abuse. Which of the
following is the nurse’s priority action?

Offer support and create a safe, trusting environment

A nurse is verifying informed consent for a client who is postoperative for a vaginal hysterectomy. Which of the following statements should the nurse identify as an indication that the client has given informed consent?

• I am thankful I am done having children ✅

A nurse is caring for a client who has AIDS. Which of the following solutions should the nurse use to disinfect the clients overbed table following a blood spill…?

bleach

• A charge nurse is reinforcing teaching with a newly licensed nurse about infection control measures. Which of the following statements by the newly licensed nurse indicates an .understanding of the teaching?

• Soiled dressings should be placed in biohazard trash receptacle✅

A nurse is caring for a client who suddenly develops chest pain and dyspnea. Which of the following should the nurse take first?

Elevate the head of the client’s bed✅

A nurse is assisting the care of a client who has schizophrenia and auditory hallucinations. Which of the following responses should the nurse make?
a. You should talk to your counselor about the voices

tell me about what the voices are saying to you

A nurse is contributing to the development of an in service training about the differences in mental health beliefs across various cultural groups. Which of the following culture bound syndromes should the nurse include when discussing Navajo Indian cultural health.

ghost sickness

A nurse is collecting data from a client who is in Buck’s Traction. Which of the following indicates the traction is functioning?a. The footplate rests against the foot of the bed.

the wights apply a pulling force continuously

A nurse is collecting data from a client who has macular degeneration. Which of the following findings should the nurse expect?

loss of central vision

A home health nurse is preparing to set up a continuous passive motion machine for a client is postoperative following total knee replacement. Which of the following actions should the nurse plan to take first?

inspect the CPM machine electric cord for damage

A nurse is reinforcing teaching with a parent of a preschooler who has atopic dermatitis. Which of the following instructions should the nurse include in the teachings?

dress the child in soft cotton clothing

A nurse is assisting with the plan of care for a client who is 72hr postoperative following an above the knee amputation. Which of the following interventions should the nurse include in the plan?

assist the client to perform range of motion of the hips

A nurse is contributing to the plan of care for a client who had prolonged exposure to cold weather and has a core body temperature of 90.5 degree F. Which of the following data is the priority for the nurse to monitor?

heart rhythm

A nurse in a provider’s office is reinforcing teaching about skin care with a client who has a new diagnosis of systemic lupus erythematosus. Which of the following statements by the client indicates an understanding of the teaching?

I will dry my skin by patting it with a towel

A nurse is reviewing the medical record for a client who is at 30 weeks of gestation and has anemia. Which of the following laboratory values should the nurse expect?

hub 9.9 g/dL

A nurse in an adult day care facility is contributing to the plan of care for a client whose family reports recent confusion and memory loss. Which of the following strategies should the nurse include in the plan?

use symbols in the communal room signage

A nurse enters a clients room and finds her sitting on the floor next to the shower. The client states that she slipped on some water outside of the shower. Which of the following actions should the nurse take first?

measure the clients vital signs

A nurse is caring for a client who has schizophrenia. Which of the following behaviors of the client should the nurse identify as a negative symptom of schizophrenia?

initiates speech rarely

A nurse is contributing to the plan of care for a client who is experiencing a herpes simplex outbreak. Which of the following interventions should the nurse recommend?

place disposable thermometers in the clients room

A nurse is receiving change of shift report on four clients. Which of the following clients should the nurse plan to see first?

A client who has a femur fracture and reports numbness of the toes

A nurse is reviewing the medical record of a client who is taking acetaminophen to relieve headache pain. Which of the following conditions in the clients history should the nurse ID as a contraindication?

hepatitis c

A nurse manager in a long term care facility is discussing evidence based practice with a group of staff members. Which of the following activities should the nurse manager identify as an example of evidence-based practice?

reviewing a newspaper article about local rates of influenza

A nurse is feeding a client who had a stroke resulting in left-sided weakness. Which of the following actions should the nurse take?

a. ask the client to tilt his head forward while swallowing

A nurse is implementing a bladder training program for a client who had a stroke. Which of the following interventions should the nurse take?

check for residual urine after voiding agree

A nurse is reinforcing discharge teaching with a client who has stable angina pectoris. Which of the following statements by the client indicates an understanding of what to do when chest pain occurs?

I will stop what I doing to lie down

A nurse is receiving a telephone prescription from the provider. Which of the following actions should the nurse take? SATA

-a. Ask the provider to spell out the name of the medication
-c. Record the date and time of the telephone prescription
-d. Request the provider confirm the read back of the prescription

A nurse is reinforcing teaching about passive range of motion exercises with the family of a client who had a stroke. Which of the following instructions should the nurse include in the teaching?

Support the extremity above and below each joint during the exercises

A nurse in a long term care facility is delegating care for a group of clients for the oncoming shift. Which of the following tasks should the nurse delegate to an AP? SATA

a. Record urine output for a client who has a suprapubic catheter
c. Transfer a client who is receiving radiation therapy to radiology
e. Measure vital signs for a client who requires contact precautions

A nurse is discussing the plan of care with a client who is 4 hours postop and has a urestomy with an AP. Which of the following statements should the nurse include in the discussion?

b. We need to check the urine output every hour

A nurse is collecting data from a client who has heart failure. The nurse notes the client has crackles in the bases of the lungs, SOB, and RR 24 breaths per minute. Which of the following actions should the nurse take?.

. Maintain the client in high-fowlers positionAgree

A nurse is collecting data from a client who has UTI and is taking ciproflaxin. The client reports pain in her calf muscle. Which of the following statements should the nurse make?

b. Discontinue the medication. I will ask your provider for another antibiotic.

A nurse in an acute mental health facility is caring for an adolescent client who is exhibiting destructive behavior. Which of the following actions should the nurse take after applying physical restraints to the client?

c. Plan to remove the restraints as soon as the client is calm

A nurse is counting respirations and notes that they are of average depth, rate is 24/min, and rhythm is regular. Which of the following alterations in the client’s breathing pattern should the nurse continue to monitor?

tachypnea

A nurse manager is assisting with negotiations between two charge nurses who are having a conflict over staff not completing assignments. Which of the following actions should the nurse manager take

. Establish ground rules before beginning discussions between the charge nursesAgree

A nurse is caring for a client who is receiving intermittent enteral tube feedings and has diarrhea. Which of the following actions should the nurse take?

Deliver a smaller amount of formula more frequently

• A nurse is contributing to the plan of care for a client who has bipolar disorder and is experiencing acute mania, which of the following interventions should the nurse include in the plan of care?

• Provide the client with high calorie finger foods

• A nurse is preparing to perform a 12 lead ECG for a client who reports palpitations, which of the following actions should the nurse take to insure an accurate capture of the client’s heart rate and rhythm? SATA

-Position the electrodes on the client’s chest and extremities
-Tell the client to expect sensations similar to static electricity during the test
• Explain to the client that movement can alter the test results

• A nurse is completing documentation in the medical record about a client who fell on the floor, which of the following statements should the nurse include in the documentation?

• The client stated “I lost my balance and fell when I got out of bed to the bathroom”

A nurse and a family practice office routinely review client’s immunization status at their annual physical examination visit, for which of the following clients should the nurse recommend human papillaou virus immunization (HPV Virus)?

A teenage boy

• A nurse is caring for a client who is 2 days post-operative following a surgical creation of an ileostomy and is tolerating oral intake, which of the following actions should the nurse take?

• Empty the ileostomy pouch every 2-3 hours

• A nurse is assisting for the plan of care for the client who has burns to his lower extremities, which of the following actions should the nurse include in the plan?

• Clean the most contaminated wound first

• A nurse is reinforcing teaching with a client who is at 36 weeks of gestation and is about to undergo an amniocentesis, which of the following information should the nurse include in the instructions?

• You will need to have an empty bladder for the test

• A nurse is reinforcing teaching about healthy lifestyle changes with a female client who has mild hypertension, which of the following statements by the client indicates an understanding of the teaching?

• I should decrease my salt intake to 2g per day

• A nurse is caring for a client who is one day post-operative following a total hip arthroplasty and is receiving Heparin subcutaneously, which of the following adverse effects of the medication should the nurse report to the provider?

• Epistaxis

• A nurse is collecting data during a muscle skeletal examination of a client who has spinal colon deformity and reports back pain, which of the following images should the nurse identify as an indication of the client has kyphosis?

• Person with higher lump on back

• A nurse is caring for a 3yr old child immediately following a clinic tonic seizure, which of the following actions should the nurse take?

• Check the child for oral injuries

• A nurse is reviewing the health history of a middle adult client who is receiving chemotherapy for leukemia, which of the following immunizations should the nurse recommend that the client receives?

• Inactive Influenza

• A nurse is reinforcing discharge teaching with a client who has angina and a prescription for sublingual nitroglycerin tablet, which of the following statement by the client indicates an understanding of the teaching?

• I should expect pain relief in 1-3 minutes

• A nurse is monitoring a client for hyperkalemia, which of the following findings should the nurse report to the provider?

• Irregular heartbeat

• A nurse is caring for a client who has advanced colorectal cancer and reports that her daughter insists on caring for her at home without assistance. The client states that she prefers help for home health care and hospice, which of the following responses should the nurse make?

• I can be with you while you explain your feelings about your care to your daughter

A nurse is collecting data from a client who is 12 hours postoperative following an intestinal surgery. which of the following findings should the nurse report to the charge nurse prior to client ambulation.

• oxygen saturation 90%

• A nurse is reinforcing teaching about home safety precautions with the parent of a 3-month-old infant which of the following should the nurse include in the teaching

• remove bibs when the infant is going to sleep

A nurse is caring for a client who is to undergo a subtotal thyroidectomy which of the following items should the nurse place at the client’s bedside

• tracheostomy kit

A nurse is reinforcing teaching with a parent of a child who has ADHD and is taking methylphenidate which of the following statements by the parents indicated the medication is effective

• our child is able to complete his homework on time

A nurse is reinforcing teaching with a client who has coronary artery disease and a new prescription for warfarin which of the following should the nurse include in the teaching

• take the medication at the same time everyday

A nurse is reinforcing teaching with a client who is at 12 weeks of gestation and has hyperemesis gravitation, which of the following client statements indicates and understanding of the nurse’s instructions.

• I will wait 1 hour before getting up in the morning before having breakfast

• A nurse is contributing to the plan of care for a chest tube set to continue a suction to relieve a pneumothorax which of the following should the nurse include

• ensure that the device is kept below the level of the clients chest

• A nurse is caring for a client who is terminally ill and is comatose the client has a living will that has a DNR order. The client’s son requests removal of the DNR order which of the following actions should the nurse take.

• place a hold on the order until review of the ethics committee

• A nurse is caring for a client who is recently diagnosed with depression the clients partner asks when he will get better which of the following is the appropriate response by the nurse

• tell me what you know about depression

A nurse is reinforcing dietary teaching with a client who has Cushing’s syndrome which of the following statements indicates an understanding of the client

• I will limit my sodium intake

• A nurse is reinforcing teaching with a group of AP about intimate partner violence the nurse should include that which of the following is a common characteristic of a predictor for intimate partner violence

• possessiveness

40. A nurse is collecting data from a client who uses a continuous positive airway pressure cpap machine at night for sleep apnea. the nurse should identify which of the following indicates proper use

• there is one finger width between he straps and the face

• A nurse is assisting in the care of a client who is at 34 weeks of gestation and is experiencing lower back pain. which of the following recommendations should the nurse make?

• perform pelvic rocking exercises several times per day

• A nurse is reinforcing discharge teaching with the family of an older adult client about safety precautions when administering a hypotonic enema to the client, which of the following instructions should the nurse include in the teaching?

• Instruct the client to inhale while inserting the rectal tube

• A nurse is administering an intermittent enteral feeding through the clients NG tube, during the installation the client reports abdominal cramping and nausea, which of the following actions should the nurse take?

• Slow the rate of the formula instillation

• A male nurse is assigned to care for an older adult female client, the client tells the nurse that she wants a female nurse to care for her, which of the following statements should the nurse make?

• I will ask to have you assigned to a female nurse

• A nurse is reinforcing discharge teaching with a client who has a prescription for a home oxygen therapy, which of the following statements by client indicates an understanding of the teaching?

• I will check my oxygen equipment at least once daily

• A nurse is reinforcing teaching about colostrum with a new mother who is breastfeeding, the mother asks why is colostrum so important for my baby, which of the following responses should the nurse make?

• Colostrum provides many important antibodies the newborn lacks

A nurse is receiving change of shift report on four clients, which of the following clients should the nurse see first

• a client who had a hernia repair 4 hours ago and is reporting nausea

52. A charge nurse in a long-term care facility is discussing ethical theories with a group of newly licensed nurses which of the following statements should the charge nurse identify that the newly licensed nurse understands utilitarianism?

• I will respect the decision of a client who has a chronic illness and stops treatment

• A nurse is positioning a client in preparation for a lumbar puncture which of the following positions should the nurse place the client

• lateral recumbent

• A nurse is reinforcing with a patient about home safety for the parents of a 6-week-old infant which of the following instructions should the nurse include

• keep one hand on your baby’s abdomen if you turn away while he is on the changing table

A nurse is reinforcing teaching with a client who is about to undergo electroconvulsive therapy the nurse should explain to the client that which of the following adverse reactions can occur following the procedure

• temporary memory loss

• A nurse is reviewing the medical records of 5 clients for which of the following events should the nurse write an incident report (select all that apply)

-• a client received the first dose of an antibiotic medication 1 hour before the collection of blood for a culture and sensitivity test
-an approximate amount of urine was recorded after the urine leaked from
-the client’s catheter bag
the client fell while ambulating to the bathroom alone

• A nurse is reinforcing teaching with a client who has hypothyroidism and a new prescription for levothyroxine the nurse should instruct the client to notify the provider for which of the following manifestations of thyroid toxicity?

• nervousness

• A nurse is caring for a client who has anorexia nervosa and who has a behavioral management plan in place. which of the following findings should the nurse identify as an indication that the plan is effective

• potassium 3.5/g dl

A nurse is contributing to the plan of care for a client who has chronic pancreatitis which of the following recommendations for the client’s nutrition should the nurse take

• increase the client’s daily protein intake

A nurse is reinforcing teaching with a newly licensed nurse about client privacy which of the following information should the nurse include in the teaching

• information about a client’s health can be emailed if encrypted

• A nurse is reinforcing teaching with a client about footcare who has type2 diabetes which of the following statements indicates an understanding of the teaching

• I will wear a clean pair of cotton socks each day

• A nurse is providing change of shift report for a client which of the following should the nurse include in the report

• The clients mother died 4 hours ago from breast cancer

A nurse is caring for a client who is receiving a continuous iv infusion. the nurse notes that the skin around the catheter site is edematous and cool which of the following actions should the nurse take first

stop the infusion

• A nurse is reinforcing teaching with a client who is postpartum about keeping her newborn safe while in the facility. which of the following should the nurse include in the teaching?

• Alert the staff if any of your identification bands are missing

A nurse is assisting with preparing a client who is to have a central venous catheter inserted for administration of TPN . which of the following should the nurse take?

• verify the amount of TPN solution the client is receiving every 4 hours

• strokes are more frequently caused by ruptured blood vessels in the brain

• a sudden severe headache is a warning sign of stroke

• A nurse is assisting with a community health course for family members of clients who half Alzheimer’s disease. which of the following should the nurse include?

• limit the number of choices for the client

• A nurse is collecting data from a group of clients who have major depressive disorder. the nurse should identify that which of the following clients is at the greatest risk for suicide

• a client who exhibits an increase in energy

• A nurse is monitoring a client who has psychosis for adverse effects of clozapine. which of the following should the nurse identify as an indicationof agranulocytosis?

WBC 1000 mm

A home health nurse is collecting data from a client following a mild cardiac infarction MI which of the following statements by the client should the nurse address first

• I am worried my body will never be the same as it was before

79. A nurse is reinforcing discharge teaching with a client who is post-operative following cataract removal which of the following instructions should the nurse include in the teaching

• avoid laying on the effected side

80. A nurse is assisting in the care of an adult client who has COPD and is receiving albuterol treating with 20mg of progesterone twice daily. The clients asks why the nurse is checking his blood glucose level, which of the following responses should the nurse make?

• prednisone can cause blood glucose levels to increase

82. A nurse is assisting with the development program of a group of older adults. which of the following action should the nurse take?

• determine the literacy level of the participants

83. A nurse is caring for a client who has moderate anxiety which of the following actions should the nurse take

• maintain eye contact when talking with the client

84. A nurse is reinforcing teaching with a client who is scheduled for a colonoscopy which of the following statements by the client indicated an understanding of the teaching.

• I will call the provider if I have increased flatulence after the examination

85. A nurse is reinforcing food safety with a group of older adult clients. which of the following information should the nurse include?

• refrigerate leftover foods within 3 hours

87. a nurse is reinforcing teaching about a low sodium diet with a client who has hypertension. which of the following indicates an understanding of the instructions?

• I will use frozen vegetables instead of canned

88. A nurse in a providers office is collecting data from a client who was just diagnosed with TB, which of the following actions should the nurse take first?

• contact those who live with the client

90. A charge nurse is reviewing communications skills with a group of newly licensed nurses. which of the following should the nurse include as an example of assertive communication?

• if you don’t come in to cover an extra shift tomorrow we won’t offer you overtime anymore

91. A nurse is collecting data from a client who has placenta previa. Which of the following findings should the nurse expect?

• bright red vaginal bleeding

92. A nurse is preparing to administer a medication to a client. to help verify a client’s identity which of the following questions should the nurse ask the client.

• what is your telephone number?

93. A nurse is reinforcing teaching about how to collect fecal occult blood sample, which of the following statements by the client indicates an understanding of the teaching?

• I should repeat the process three times

94. A nurse is contributing to the plan of care of a client who has HIV, which of the following interventions should the nurse recommend?

• Remove all fruits and raw vegetables from client’s diet

95. A charge nurse is monitoring a group of assistant personnel regarding the use of gloves in contact isolation, which of the following actions by the AP should the charge nurse intervene?

• Take gloves off inside when tasks are complete

96. A nurse is collecting data from an older adult client who has been taking furosemide for the past week, if the client has developed dehydration as an adverse effect of the medication, which of the following finding should the nurse expect?

• Postural hypotension

97. A nurse is planning care for 4 clients, which of the following task should the nurse ask the charge nurse to reassign to the RN?

• Provide discharge teaching about home IV medication therapy

99. A nurse in a mental health facility is caring for a client who expresses anxiety about exercising in the outdoor courtyard. The nurse promises to walk with the client in the courtyard each day. Which of the following ethical principles is the nurse demonstrating?

• Nonmaleficence

102. A nurse on the medical surgical unit is caring for a client immediately following a bronchoscopy. Which of the following actions should the nurse take?

• Inform the client that he might experience a flushed feeling

104. A nurse is reinforcing discharge teaching with a client following a gastrectomy. Which of the following foods should the nurse instruct the client to avoid to prevent dumping syndrome?

• Ice cream

105. A nurse in a community health clinic is discussing informed consent with a group of newly licensed nurses. which of the following statements should the nurse include?

• A pregnant adolescent can provide informed consent for herself

106. A nurse is caring for a client who has a chlamydia infection, a new prescription for doxycycline. The client reports nausea and vomiting after starting the medication. Which of the following recommendations should the nurse make?

• Take the medication with crackers

107. A nurse on a medical surgical unit receives a change of shift report for a group of adult clients. Which of the following clients should the nurse attend to first?

A client who has a blood pressure of 186/120

108. A nurse is collecting data from a client who has a long leg cast on his left leg. Which of the following findings is priority?

• Diminished pulse on the effected extremit

110. A nurse caring for an older adult client who had a stroke 3 weeks ago and has left sided facial paralysis. Which of the following actions should the nurse take to prevent aspiration?

• Place food on the stronger side of the client’s mouth

111. A nurse is reinforcing teaching with a client who has stomatitis. Which of the following instructions should the nurse include in the teaching?

• Consume soft bland foods

113. A nurse is collecting data from an older adult client who has hypertension and is taking amlodipine. The nurse should identify which of the following manifestations as an adverse effect of the medication?

dizziness

114. A nurse is caring for a client who is 12 hours postpartum and has an episiotomy and reports perennial pain. Which of the following actions should the nurse take?

• Encourage the client to use a sitz bath twice daily for 20 minutes

117. A nurse is collection data from a client who is 1-daypost-operative following a cesarean birth. Which of the following findings is the nurses priority? Click on the exhibit button for addition information about the client. There are 3 tabs that contain separate categories of data.

hemoglobin 9, hematocrit 30%

119. A nurse is caring for an older adult client who as acute delirium. Which of the following actions should the nurse take first?

• Determine the clients level of conscientiousness

121. A nurse is providing pre-operative care for a client who reports he has no one at home tohelp him out after his out-patient surgery. Which of the following actions should the nurse take?

• call the provider about admitting the client to a facility over night

122. A community health center is assisting in the development of a brochure about
hypertension. Which of the following action should the nurse take?

• explain medical terminology using basic one syllable words

123. A nurse is collecting data from a client who reposts recently using cocaine. Which of thefollowing manifestation should the nurse expect?

• Hyperthermia

124. A nurse is reinforcing teaching with a newly licensed nurse about caring for a client who ison isolation precautions. Which of the following instructions should the nurse include in theteaching?

• roll used gowns inside out for disposal

125. A nurse is administering hydromorphone to a client who is experiencing postoperative pain.Which of the following findings is an adverse effect of this medication?

• urinary retention

126. A nurse is reinforcing teaching about home management with their partner of a client whohas dementia. Which of the following instructions should the nurse include in the teaching

• label the door to the bathroom with a symbol

127. A nurse is collecting data from a client who has heart failure and edema of the lower
extremities. Which of the following actions should the nurse take to monitor for pitting edema?

• press firmly with the finger tip on the edematous areas

128. A nurse is caring for a school aged child who has scoliosis and reports skin discomfort
under his brace. Which of the following actions should the nurse suggest managing the
discomfort?

• wear a t-shirt under the brace

129. A nurse is caring for a group of clients. Which of the following clients should the nurserecognize is experiencing fluid volume excess?

• a client who has heart failure and has had orthopnea for 2 days

132. A nurse is reinforcing teaching with a client about measures to reduce the manifestationsof irritable bowel syndrome. Which of the following statements should the nurse make?

• “you should chew sugarless gum to reduce bloating”

133. A nurse is a provider’s office is collecting data from a client who has candidiasis Which of thefollowing should the nurse expect?

• thick white vaginal discharge

136. The nurse is collecting data for an adolescent client. The client reports a fever, abdominalpain and vomiting. Which of the following areas should the nurse should identify as McBurneypoint. (image of the stomach with hot spots on the client’s abdomen)

LowerRightQuandrant- choose the lower right quadrant on the image. Make sure you choose the clients
right, not your right

137. A nurse at a provider’s office is caring for a pregnant client who is in her third trimester ofher pregnancy. Which of the following findings should the nurse report to the provider?

• blurry vision

138. A nurse is caring for a 2-year-old child who has clostridium difficile. Which of the followingactions should the nurse take?

• initiate contact precautions

139. A nurse is preparing a capillary blood specimen to measure a client’s blood glucose. Whichof the following action should the nurse take?

• allow the antiseptic to dry before puncturing

141. A nurse is reinforcing teaching with a client who has a new diagnosis of hypertension and anew prescription for captopril. Which of the following interactions should the nurse include?

• rise slowly when moving from a sitting position to a standing position

142. A nurse is preparing change of shift report on a client who is 2 days postoperative followinga total knee arthroscopy (or arthroplasty). Which of the following information about the clientshould the nurse include in the report?

• time of last pain medication

143. A nurse is reinforcing teaching with a client who has a new prescription for spironolactone.Which of the following findings should the nurse instruct the client to avoid?

• dried apricots

18. Iv fluids and pump malfunction
Discontinue and tag machine

assess Pt remove malfunctioning machine

20. Postpartum hemorrhage Pt given methylergonovine to control post party hemorrhage

Monitor for adverse reactions: hypertension, n/v,HA
Do not administer to Pt with high bp.

19. ****Home oxygen therapy with nasal cannula teaching

Candice answer is petroleum jelly to keep nares moist.
Not supposed to use petroleum or oil based
Only water based products

21. tonsillectomy what to watch for

Frequent swallowing

23. Pt acute pancreatitis

reports pain LUQ

24. AP talking about Pt in hallway

Tell AP to stop convo

30. What’s an example of confidentiality breach

Nurse tells Chaplin Pt has cancer

cervical cap

Leave in place for at least 6hrs after intercourse

plaster cast how do relieve itching

Hair dryer

Phytendione injection in newborn

(vit k)
Prevents bleeding problems

46. Or with antisocial personality – smoke break-

Talk to charge nurse about breaks adjusted ??

48. panic attacks (& what’s the best treatment?)

Abdominal breathing ?? Relax techniques

51. Newborn 18hrs old

Yellow sclera (newborn needs phototherapy) ?

56. Diabetes Pt with 24hr creatinine test

57. Graves’ disease developing thyroid storm

Fever, fast and irregular hr, hypertension

58. Seizure disorder supplies at bedside

Suction machine

59. Arterial pulse (picture of leg) dorsalis

pedis
Foot

63.Anorexia nervosa nutrition

Creatinine 2mg LOW Malnutrition ****

66. Upset Pt with terminal diagnosis

I will stay

Oral meds

Verify meds 3x before administration

70. Sexual dysfunction meds

gentamicin

78. Interprofessional conference

Spinal cord injury

16 weeks of gestation ferrous sulfate teaching

Increases fiber intake to prevent constipation

Parathyroidectomy:

image on test

Hospice:

music therapy for pain/ non pharmacological measures

a nurse is caring for an older adult client who wears hearing aids with detachable ear molds which the following actions should the nurse take when caring for the hearing aids.

clean the hearing aid with a soft cloth

a nurse is collecting data from a client who has heart failure and its taking furosemide which of the following findings should indicate to the nurse that he dedication is effective.

increase urinary output

a nurse ids reinforcing teaching with a new parent about bathing her newborn which of the following statements should the nurse include

perform sponge baths until the baby umbilical cord falls off.

a nurse at a long-term care facility is preparing to irrigate a client ear which of the following actions should the nurse take>?

position the client on the unaffected side following irrigation.

– ( direct the solution toward the wall of the ear canal.

a nurse is collecting data from a client about risk factors for cardiovascular disease the nurse should identify that which of the following findings is a modifiable risk factor.

cholesterol 240 mg/dl

a nurse is caring for client who has a chest tube that was inserted 4 days Aho which of the following findings should the nurse report on the charge nurse

400 ml of drainage in the collection chanber with 4 hrs.

a nurse is reinforcing teaching with a client who will collect stool specimens at home for fecal occult blood testing which of the following foods should the nurse instruct the client to avoid for 3 days before the test.

red meat

a nurse is collecting data from a client ho delivered vaginally 4 hr. Aho the nurse notes the uterus is soft and is located two fingers breadths above the umbilicus to the left of the midline which of the following actions should the nurse take first

massage the clients fundus

a nurse is reinforcing teaching with a client who is about to start using an albuterol dose inhaler which of the following instructions should the nurse include in the teaching?

close your mouth around the mouthpiece

a nurse is talking with a partner of the client who recently died which of the following statements should the nruse make.

tell me what I can do for you at this time.

a nurse is caring for a client who has 0.9 sodium chloride IV solution infusion through a peripheral iv catheter. which of the following findings should the nurse expect of the client has infiltration.

redness around the insertion site.

a nurse is caring for a client who has peptic ulcers disease and is scheduled to undergo an esophgogastroduodenoscopy which of the following actions should the nurse take prior to the procedure.?

ensure that the client gave informed consent

A nurse has delegated care to an assistive personnel. At the end of the shift, the Ap asks the nurse to enter data for her because the AP has forgotten her password and needs to leave. Which of the following actions should the nurse take?

Tell the AP to contact the IT department for charting assistance.

A nurse is reinforcing teaching about car seat safety to the parents of a newborn. The nurse should instruct the parents to place the car seat in a….

Rear-facing position in the back seat.

A nurse is preparing a client for surgery. The client tells the nurse that he is concerned about the safety of a large sum of money in his wallet. Which of the following actions is appropriate for the nurse to take?

contact security personnel to place the money in the facility safe.

A nurse is caring for a toddler who is admitted to the pediatric unit and is 2 hr postoperative following a tonsillectomy. Which of the following findings is a sign on hemorrhage?

Frequent Swallowing

A nurse is reinforcing discharge teaching with an older adult client’s family about safety precautions when administering a tap water enema to the client. Which of the following should the nurse include?

Instruct the client to bear down during rectal tube insertion.

A nurse is caring for a client. Which of the following client statements should the nurse attend first?

“My heartburn pain is going into my jaw now”

A nurse is caring for a client who is 3 days postoperative following abdominal surgery and has a Penrose drain. Which of the following actions should the nurse take?

Leave the drain insertion site open to air

A nurse is reinforcing with a client who is about to undergo an upper gastrointestinal series with fluoroscopy. Which of the following information should the nurse provide?

“You will have to drink a contrast medium before the test.”

A nurse is reviewing the medical record of a client who has sustained a full-thickness burn and is in the emergent phase of burn. Which of the following findings should the nurse expect?

Hypernatremia

A nurse is planing to administer an ophthalmic medication to a client. Which of the following actions would minimize systemic absorption of the medication?

Wait 5 mins after instillation before instilling the drops in the other eye

A nurse is preparing morning medication for a client who has potassium level of 3.2 mEq/L. For which of the following medications should the nurse withhold administration and notify the provider?

Furosemide

A nurse is collecting data from a client who has preeclampsia and is receiving magnesium sulfate via a continuous IV infusion. Which of the following findings should the nurse report to the provider?

Facial flushing

A nurse in a provider’a office is collecting data from a client who was discharged from the hospital 7 days ago following treatment from deep-vein thrombosis. Which of the following findings is the nurse’s priority?

The client takes ibuprofen to treat musculosketal pain.

A nurse in an urgent care clinic is collecting data from a client who reports having diarrhea for the past 3 days. Which of the following findings indicates hypokalemia?

Muscle weakness

A nurse is participating in a performance improvement process to decrease the rate of health care-associated infections within the facility. The nurse should recognize that which of the following is an appropriate performance improvement outcome indicator?

Client data indicates a decreased rate of infection

A nurse is establishing a baseline postoperative assessment for a client who is recovering from a right femoropopliteal bypass graft. Which of the following findings in the assessment of the clients’s right leg should be of the most concern to the nurse?

The client’s foot feels cooler than the previous assessment

A nurse is preparing a client regimen. Which of the following insulins can be mixed?

Aspart
Regular
Detemir
Lispro
Glargine cannot be mixed with any insulin

A nurse is caring for a school-age child whose family adheres to a vegan diet in the home. The nurse should recognize the child is at risk for deficiency of which of the following?

Vitamin D

A nurse is reinforcing teaching with a postpartum client. Which of the following should the nurse explain as the most important reason for staff to wear identification?

Reducing the risk of newborn abduction

A nurse is reinforcing teaching with a group of expectant parents regarding the proper use of a car seat. Which of the following statements by the parent indicates an understanding of the teaching?

I can place a rolled towel on each side of my newborn’s head until he can hold his head up.

A nurse on a mental health unit is discussing client rights with a group of coworkers. Which of the following statements should the nurse include?

A nurse must withdraw consent for treatment in writing if he is competent to do so.

A nurse from a medical-surgical unit is floating to a postpartum unit. Which of the following clients is an appropriate assignment for the nurse to accept?

A client who had a cesarean delivery 24hr ago

A nurse is caring for a client who is in mechanical restraints after becoming violent with a staff member. Which of the following actions should the nurse take?

Document in the client’s medical record every 15 mins

A nurse is reinforcing teaching with the family of a client who is receiving palliative care for terminal cancer. Which of the following guidelines should the nurse emphasize about appropriate pain control measures?

Administer schedule doses of medication from a patient-controlled analgesia (PCA) pump.

A nurse is assisting with a group therapy session in which a client begins to monopolize the group’s time. Which of the following interventions is appropriate?

schedule one-on-one discussion time with the client

A nurse is preparing to administer required immunization to a toddler during a well-child visit. The parent ask the nurse how many baby aspirin he should administer if the toddler develops a fever. Which of the following responses should the nurse make?

“You should avoid administering aspirin to your child”

A nurse is assisting with the admission of an older adult client who has impaired mobility and is at risk for falls. Which of the following actions should the nurse plan to perform first?

Document the client’s risk in the medical record.

A nurse is assisting with the admission of a client who has Hodgkin’s disease and is receiving chemotherapy. Because a private room is not available, the nurse should recommend that the client share a room with a client who has which of the following diagnoses?

Sickle-cell anemia

A nurse is caring for a child who has terminal cancer. Which of the following responses by the child’s siblings should the nurse expect?

The school-age sister views death as being a type of temporary sleep

A nurse is reinforcing teaching with a client about the importance of a low-sodium diet. Which of the following foods should the nurse recommend as the best choice to include in a low-sodium diet?

medium baked potato

A nurse is caring for a client who is receiving total parenteral nutrition. Which of the following laboratory results indicates a possible complication of this therapy?

Serum calcium 12.5 mg/dL

A nurse is caring for a client who is asking about the technique of effleurage and its use in labor and delivery. Which of the following responses should the nurse make regarding this technique?

“It is a light stroking of the skin during a uterine contraction.”

A nurse in a clinic is reinforcing teaching with a client who has a new prescription for a combination contraceptive transdermal patch. Which of the following should the nurse include in the teaching?

“Apply the patch to the lower abdomen.”

A nurse is collecting data from an 18-month-old toddler at a well-child visit. Which of the following findings should the nurse report to the provider?

The toddler can say four words.

A nurse in an acute mental health facility observes a client having a panic attack. Which of the following interventions should the nurse implement first?

Instruct the client to use abdominal breathing.

A nurse is collecting data from a client who has a long-leg cast on his left leg and reports severe pain. Which of the following findings should the nurse identify as an indication that the client might have compartment syndrome? (Select all that apply.)

Pallor in the exposed portion of the left foot

Inability to move the left foot

Paresthesia in the left foot

A nurse in a provider’s office is talking on the phone with the parent of a school-age child who has varicella. The parent asks the nurse when the child can return to school. The nurse should tell the parent that which of the following is an indication that the longer contagious?

All lesions are healed.

A nurse is assisting in the care of a client who is experiencing a postpartum hemorrhage. Which of the following medications should the nurse plan to administer?

Methylergonovine

A nurse is caring for a client who is postoperative following a hemicolectomy. Which of the following is the best indication that the client needs PRN pain medication?

The client reports pain.

A nurse is reinforcing healthy eating habits for an older adult client to ensure adequate dietary intake. Which of the following is an appropriate recommendation?

Snack between meals.

A nurse is caring for a client who has a new prescription for a protective safety restraint. Which of the following actions should the nurse take?

Secure the restraint with an easy-to-release tie.

A nurse is caring for a client who has a new prescription for a protective safety restraint. Which of the following actions should the nurse take?

Secure the restraint with an easy-to-release tie.

A nurse is reinforcing discharge teaching with a client who is postoperative following laser surgery for open-angle glaucoma. Which of the following statements by the client indicates an understanding of the instructions?

“I will ask to work the night shift, so I will not be driving in bright sunlight.

A nurse is caring for a client who has bipolar disorder and is experiencing mania. Which of the following actions should the nurse take?

Frequently remind the client of the expectations for her behavior.

A client who is prescribed metoprolol for hypertension tells the nurse, “I don’t want to take this medication because it makes me tired all the time.” Which of the following is the appropriate response?

“Let’s talk with your doctor about other options.”

A nurse is reinforcing teaching with a client who has rheumatoid arthritis and is using a straight-legged cane for ambulation. Which of the following instructions should the nurse include?

“Move the stronger leg forward first.”

A nurse is assisting with the plan of care for a client who has Alzheimer’s disease. Which of the following actions should the nurse recommend for the plan of care?

Orient the client to time and place twice per day.

A nurse is speaking to a client who called the provider’s office to report chest pain and requests clarification on how to take his sublingual nitroglycerin tablets. Which of the following instructions should the nurse give the client?

Wait 5 min between doses.

A nurse is caring for a client who is undergoing evaluation for a thoracic aortic aneurysm. Which of the following findings should make the nurse suspect an impending rupture of the aneurysm?

Report of sharp pain in the back

A nurse is reinforcing discharge teaching with an older adult client who has osteoarthritis. Which of the following statements by the client indicates an understanding of the teaching?

“I will apply cold compresses when my joints are painful.

A nurse is caring for a client who has COPD. The client tells the nurse, “I get short of breath during meals. It is too much trouble to eat.” Which of the following instructions should the nurse suggest?

“Have several small meals during the day.”

A nurse is caring for a client who has urolithiasis. Which of the following actions should the nurse take?

Strain the client’s urine.

A nurse is preparing to administer purified protein derivative (PPD) to a client. The nurse should use which of the following routes?

Intradermal

A nurse monitoring a client who is receiving a blood transfusion. Which of the following findings indicates the client is experiencing an allergic reaction?

Urticaria

A nurse is monitoring a client who is receiving a blood transfusion. Which of the following findings indicates the client is experiencing an allergic reaction?

Urticaria

A nurse is reinforcing teaching with a client who has a new prescription for lithium carbonate. Which of the following client statements indicates an understanding of the teaching?

“I will notify my provider if I develop a tremor.”

A nurse is collecting data from a child who has acute glomerulonephritis. Which of the following findings should the nurse expect?

Periorbital edema

A nurse is caring for a client who is postoperative following a femoral cardiac catheterization. Which of the following actions should the nurse take during the first 30 min?

Compare the bilateral peripheral pulses.

A nurse is caring for a client who is newly diagnosed with type 1 diabetes mellitus. The nurse should recognize that the client needs a referral for diabetic education when the client does which of the following?

Lists sweating, shaking, and palpitations as symptoms of hyperglycemia

A nurse is contributing to the plan of care for a client who has bipolar disorder and is experiencing mania. Which of the following actions should the nurse include in the plan of care?

Decrease the level of environmental stimuli

A nurse is contributing to the discharge plans for four clients. The nurse should identify that which of the following clients requires an interdisciplinary care conference?

A client who has hemiparesis and lives alone

A nurse is applying a condom catheter to a male client who is incontinent. Which of the following is an appropriate technique to use?

Leave space between the tip of the penis and the end of the condom catheter.

A nurse is assisting in the preparation of an in-service for a group of unit nurses about ethical principles. Which of the following nursing actions should the nurse include as an example of fidelity?

A nurse keeps a promise made to a client.

A nurse is reinforcing teaching with the parents of a newborn who had a circumcision. Which of the following client statements indicates understanding of the teaching?

“I will apply petroleum jelly to the penis with each diaper change.”

A nurse receives report on four clients. The nurse should first collect data about the client who has which of the following?

A decreased level of consciousness and vomiting

A nurse is planning to reinforce teaching with a preschooler who is about to undergo an incision and drainage for cellulitis on the left arm. Which of the following techniques should the nurse use?

Help the child put a dressing on a doll.

A nurse is contributing to the discharge plan of an older adult client who had a total hip arthroplasty. The client is unable to ambulate independently and lives alone. Which of the following settings should the nurse recommend for this client?

Home health nurse visits

A nurse working on a mental health unit is meeting with a client who has been on the unit for 2 days. The nurse greets the client and asks, “What has been happening with you today?” Which of the following therapeutic techniques is the nurse using?

Giving broad openings

A nurse is caring for a group of clients, which of the following can be assigned to an assistive personnel?

collecting a stool speciment ot test for occult blood.

A nurse is working on a unit for clients with dementia. Which of the following client situations requires the nurse to write an incident report?

A client is found lying on the floor next to a chair.

A nurse is discharging a client who was admitted for newly diagnosed type 2 diabetes mellitus. The client is independent and lives alone. Which of the following should be included in the discharge plan?

refer the client to a diabetic support group.

A nurse is caring for a client who has type 2 diabetes mellitus and a blood glucose level of 60 mg/dL. For which of the following findings should the nurse monitor?

Clammy Skin

A female client who is an abusive marriage has discusses with the nurse strategies to prevent this abuse. which of the following client statements indicate an understanding of an appropriate strategy?

“I need to recognize the signs that my husband is becoming abusive.”

“I need to identify what triggers my husband’s anger to prevent his abuse.”

A charge nurse in a long-term care facility is preparing to administer noon insulin to a client. The nurse observes that the assistive personnel (AP) has not documented the client’s blood glucose level. Which of the following actions should the charge nurse take first?

Determine if the AP has completed the assignment.

A client is scheduled for an outpatient colonoscopy. which of the following actions is a nursing responsibility in the informed consent process?

Verify that there is a signed and witnesses consent form in the client’s chart.

A nurse smells alcohol on the breath of an assistive peronnel (AP) during report. Which of the following actions should the nurse take?

Report the situation to the nurse manager.

A nurse in a provider’s office is collecting data from a parent of an infant who is being screened for cyctic fibrosis. Which of the following supports a diagnosis of cystic fibrosis?

Frothy stools.

When caring for an assigned group of clients, the nurse should wear gloves when…

performing oral hygiene.

A nurse is caring for a client who is receiving heparin. Which of the following is the appropriate route of administration?

Subcutaneously.

A nurse is caring for a client and recognizes the client’s rights to confidentiality have been breached in which of the following situations?

A hospital risk manager includes information from a client’s medical record in

A nurse is caring for a client who had a femoral-popliteal bypass graft 2 days ago. When monitoring peripheral pulses, the nurse is unable to locate a pulse on the affected leg. Which of the following actions should the nurse take?

Notify the charge nurse of the finding.

A nurse is caring for a full-term newborn who was circumcised 6 hrs. ago. Which of the following findings indicates that the newborn is experiencing pain?

Awakens every 3 hours
Furrowed Brow
Oxygen Saturation of 95%
Positive babinski reflex.

A nurse is reinforcing teaching with a client about organ donation. Which of the following client statements indicated a need for further teaching?

“My doctor should decide if my organs will be donated.”

A nurse is preparing to administer an IM injection to a client. To reduce the risk of neddlestick injury, the nurse should

dispose of the used needle immediately in a puncture-proof sharps container.

A client who is 24 hour postoperative suddenly develops chest pain, dyspnea, anxiety, diaphoresis, and cough. Which of the following actions should the nurse take first?

Elevate the head of the client’s bed.

A nurse is caring for a 17-year old client who is admitted for an emergency appendectomy. Which of the following is an appropriate action by the nurse in obtaining informed consent?

Obtain verbal consent from the client while waiting for the parents to arrive.

Witness the signature of the client’s parent when he arrives.

Have the client’s older sibling give consent if a parent is not available.

Delay the procedure if the provider cannot contact the parents.

A client who is 24 hour postoperative suddenly develops chest pain, dyspnea, anxiety, diaphoresis, and cough. Which of the following actions should the nurse take first?

Elevate the head of the client’s bed.

A nurse is reinforcing discharge teaching to a client following a gastrectomy. To prevent dumping syndrome, which of the following foods should the nurse instruct the client to avoid?

Ice Cream

A nurse is caring for a client admitted with shortness of breath who will be undergoing arterial blood gas testing. Which of the following client statements indicated an understanding of the purpose of drawing arterial blood gases?

“This will indicate how much acid is building up in my blood.”

2 hrs post-op following an ileal conduit procedure bladder cancer

Dusky-colored stoma

Administering an INJ to a client even after the client states, “I don’t want that injection. The last time I got that I was sore for a week.”

Battery

Admitting a client to the medical-surgical unit. Which of the following actions should the nurse first take?

Observe the client’s level of mobility

Adolescent client who has no previous history of a seizure disorder and is admitted after having a 3-min tonic-clonic seizure 2 wks after sustaining a mild concussion. EEG

Explain that the client may not have coffee prior to the EEG

Adolescent who has cystic fibrosis and has a prescription for pancrelipase

Take w/ meals

After evaluating the morning lab results on several clients, the provider writes prescriptions for 4 clients assigned to the nurses care?

Infuse 10 meQ/l potassium chloride IV over 1 hr to a client who has potassium of 3.2

After making morning round, the charge nurse on a surgical unit delegates the follow tasks to the AP.

Set up a room for an expected postoperative admission

AP reports to the nurse that another ap has spent the entire morning on the phone and has not completed the morning assignment

Redirect the AP to discuss the issue with the other AP

Assessing client who systemic lupus erythematous SLE which of the following is an expected findings

Dry, raised facial rash

At the start of an evening shift on a cardiac unit, a LPN brings the nurse a list of client reports

Indigestion

Benefit from a three-way indwelling catheter

Prone to development of blood clots un the urine

Caring for a client who has a new prescription for chlorpromazine by IM injection

Check orthostatic BP 1 hr after admin

Caring for a client who was involved in a motor-vehicle crash. The client reports shortness of breath and chest pain and asks the nurse “Am I dying?”

Admin pain meds

Charge nurse creating assignments for next shift for several nurses and one of the nurses is pregnant – which pt the nurse should NOT be assigned to?

60 yr old who is recovering from shingles

Charge nurse on pediatric unit is making assignments for a float nurse from the medical unit. Pt to see first?

10 yr old who has pneumonia & is receiving respiratory txs

Child who has pediculosis capitis

store non-washable items in plastic bags for 14 days

Child who is unresponsive has ICP

Pad the side of the bed

A client ask the nurse if it is safe for him to take glucosamine supplement?

Shellfish allergy

Client following an open colectomy

Uncontrolled pain

A client has just returned to the nursing unit following cardiac catheterization. In the immediate postoperative post procedure period which of the following is the priority nursing actions?

Immobilizing the affect extremity

A client hopitialized for a bone marrow transplant is in protective isolation while undergoing total body radiation and intense chemotherapy. The clients sibling comes to visit but has obvious manifestations of an upper respiratory infection.

Allow the sibling to visit after donning a sterile gown, mask, gloves but prohibits physical contact

Client recovering from closed head injury assumes a decerbrate posture in response to stimuli

D lying flat ?? (Extentend extremities) The client rigidly extends his arm

Client reports chest pain. Which of the following lab findings indicates myocardial damage

Troponin 1.8

Client who has a left femur fracture and is skeletal traction. The clients reports pain due to muscle spasms in affected leg

Realign the extremity in traction

Client who has given birth to a stillborn

Acknowledging the reality of the newborn’s death

Client who is 2 days postoperative following a lumbar laminectomy and is reporting nausea

Admin an antiemetic (Aprepitant for nausea)

Client who is 8hr postoperative following a right-modified radical mastectomy – which findings to report to provider? (SATA)

Coughing
Frothy, pink secretions

Client who is receiving a controlled epidural analgesia infusion

Covering the insertion site with a transparent dressing

Client who reports the use of chondroitin and glucosamine

Improve joint functioning

Community health nurse is completing a newborn home visit and observes family members smoking cigarettes in the house

Review the effects of secondhand smoke w/ family members

A community health nurse is evaluating eligibility for home assistance for a client who is quadriplegic

Determine the client living situation

A community health nurse is planning to make home visits to several clients who all live within a few miles radius of the office – Who to see first?

An infant who has failure-to-thrive and was discharged from a facility yesterday to a foster home on daily weights

Creating and activity plan for a home bound older adult client. In the planning, the nurse considers the physiologic changes that may affect pulmonary function r/t the normal aging process

Decreased blood oxygenation

Delegates the following tasks to the AP bathe four clients, distribute fresh water, obtain morning vital signs

Set a clear time frame for the completion of each task

DI w/ severe head injury

Urine output 250

Diagnosis of antisocial personality disorder

Poor impulse control

Diagnosis of complete placenta previa is admitted to the labor and delivery suite at 36wks gestation w/ contractions 5 min in frequency and 1 min in duration

Prepare the client for c-section

Dietary intervention for a client who is immobile due to pelvic and femur fractures

Provide a high-protein diet

Difficulty falling asleep at night

Eat a bedtime snack (containing carbohydrates??)

Disaster triage following a natural disaster

Amputation

Discharge teaching about disease prevention to a client who has active Tb

Educating the client how to cover nose and mouth with tissues when coughing

Discussing a living will with a client

It communicates my wishes for end of life care

Discussing care w/ newly licensed nurse for a client who practices orthodox Judaism – which food not given?

Roast beef and ice cream

Education about ECT major depressive disorder

General anesthetic admin prior ECT treatments(This treatment are for pts who aren’t responding to their pharmaceutical meds)

Education to the mother of a toddler who has pertussis

Pertussis booster

An emergency department nurse triages a group of school children injured in a school bus crash

A child who reports diplopia and nausea and was unconscious at the scene but is now awake

Experiencing acute mania …

Provide high-calorie nutritional supplements (provide pt with portable nutritious food because pt may not be able to sit down)

Gravida 2, para 1. The client is at 41 wks gestation and is receiving oxytocin for the augmentation of labor

Contractions occur every 90 sec

Haloperidol for the tx of schizophrenia

Screen the client for Tardive Dyskinesia (inform pt to lower doses to decrease the chances of TD)

health nurse is performing a vision screening on a 4-month-old infant. When shining a light source into the infant’s visual field – rx?

Closes eyes

Hearing impairment – incorporate which communication methods?

Rephrase sentences the client doesn’t understand

Home health nurse is admitting a client who is prescribed peritoneal dialysis

Clarify the clients actual and perceived health needs

A home health nurse is conducting an initial home visit for a client who has terminal breast cancer. The client who has 2 minor children and a limited support system

Ask the client about her understanding of the diagnosis

A home-health nurse is providing teaching about self-administration of insulin to client who is newly diagnosed w/ diabetes mellitus

I can use the same area as long as I rotate injection sites

Home health nurse is reviewing treatment goals w/ a client who has diabetes mellitus evaluate lab tests to determine effective long-term management of blood glucose levels

HbA1C

Hyperthyroidism – which of the following rooms is appropriate for the nurse to assign this client?

Cool room (reduced room temp)

If something happens to me from which I cannot recover, I don’t want to go on a ventilator:

You’re concerned that something may go wrong?

Ileal conduit due to treatment for bladder cancer

Cleanse w/ moisturizing soap

Immediate postoperative period following thorac surgery, a nurse medicates a client for pain on a schedule

Reduces the RR

Infection control nurse is reviewing care procedures for four clients

A client who has varicella zoster is placed in a negative pressure room

In-home assessment for an older adult client who is at risk for falls. Which of the following assessment findings should the nurse identify as a potential hazard?

A comp. chair wheels

In patient psychiatric unit is setting short-term goals for an adolescent who was admitted for tx of anorexia nervosa

The client will develop personalized meal plan

Involuntarily admitted to psychiatric unit following a failed suicide attempt – experimental tx

Witness consent before medication administration

Lab data on client who has dehydration

Elevated BUN

Lab values on a client who has taken an overdose of acetaminophen – organ damage

ALT

Mastitis of left breast – does this mean that i must stop nursing my baby?

No, you can continue to nurse from both your breasts

medical unit received change-of-shift report …

68 yr old

Morning lab values to several clients …

Client who is prescribed digoxin and furosemide

Pt w/ Neutropenia (MS pg. 543)

Remove fresh flowers from a client’s room

Newborn nursery is performing assessments on four neonates that are all less than 24 hr old – which pt is priority?

Pinna below the outer canthus of the eye (The eye should be even with the upper tip of the pinna of the newborn’s ear; MN p. 158)

Pt is Nulliparous and in the first stage of labor. The last internal assessment revealed 100% cervical effacement w/ 5cm of dilation. At the end of last contraction, the nurse observes a large gush of fluid coming out of clients perineal area. What should nurse do first?

Check FHR!!

A nurse at a public health clinic is caring for a group of clients. Which of the following should the nurse identify as a reportable dx to the CDC?

Hepatitis A

A nurse at the family planning clinic triages several clients over the phone – (PRIORITY)?

Sharp pain in RIGHT shoulder

A nurse delegates tasks to a LPN and an AP. When admitting a client who is experiencing acute liver failure and who has ascites and an NG tube – which of the following tasks is most appropriate to delegate to the LPN?

Insert an indwelling catheter if the client has not voided in 3 hr

A nurse educator is discussing modes of transmission w/ nursing students. Which of the following should the nurse educator include as an example of vector-borne transmission?

West Nile virus from mosquito bite

A nurse in a mental health clinic is observing clients in the day room. The nurse sits down to talk w/ an adolescent client who was admitted w/ clinical depression. After a few minutes of convo, the adolescent ask the nurse “why did you choose to talk to me out of this room full of kids?”

You’re curious why I am interested in you and not the others?

A nurse in a mental health clinic is reviewing the hx and physical for a 17-yr old client who has a new dx of social phobia:

Client reports minimal alteration in performance at a part-time job

Nurse in an emergency department is assessing a client who has a nasal fracture.

Clear fluid drainage from the nares

A nurse in a pedi unit is caring for a group of clients. Which of the following diseases should the nurse implement droplet precautions?

Pertussis

A nurse in a provider’s office is caring for a client who has Parkinson’s Disease and has been taking levodopa for the past 5 wks. The client reports a new onset of muscle twitching – best response? 

This is a manifestation of drug toxicity & may require a dosage reduction

A nurse in the emergency department is caring for a client following a motor vehicle crash – PRIORITY?

Stabilize the cervical spine

A nurse in the emergency department is caring for a client who has abdominal pain. Which of the following actions by the nurse demonstrates VERACITY?

A nurse explains the potential risks of tx.

(Veracity = truth/not withhold all info; Fidelity = keeping promises)

A nurse in the emergency department is providing discharge teaching to a client who has a sprained ankle. For the first 24hr following sx, the nurse should instruct the client to do which of the following?

Intermittently place ice on the ankle (30 mins – 1 hr??)

A nurse in the PACU is caring for four post-op clients. The nurse realizes that coughing poses a risk to which of the following clients?

Thyroidectomy

(Coughing can cause incision to reopen & bleeding can occur)

A nurse is admitting a client who has chronic gout – What med should the nurse admin?

Allopurinol

A nurse is admitting an infant who has bacterial meningitis. Nursing intervention?

Minimize environmental stimuli

A nurse is admitting an unidentified female who was brought to the emergency department:

3 months

A nurse is analyzing cardiac rhythms of four telemetry clients in the coronary care unit:

C looks normal but lots of artifact
or
Fibs in between QRS’s

A nurse is assessing a client 1 wk after a successful bone marrow transplant. The client reports peeling of skin on her hands and feet. The nurse should recognize this desquamation as an indication of which of the following complications?

Graft vs host disease

A nurse is assessing a client who has an IV infusing per gravity at 125ml/hr

Blood is backing up in the IV tubing

A nurse is assessing a client who is 2 hrs post-op following insertion of a chest tube connected to chest drainage system as depicted in the diagram above:

Fluctuation of the fluid level chamber b (middle)

A nurse is assessing a client who is post-op following abdominal sx. States “I feel like my incision ripped open.”

Place client in Low-Fowler’s position

A nurse is assessing a client w/ schizophrenia – what is a manifestation?

Auditory hallucinations

A nurse is caring for a client following insertion of a subclavian non-tunneled percutaneous central venous catheter – What should nurse do?

Review the chest x-ray

A nurse is caring for a client who has a partial laryngectomy and is receiving continuous enteral feedings at 65ml.hr through a gastrostomy tube (intervene??)

The client is lying in a supine position

A nurse is caring for a client who has newly implanted sealed internal radiation device to treat cervical cancer.

Keep a 3 ft distance from the radiation implant

A nurse is caring for a client who has a new prescription for lithium carbonate. PRIOR TO ADMINISTERING THE FIRST DOSE, WHICH OF THE following lab values should the nurse evaluate?

Sodium

A nurse is caring for a client who has a typical depression and is taking phenelzine – What can you eat?

Low-fat yogurt

A nurse is caring for a client who has cirrhosis of the liver:

Monitor for abdominal ascites

Nurse is caring for a client who has end stage liver disease. The daughter of the clients ask about her father’s DNR request.

Tell me your feelings bout your father’s prognosis.

A nurse is caring for a client who has heart failure and has started taking a loop diuretic:

Decreased reflexes

A nurse is caring for a client who has just returned to the unit following a bronchoscopy:

Offer oral fluids to the client

A nurse is caring for a client who is manic. Which of the following activities is appropriate for the nurse to suggest?

Taking walks (physical activity)

A nurse is caring for a client who is postoperative and at risk for development of DVT – Risk factor?

Prolonged bed rest

A nurse is caring for a client who is receiving enteral nutrition. What lab level should be reported?

Serum sodium of 128

A nurse is caring for a client who is receiving gentamicin. Which of the following findings indicates the client is developing toxicity?

Tinnitus 

P-pee (furosemide) 

A-abx & aspirin

C-cancer (tamoxifen)

T-tinnitus

A nurse is caring for a client who is receiving IV antibiotics and tests positive for C. diff – PRIORITY?

Place client on contact precautions

A nurse is caring for a client who will be receiving a transfusion of platelets – Effectiveness?

Decrease in bleeding from puncture sites

A nurse is caring for a client with fractured ribs, has developed thrombophlebitis, and is being treated with a heparin drip. The client develops hematuria and has an aPTT of 100 seconds – What nurse do next??

Turn off heparin drip

A nurse is caring for a group of clients. Which of the following tasks is appropriate for the nurse to delegate to the AP? (SATA)

-Transfer a client from bed to chair w/ mechanical lift

-Provide postmortem care on a client who experienced cardiac arrest

A nurse is caring for a mother who was prescribed methadone during pregnancy (MN p. 183):

-Tachypnea
-Irritability
-Tremors
or
-Increased seizures, sleep pattern disturbance, higher birth wt

A nurse is caring for an infant who is being treated for dehydration – indication for further education?

-Flat anterior fontanel

(Reason: Fontanels should be soft & flat; No tears = dehydration)

A nurse is caring for a post-op client. Which of the following interventions will reduce the risk of DVT?

Apply venous plexus foot pumps

A nurse is caring for a preschool-age child who has a short-leg, plaster cast applied 1 hr ago.

Support the affected leg on a pillow

A nurse is discussing the z-track method to admin hydroxyzine hydrochloride w/ a newly licensed nurse – Understanding of teaching?

This technique decreases the risk of subcutaneous infiltration

A nurse is electronically documenting assessment findings for a client – How should document/assess pain?

The client reports a pain level of 6 on a scale of 0-10

A nurse is giving change-of-shift report about a client who is 36-hr post-op to another nurse:

Pain relieved by changing position

A nurse is interviewing a client who presents with multiple injuries that are consistent with intimate partner abuse. After establishing trust and rapport, which of the following should the nurse say?

“Let’s talk about what happened to you”

A nurse is planning care for a client who has a new diagnosis of HIV.

I cant seem to gain any weight

A nurse is planning care for a client who is comatose and has a stage II decubitus ulcer on the coccyx – What should nurse do?

Provide the client with an alternating pressure mattress

A nurse is planning care for a client who is prescribed a cane for ambulation. Which of the following nursing actions should the nurse include in the plan of care?

Remind the client to place the cane on the unaffected side.

Rationale: Keep the cane on the stronger side of the body
1- Move cane forward 15cm
2- Then, move weaker cane forward
3- Advance stronger cane

A nurse is planning care for a newborn who has hyperbilirubinemia and is to receive phototherapy:

Place the newborn 45cm or 18 in from the light source

A nurse is planning teaching for a client who is at 10 weeks of gestation and has a history of urinary tract infections UTI:

Empty bladder before & after sexy time

A nurse is planning to change the dressings on a school-age child who has sustained multiple burns:

Apply the dressings in a proximal to distal pattern

A nurse is planning to provide community education about viral hepatitis – What info should include?

Clients who have a hx of viral hepatitis are unable to donate blood

A nurse is preparing the auscultate a client’s apical pulse at the point of maximal impulse (PMI):

5th intercostal space at the left midclavicular line

A nurse is preparing to administer potassium chloride intravenously to a client who has hypokalemia. The client is receiving a current infusion of 0.9% sodium chloride at 125ml/hr. Which of the following actions should the nurse plan to take?

Dilute the solution prior to the infusion.

Rationale: Dilute K+ and give no more than 40mEq/L of IV solution to prevent vein irritation

A nurse is providing teaching for a client who has a new prescription for lithium carbonate – (Need for further education???)

Take medication w/ milk.

Rationale: Taking lithium with food would help with decreasing GI distress

A nurse is providing teaching to the parent of a preschooler who was newly diagnosed with a latex allergy – What foods to avoid?

Bananas

A nurse is reviewing the employee health program for new employees. Which of the following diagnostic assessments should the nurse recommend for all the new employees to screen for the presence of tuberculosis?

Mantoux test

A nurse is reviewing medical records for four clients – Which of the following represents appropriate documentation?

Carafate 1g PO 1hr AC

A nurse is reviewing the hx and physical of a client who has right ventricular heart failure: 

Hepatomegaly (liver enlargement)

A nurse is teaching a client who has a new dx of GERD – Include in teaching?

Sleep w/ HOB elevated

A nurse is teaching a client who has a new prescription for digoxin. Which of the following should the nurse include in the teaching?

Notify your provider if you experience muscle weakness!

A nurse is teaching a client who has left-leg weakness how to use a standard walker (further education?)

Moves the left leg and walker ahead together

A nurse is teaching a client who has a low literacy level about home management of diabetes mellitus:

Show the client an educational video

A nurse is teaching a family about home hospice care:

Hospice care improves the quality of life through palliative care

A nurse is teaching a prenatal class about evidence of effective breastfeeding to a group of parents – What include in teaching? (SATA)

-Swallowing sounds are audible
-Stools are yellow and seedy after 7 days
-Maternal breasts become soft following feedings

A nurse is writing a plan of care for a client who is home bound and has stage 2 Alzheimer’s Disease. Which of the following should the nurse include in the plan of care?

Assist with money management

A nurse manager is making staffing assignments for the med surg unit. Which of the following clients is appropriate to assign to a float nurse from the postpartum unit?

A client who is 2 days posto-op following a colon resection

A nurse manager is presenting an in-service for newly licensed nurses about advance directives:

Advanced directives must benotarized to be legally implemented

A nurse manager is presenting info to the nursing staff regarding the appropriate use of client restraints – What should be included? (SATA)

-Nurse should pad the bony prominences.

-Providers prescription should include the type of restraint to use.

-Remove or replace restraints frequently to ensure good circulation to the area

A nurse notices smoke coming from aclients room and discovers a fire in the wastebasket. After moving the client to safety what nurse do next?

Co said: Notify the facility operator

I found: Activate fire alarm

A nurse plans to ambulate a client on the third day after cardiac sx.

Premedicate the client w/ the prescribed analgesic

A nurse supervisor receives notification of a disaster in the community and the possibility for multiple admissions to the facility – Which ppl can be discharged? (SATA)

– 1 day post op
-laparoscopic
-cholecystectomy
-temp 98

A client who has an abdominal wound and is receiving negative pressure wound therapy

Report to the nurse manager

Rationale: negative pressure wound therapy is used to reduce edema

A nurse volunteers to assist after witnessing a mass casualty incident involving a train derailment. Which of the following clients should the nurse immediately and completely immobilize?

Pt w/ Glasgow coma????

A nurse who typically works on the postpartum unit is assigned to float to the maternal newborn unit. The nurse is very anxious about floating and uncomfortable with the assignment the charge nurse has selected – What should nurse do?

Ask the charge nurse to assign an experienced nurse to act as a resource

A nurse working in an inpatient mental health facility observes a client who is agitated and threatening staff members in the day room – What should do?

Accompany the client awayfrom the common area

Nutritional assessment on a client who has a calcium deficiency

Osteogenesis

OCD – Further education??

Teach the client to meditate about obsessive thoughts

An older adult client tells the nurse, “I thought immunizations were for kids.” The nurse informs the client that older adults should receive which of the following immunizations?

Herpes zoster vaccine

Orthopedic floor is completing the morning assessments on several clients – PRIORITY client?

24-year old male who has a casted femur fracture

Performing postmorten care on an older adult client

Place absorbent pads under the buttocks area

Rationale: Apply fresh linens w/ aborbent pads on bed and a gown

Perform wound irrigation for a client who has an open secondary wound when creating a sterile field

Hold bottle w/ the palm over …. while pouring

Postpartum unit is caring for several clients (which pt do you see first?)

1 day postpartum has not voiding in 8hrs

Preparing a client for sx and nursejust administered the preoperative injection (what is the next step or first step?) *safety question

Raise the side rails on the bed

Preparing to administer eye drops to a client:

Use aseptic technique and drop the medication into the conjunctival sac

Preparing to obtain informed consent for a colonoscopy from a client who does not speak english:

Provide a consent form in the client’s spoken language

Prescribed furosemide – indication of fluid overload???

Rales in lower lobes of lungs

Primagravida client in the emergency department at 12 weeks of gestation hyperemesis gravidarum:

1,000 mL/hr

Providing dietary teaching for a client who has a hx of nephrolithiasis – Which of the following is appropriate to include in the teaching?

Avoid foods that have high levels of oxalates


Rationale: avoid foods high in oxalate sources such as spinach, black tea, cocoa, beets, pecans, peanuts, okra etc.

Providing discharge teaching to a client who is postpartum and plans to breastfeed. Which of the following should the nurse recommend the client increase in her diet during lactation?

Calcium

Providing postoperative teaching to a client who has a newly inserted pacemaker (if for further education this maybe the answer because this is not in the education)

I will use my cellphone on the earopposite of my pacemaker

Recognizes that the teen pregnancy rate in the community has increased:

Arrange a meeting with teenage mothers who are high school students in the community


Rationale: Provide pregnancy counseling, including alternative course of actions, instructions and encourgament in parenting role in home and peer group settings.

Recovering from an acute myocardial infarction – left sided heart failure:

Bilateral lung crackles

Recovery room is assessing a client who has a new chest tube. The nurse finds that the water seal is no longer tidaling – What could be problem?

The tubing may be kinked

Rationale: Tidaling is expected.
Cessation of tidaling in water seal chamber signals lungs re-expansion or obstruction.
Continuous bubbling indicates an air leak.

Referral for a speech language pathologist:

Client who has difficulty swallowing

Reviewing data of four young children who are receiving treatment. Which of the following should alert the charge nurse to the possibility of child abuse?

A 3-month old infant who has a skull fracture from reportedly rolling off the changing table

“Reviewing” medications for a group of clients labor and delivery (which would you NOT give?)

Phenytoin for seizure disorder

Rising number of sexually transmitted infections in the community????

?????

School nurse scabies

Red itchy papules on child’s groin area

Several client measurements were obtained with morning vital signs – pt priority*??

A 4-year old who has closed head injury and heart rate of 60

Tetralogy of Fallot:

Pulse oximeter

Urinalysis done for a preschool age child:

Pyelonephritis ????

PRIORITY action for post-op ileus?

Monitor bowel sounds

Idenfy risks for fat embolism syndrome (FES):

Older adults (70 or older) and hip/pelvis fractures are common causes

(turn upside down)?

Disorder of eye post-op risk?

Bleeding and infection

A nurse is reinforcing teaching with a postpartum client. Which of the following should the nurse explain as the most important reason for staff to wear identification?

Reducing the risk of newborn abduction

A nurse is reinforcing teaching with a client who plans to bottle feed her newborn. Which of the following statements indicates an understanding of the instructions?

I will feed my baby six to eight times a day

A nurse is reinforcing teaching with a client diagnosed with osteoarthritis who reports joint pain,swelling, and stiffness. Which of the following client statements indicates understanding of the teaching?

I will exercise my joints as much as i can when they are inflamed

A nurse is reinforcing teaching with a client who is scheduled for a mammogram. Which of the following instructions should the nurse include in the teaching?

Refrain from using deodorant on the morning of the test

A nurse is preparing a clients insulin regimen. Which of the following insulin’s can be mixed?

Insulin aspart, regular insulin, insulin lispro

A nurse is reinforcing discharge teaching with a client who is postoperative following laser surgery for open-angle glaucoma. Which of the following statements by the client indicates an understanding of the instructions?

I will take a stool softener to prevent constipation

A nurse is caring for a client who had a stroke and is having difficulty swallowing. The nurse should recommend a referral to which of the following members of the interprofessional team?

Speech therapist

A nurse is reinforcing teaching with a client who is 12 hr postpartum and has an episiotomy. Which of the following instructions should the nurse include?

Change the perineal pad with each void

A nurse is collecting data from a child who has acute Glomerulonephritis. Which of the following findings should the nurse expect?

Periorbital edema

A nurse is preparing to administer a liquid medication to a 6-month-old infant who is crying. Which of the following actions should the nurse take to reduce the risk of aspiration?

Administer using a needless syringe in the buccal cavity

A nurse is reinforcing teaching with a client who has peptic ulcer disease. Which of the following statements by the client indicates an understanding of the teaching?

I will plan to have my meals at regular intervals

A nurse in long-term care facility has received change-of-shift report about four clients. Which of the following clients should the nurse attend to first?

A client who has COPD and dementia and was agitated during the night shift

A nurse is collecting data from a client who has substance use disorder and reports recently taking opioids. Which of the following findings should the nurse identify as a manifestation of opioid intoxication?

Pinpoint pupils

A nurse in a urgent care clinic is collecting data from a client who reports having diarrhea for the past 3 days. Which of the following findings indicates hypokalemia?

Muscle weakness

A nurse is contributing to the discharge plans for four clients. The nurse should identify that which of the following clients requires an interdisciplinary care conference?

A client who has hemiparesis and lives alone

A nurse is reinforcing teaching with an adolescent who has a new prescription for cefazolin. For which of the following findings should the nurse instruct the adolescent to monitor and report to the provider?

Depression

A nurse is reinforcing teaching with a group of expectant parents regarding the proper use of a car seat. Which of the following statements by a parent indicates an understanding of the teaching?

I can place a rolled towel on each side of my newborns head until he can hold his head up

A nurse is caring for a school-age child who has epistaxis. Which of them following actions should the nurse take?

Apply pressure at the bridge of the child’s nose

A nurse is caring for four clients. Which of the following clients statements should the nurse attend to first?

My heartburn pain is going into my jaw now

A nurse in a providers office is talking on the phone with the parent of a school-age child who has varicella.

All vesicles have crusted over

A nurse is collecting data from a client who hgb is 8.8 mg/dL. Which of the following statements should the nurse expect?

I feel tired all the time

A nurse is administering a cleansing enema to a client who reports mild cramping during instillation of the solution. Which of the following actions should the nurse take?

Slow the rate of instillation

A nurse receives report on four clients. The nurse should first collect data about the client who has which of the following?

A decreased level of consciousness and vomiting

A nurse is setting up a sterile field prior to performing a dressing change. Which of the following actions should the nurse take?

Open the outermost flap of the wrapper toward the body

A nurse is caring for a client who has bipolar disorder and is experiencing mania. Which of the following actions should the nurse take?

Frequently remind the client of the expectations for her behavior

A nurse is caring for a client who has viral pneumonia. Which of the following actions should the nurse take?

Administer Azithromycin

A nurse who orienting to a medical-surgical unit is having difficulty finishing client care tasks during his shift. Which of the following suggestions should the nurse’s preceptor make to help the nurse with time management?

Try to complete a task before moving on to the next.

A nurse is caring for an older adult client who is postoperative following a total hip arthroplasty. The nurse is preparing to change the client’s dressing. Which of the following actions should the nurse take to demonstrate sensitivity to age-related changes?

Use paper tape for securing the new dressing.

A nurse working in an inpatient mental health facility is assisting with the plan of care for a client who has anorexia nervosa. Which of the following should the nurse recommend to include in the plan of care?

Maintain continuity of staff members.

A nurse is assisting with an admission interview for a client who has schizophrenia. He tells the nurse that he is receiving special audible messages from the Central Intelligence Agency that no one else is able to hear. The nurse should identify that the client is having which of the following alterations in perception?

Hallucination

A nurse is reinforcing teaching with a postpartum client about bathing her newborn. Which of the following statements should the nurse include?

“Wash your newborn’s head under a stream of running water.”

A nurse is caring for a client who has heart failure and reports difficulty limiting sodium in his diet. Which of the following recommendations should the nurse provide?

Replace bottled salad dressing with homemade vinegar and oil dressing.

A nurse is reinforcing teaching about nonpharmacological pain relief with a client who is 6 hr postpartum following a vaginal delivery. The client has an episiotomy and swelling of the labia. Which of the following instructions should the nurse include?

Sit on soft pillows throughout the day.

A nurse is caring for a client who has a new colostomy. He voices a reluctance about resuming sexual relations. Which of the following is an appropriate response by the nurse?

“I’m available if you’d like to talk about your concerns.”

A nurse is reinforcing discharge teaching about transmission precautions with a client who has hepatitis C. Which of the following information should the nurse include?

Avoid sharing razors with other family members.

A nurse is reinforcing teaching about foot care with a client who has diabetes mellitus. Which of the following client statements indicates to the nurse a need for further teaching?

“I will be sure to wear cotton socks.”

A nurse is contributing to the plan of care for a client who has bulimia nervosa. Which of the following interventions should the nurse recommend?

Observe the client for 1 hr after meals.

A nurse is assisting with the discharge planning for a client following a myocardial infarction. Which of the following is an appropriate referral for this client?

Respiratory therapist for breathing exercises

A nurse is contributing to the discharge plan of an older adult client who had a total hip arthroplasty. The client is unable to ambulate independently and lives alone. Which of the following care settings should the nurse recommend for this client?

Skilled nursing facility

A nurse is reinforcing teaching with the parents of a preschool-age child who has a new diagnosis of celiac disease. Which of the following foods should the nurse recommend?

Corn tortillas with black beans

A nurse is assisting with the admission of a client who has mononucleosis. Which of the following isolation precautions should the nurse initiate?

Droplet

A nurse is speaking to a client who called the provider’s office to report chest pain and requests clarification on how to take his sublingual nitroglycerin tablets. Which of the following instructions should the nurse give the client?

Wait 5 min between doses.

A nurse is caring for a client and begins to suspect he is experiencing transference. Which of the following client statements should the nurse identify as an indicator of transference?

“I feel really close to you because you remind me of my ex-boyfriend.”

A nurse is caring for a client who has been placed in restraints. Which of the following is appropriate?

Monitor the client’s skin integrity on a regular schedule.

A nurse is caring for a client who is newly diagnosed with type 1 diabetes mellitus. The nurse should recognize that the client needs a referral for diabetic education when the client does which of the following?

States that he will treat hypoglycemic reactions with 15 g of carbohydrates

A nurse is preparing a client for surgery. The client tells the nurse that he is concerned about the safety of a large sum of money in his wallet. Which of the following actions is appropriate for the nurse to take?

Contact security personnel to place the money in the facility safe.

A nurse on a medical-surgical unit is planning care for a group of clients. Which of the following tasks is appropriate to delegate to an assistive personnel?

Administer a large-volume enema to a client.

A nurse is reviewing the medical record of a client who has sustained a full-thickness burn and is in the emergent phase of the burn. Which of the following findings should the nurse expect?

Hypercalcemia

A nurse is collecting data from an 18-month-old toddler at a well-child visit. Which of the following findings should the nurse report to the provider?

The toddler can say four words.

A nurse delegates a task to an assistive personnel (AP) and the AP refuses the assigned task. Which of the following actions should the nurse take first?

Perform the task.

A nurse on a medical unit is observing an assistive personnel (AP) delivering food trays. Which of the following actions by the AP requires intervention?

Providing a ham and cheese sandwich to a client who follows a kosher diet

A nurse is reinforcing teaching with a client who has a vitamin C deficiency. Which of the following foods should the nurse include as having the highest vitamin C content?

1 cup raw broccoli

A nurse is assisting with triaging clients in a mass casualty situation. The nurse should recommend that which of the following clients receive care first?

A client who has diminished breath sounds and paradoxical chest movement

A nurse is working in an urgent care clinic when a client who does not speak the same language arrives with a laceration that will require suturing. Which of the following actions appropriate for the nurse to take?

Ask an assistive personnel who speaks the same language as the client to assist in obtaining informed consent.

A nurse is collecting data from a client who has dehydration and is receiving continuous IV therapy. Which of the following findings indicates effectiveness of the IV therapy?

BUN 16 mg/dL

A nurse is caring for a child who has terminal cancer. Which of the following responses by the child’s siblings should the nurse expect?

The school-age sister is concerned about the impact of her sibling’s death on herself

A nurse is caring for a client who attempted suicide. Which of the following actions should the nurse take?

Serve meals with plastic utensils.

A nurse is removing a female client’s indwelling urinary catheter. Which of the following actions should the nurse take?

Withdraw the fluid from the catheter’s balloon.

A nurse is collecting data from a parent of a preschooler at a well-child visit. Which of the following findings is a contraindication to a measles, mumps, and rubella (MMR) immunization?

Long-term use of immunosuppressants

A home health nurse is reinforcing teaching with a client about kitchen safety. Which of the following instructions should the nurse reinforce about using a fire extinguisher?

Sweep the extinguisher from side to side to put out the fire.

A nurse is caring for a client who is asking about the technique of effleurage and its use in labor and delivery. Which of the following responses should the nurse make regarding this technique?

“It is a light stroking of the skin during a uterine contraction.”

A nurse is preparing to administer required immunizations to a toddler during a well-child visit. The parent asks the nurse how many baby aspirins he should administer if the toddler develops a fever. Which of the following responses should the nurse make?

“You should follow the label directions based on your child’s weight.”

A nurse is caring for a client who is in mechanical restraints after becoming violent with a staff member. Which of the following actions should the nurse take?

Request that the provider write an as-needed prescription for restraints.

A nurse is reinforcing teaching with an older adult client who has a new prescription for digoxin for heart failure. Which of the following effects should the nurse include as an expected outcome of this medication?

Increased heart rate

A nurse is assisting with the plan of care for a client who has Alzheimer’s disease. Which of the following actions should the nurse recommend for the plan of care?

Give directions using simple phrases.

A nurse is caring for a client in a provider’s office. The client asks about the purpose of a living will. Which of the following is the appropriate response by the nurse?

“It expresses your wishes regarding health care when you can no longer communicate.”

A nurse is planning to administer an ophthalmic medication to a client. Which of the following actions will minimize systemic absorption of the medication?

Wait 5 min after instillation before instilling the drops in the other eye.

A nurse is caring for a client who has a methicillin-resistant Staphylococcus aureus (MRSA) infection. Which of the following actions should the nurse plan to take?

Don a gown and gloves when providing perineal care.

A nurse is reviewing the medical record of a client who is requesting an oral contraceptive. Which of the following findings should the nurse identify as a contraindication to the use of oral contraceptives?

History of renal calculi

A nurse is reinforcing teaching with a client who has a new prescription for lithium carbonate. Which of the following client statements indicates an understanding of the teaching?

“I will notify my provider if I develop a tremor.”

A nurse is caring for a client who is undergoing evaluation for a thoracic aortic aneurysm. Which of the following findings should make the nurse suspect an impending rupture of the aneurysm?

Cough with hemoptysis

A nurse collecting data from a client who is receiving a continuous IV infusion of 0.9% sodium chloride. The nurse discovers the IV infusion pump was set incorrectly and the client received 200 mL more than prescribed. Data collection reveals that the client is stable. The nurse recognizes that this incident is not considered malpractice for which of the following reasons?

The client was not harmed as a result of the incident.

A nurse is escorting a client to his car following discharge from an acute mental health facility. The client states intentions to harm a friend when he leaves the facility. Which of the following actions should the nurse take?

Prevent the client from leaving the facility.

A nurse is caring for a client who is 5 days postoperative following abdominal surgery. The client tells the nurse that he was coughing and “felt a pop” at the incision site. The nurse examines the incision and finds wound dehiscence and evisceration. Which of the following actions should the nurse take?

Place the client in a side-lying position.

A nurse is planning to reinforce teaching with a preschooler who is about to undergo an incision and drainage for cellulitis on the left arm. Which of the following techniques should the nurse use?

Take the child on a tour of the surgery and recovery areas.

A nurse receives a verbal order for a client to receive a stat dose of meperidine 100 mg PO. She administers the medication, charts the administration, and then realizes she has administered phenytoin 100 mg PO. Which of the following actions should the nurse take first?

Check the client’s vital signs.

A nurse is caring for a postoperative client and obtains a pulse oximeter reading of 89%. Which of the following interventions should the nurse take first?

Repeat the test on another finger.

A nurse is preparing to administer 5 units of regular insulin and 20 units of NPH insulin to a client who has type 1 diabetes mellitus. Which of the following actions should the nurse take first?

withdraw 5 units of regular insulin from the vial.

A nurse is caring for a client who has urolithiasis. Which of the following actions should the nurse take?

Strain the client’s urine.

A nurse is observing the mother of a 12-hr-old newborn. Which of the following actions by the mother requires intervention by the nurse?

The mother places the newborn in prone position.

A nurse in a mental health facility is reviewing the laboratory reports for four clients. Which of the following results should the nurse report to the provider immediately?

Sodium level of 130 mEg/L for a client who has bipolar disorder and takes lithium

After receiving change-of-shift report, which of the following clients should the nurse collect data from first?

A client who has heart failure and reports severe dyspnea

A nurse is assisting with the admission of a client who has Hodgkin’s disease and is receiving chemotherapy. Because a private room is not available, the nurse should recommend that the client share a room with a client who has which of the following diagnoses?

Community-acquired pneumonia

A nurse is reinforcing teaching with a client who is about to undergo an upper gastrointestinal series with fluoroscopy. Which of the following information should the nurse provide?

“You will receive an injection of contrast dye during the test.”

A nurse is evaluating a client who just attended a breastfeeding class. Which of the following statements by the client demonstrates that she has understood the teaching?

“I will feed my baby each time he cries.”

A nurse is preparing morning medications for a client who has a potassium level of 3.2 mEq/L. For which of the following medications should the nurse withhold administration and notify the provider?

Furosemide

A nurse in a pediatric clinic is reviewing the urine laboratory results for an adolescent. For which of the following results should the nurse notify the provider?

Ketones positive

A nurse is admitting a client who has active tuberculosis. Which of the following nursing interventions is appropriate?

Place the client in a room that is ventilated to the outside.

A nurse is caring for a client who was recently admitted to an inpatient mental health unit. The client tells the nurse that he is not coming out of his room anymore because other clients on the unit make fun of him. Which of the following responses by the nurse is appropriate?

“You feel upset by the responses of others.”

A nurse is caring for a client who is receiving total parenteral nutrition. Which of the following laboratory results indicates a possible complication of this therapy?

Serum calcium 12.5 mg/dL

A nurse in the newborn nursery is planning to obtain blood from a newborn by a heel stick. Which of the following actions should the nurse plan to take?

Apply a heat pack 5 to 10 min prior to the procedure.

A nurse from a medical-surgical unit is floating to a postpartum unit. Which of the appropriate assignment for the nurse to accept?

A client who had a cesarean delivery 24 hr ago

A client who is participating in an anger management session explains that his recent behaviors are related to his job loss. Which of the following defense mechanisms is the client using?

Rationalization

A nurse working at a crisis hotline call center receives a call from a client who states, “I cannot take it. My life is over.” Which of the following is the priority response by the nurse?

Are you thinking of harming yourself?”

A nurse is reinforcing teaching with the parents of a newborn who had a circumcision. Which of the following client statements indicates understanding of the teaching?

“I will apply petroleum jelly to the penis with each diaper change.”

A client’s daughter calls the nurse requesting information about her mother’s condition. The client’s chart does not specify that information can be released to the daughter. Which of the following is an appropriate response by the nurse?

“You will need to contact your mother directly about her condition.”

A nurse is caring for a 17-year-old client who admitted for an emergency appendectomy. Which of the following is an appropriate action by the nurse in obtaining informed consent?

Delay the procedure if the provider cannot contact the parents.

A nurse is reinforcing teaching about home care with the parents of a child who has a seizure disorder. Which of the following instructions should the nurse include?

Call EMS if a seizure lasts 5 min or more.

A nurse in a mental health unit asks a client who has schizophrenia, “How are you?” Which of the following responses should the nurse identify as the speech alteration of echolalia?

“How are you?”

A nurse is reinforcing teaching with the family of a client who is receiving palliative care for terminal cancer. Which of the following guidelines should the nurse emphasize about appropriate pain control measures?

Administer pain medication on a regular schedule.

A nurse in an acute mental health facility observes a client having a panic attack. Which of the following interventions should the nurse implement first?

Instruct the client to use abdominal breathing.

A nurse is collecting data from a 3-month-old infant who is 6 hr postoperative following a cleft palate repair. Which of the following pain rating tools should the nurse use?

FLACC scale

A nurse is caring for a client who has a new prescription for furosemide. The client asks the nurse about the purpose of the medication. The nurse tells the client, “This medication is a diuretic, and you should monitor your weight while taking it.” Which of the following ethical concepts is the nurse exhibiting?

Veracity

A nurse is expressing concern about his assignment with the charge nurse. The nurse states that he has more work to do than anyone else. Which of the following statements by the charge nurse demonstrates the conflict resolution strategy of competing?

“We will take a look at the assignments together and attempt to modify the workload you are concerned about.”

A nurse working on a mental health unit is meeting with a client who has been on the unit for 2 days. The nurse greets the client and asks, “What has been happening with you today?” Which of the following therapeutic techniques is the nurse using?

Focusing

A nurse discovers a fire in a trash can in a client’s room. Which of the following actions should the nurse take first?

Walk the client to the waiting room.

A nurse is caring for a client who is postoperative following a femoral cardiac catheterization. Which of the following actions should the nurse take during the first 30 min?

Compare the bilateral peripheral pulses.

A client who has inoperable cancer tells the nurse that she does not want to pursue the recommended treatment. She asks if the provider can force her to have the treatment. Which of the following is an appropriate response by the nurse?

“You have the right to refuse the recommended treatment plan.”

A nurse is reinforcing teaching with a client who is about to undergo a thoracentesis. Which of the following statements by the client indicates an understanding of the information?

“I will have a chest x-ray following the procedure.”

A nurse is reinforcing teaching with a client who has a permanent pacemaker in place. Which of the following statements by the client indicates an understanding of the teaching?

“I will remove my microwave oven from my home.”

A nurse and an assistive personnel (AP) are caring for a client who requests a PRN pain medication. After the nurse administers the medication, which of the following tasks should the nurse assign to the AP?

Document the client’s respiratory rate in 1 hr.

A nurse is reinforcing teaching with a client who has a new diagnosis of myasthenia gravis (MG) and a prescription for neostigmine. Which of the following information should the nurse include about the action of the medication?

Improves muscle strength

A nurse is assisting with the admission of an older adult client who has impaired mobility and is at risk for falls. Which of the following actions should the nurse plan to perform first?

Check the client’s ability to use the call light.

A nurse is assisting with the admission of an older adult client. Which of the following subjective findings suggests that the client may have cataracts?

Cloudy vision

A nurse is caring for a client who was diagnosed with hyperemesis gravidarum. Which of the following findings should the nurse anticipate?

Poor skin turgor

A nurse is reinforcing discharge teaching with an older adult client who has osteoarthritis. Which of the following statements by the client indicates an understanding of the teaching?

“I will limit purine intake in my diet.”

A nurse is discussing incident reports with a group of newly licensed nurses. The nurse should include that which of the following situations requires completion of an incident report?

A nurse collects data from a client who reports severe pain and finds a dose of hydromorphone in the client’s bed.

A nurse is reinforcing teaching with a client who has rheumatoid arthritis and is using a straight-legged cane for ambulation. Which of the following instructions should the nurse include?

“Keep the cane on the stronger side of the body.”

A nurse is participating in a performance improvement process to decrease the rate of health care- associated infections within the facility. The nurse should recognize that which of the following is an appropriate performance improvement outcome indicator?

Client data indicates a decreased rate of infection.

A nurse is contributing to the plan of care for a school-age child who has sickle-cell disease and is experiencing a vaso-occlusive crisis. Which of the following should the nurse recommend to include in the plan of care?

Perform passive range-of-motion exercises.

A nurse is collecting data from a client who has hepatitis A. Which of the following findings should the nurse expect?

Abdominal pain

A nurse is reinforcing teaching about self-administration of nasal drops with a client. Which of the following positions should the nurse recommend for instillation of the drops?

Supine

A nurse is collecting data from a client who has preeclampsia and is receiving magnesium sulfate via a continuous IV infusion. Which of the following findings should the nurse report to the provider?

Urine output 20 mL/hr

A nurse is orienting a newly licensed nurse to a medical-surgical unit. Which of the following actions should the nurse take to verify that the newly licensed nurse understands the procedure for measuring a client’s blood glucose level?

Have the nurse perform a return demonstration.

A nurse is receiving change-of-shift report on a group of clients. Which of the following clients should the nurse see first?

A client who is exhibiting flight of ideas

A nurse in the newborn nursery is collecting data about a newborn’s Moro reflex. Which of the following actions should the nurse take to elicit this reflex?

Strike the crib surface on which the newborn is lying.

A nurse is caring for a client who has a new prescription for a protective safety restraint. Which of the following actions should the nurse take?

Secure the restraint with an easy-to-release tie.

A nurse is caring for a client who has a chronic illness. In which phase of the therapeutic relationship should the nurse help the client develop problem-solving skills?

Working phase

A nurse is monitoring a client who is receiving a blood transfusion. Which of the following findings indicates the client is experiencing an allergic reaction?

Urticaria

A nurse is assisting in the care of a client who is experiencing a postpartum hemorrhage. Which of the following medications should the nurse plan to administer?

Methylergonovine

A nurse is caring for an older adult client who has pneumonia. When reinforcing teaching about the diagnosis and age-related changes with the client’s family, which of the following alterations should the nurse include?

Increased elasticity of the thoracic cage

A nurse is collecting data from the guardian of a toddler during a well-child visit. The guardian expresses concern to the nurse because his child has a poor appetite, but drinks a quart of milk each day. The nurse should identify that this practice places the toddler at risk for which of the following conditions?

Lactose intolerance

A nurse is applying a condom catheter to a male client who is incontinent. Which of the following is an appropriate technique to use?

Leave space between the tip of the penis and the end of the condom catheter.

A nurse is assisting in the preparation of an in-service for a group of unit nurses about ethical principles. Which of the following nursing actions should the nurse include as an example of fidelity?

A nurse keeps a promise made to a client.

A nurse is caring for a school-age child who is scheduled for a bone marrow aspiration biopsy. The child asks the nurse, “Will this be painful?” Which of the following is an appropriate response by the nurse?

“There will be some pain, but I can give you medicine to make you more comfortable.”

A nurse is assisting with the admission of a client who attempted suicide. When the nurse processes the client’s belongings, which of the following items should the nurse remove from the client’s suitcase?

Perfume

A nurse is assisting with a group therapy session in which a client begins to monopolize the group’s time. Which of the following interventions is appropriate?

Schedule one-on-one discussion time with the client.

A nurse is caring for a client who is 3 days postoperative following abdominal surgery and has a Penrose drain. Which of the following actions should the nurse take?

Reposition the sterile safety pin when the surgeon advances the drain.

A nurse is caring for a toddler who is admitted to the pediatric unit and is 2 hr postoperative following a tonsillectomy. Which of the following findings is a sign of hemorrhage?

Frequent swallowing

A nurse in a long-term care facility is observing an assistive personnel (AP) who is performing hand hygiene. Which of the following actions by the AP requires intervention?

Holding his hands up and elbows down while washing his hands

A nurse is establishing a baseline postoperative assessment for a client who is recovering from a right femoropopliteal bypass graft. Which of the following findings in the assessment of the client’s right leg should be of the most concern to the nurse?

The client’s pedal pulse in the right foot is not palpable.

A nurse is caring for a client who is experiencing chest pain. The client states, “I am really afraid because my brother died a year ago of a heart attack.” Which of the following responses is appropriate?

“This must be a frightening experience for you.”

A nurse is reviewing standard precautions with an assistive personnel (AP) who is new to the newborn nursery. For which of the following newborn care procedures should the nurse remind the AP to wear clean gloves?

Applying an identification bracelet

A nurse is collecting data from a client who is 1 day postoperative following a total hip arthroplasty and has deep-vein thrombosis. Which of the following findings should the nurse expect in the affected extremity?

Dull, aching pain

A nurse is contributing to the plan of care for a client who has bipolar disorder and is experiencing mania. Which of the following actions should the nurse include in the plan of care?

Decrease the level of environmental stimuli.

A nurse is caring for a client who is receiving morphine 2 mg subcutaneously every 4 hr for pain. The medication is available in a vial containing 10 mg/mL. How many ml should the nurse administer? (Round the answer to the nearest tenth. Use a leading zero if it applies. Do not use a trailing zero.)

0.2 ML

A nurse is caring for a client who is receiving prazosin. The client’s blood pressure is 100/60 mm Hg. Which of the following actions should the nurse take?

Instruct the client to stand up slowly.

A nurse is checking the home environment of a client for safety hazards. Which of the following items require intervention by the nurse?

The television set is turned to a loud volume.

A nurse is reinforcing teaching with a client about the importance of a low-sodium diet. Which of the following foods should the nurse recommend as the best choice to include in a low-sodium diet?

1 medium baked potato

A nurse in a prenatal clinic is reinforcing teaching with a client about a nonstress test. Which of the following statements should the nurse include?

“You will be asked to press a button when you feel your baby move.”

A nurse is preparing to instill an otic suspension into an adult client’s ear. Which of the following methods should the nurse plan to use?

Pull the auricle upward and backward.

A nurse is reinforcing teaching about colostomy care with a client. Which of the following client statements indicates an understanding of how to care for the colostomy?

“I will empty the colostomy bag when it is one-half full,”

A nurse is caring for a client who is postoperative following a hemicolectomy. Which of the following is the best indication that the client needs PRN pain medication?

The client reports pain.

A charge nurse in a long-term care unit is planning care for a group of clients. Which of the following care tasks should the nurse plan to delegate to an assistive personnel?

Measuring urine output every 2 hr for a client recently diagnosed with a urinary tract infection

A nurse is preparing to administer cefaclor to a preschooler who weighs 20 kg. The child is to receive cefaclor 30 mg/kg/day to divide equally every 8 hr. Available is cefaclor suspension 125 mg/5 mL. How many ml should the nurse administer for one dose? (Round to the nearest whole number. Use a leading zero if it applies. Do not use a trailing zero.)

8

A nurse in a provider’s office is reinforcing teaching with the parents of a school-age child who has an active case of Pediculosis humanus capitis. Which of the following should the nurse include in the teaching?

Wash the bed linens in hot water.

A nurse in an inpatient mental health facility is caring for a client who has bipolar disorder and is in an acute manic episode. Which of the following activities should the nurse suggest for the client?

Take a walk with a staff member,

A nurse is reinforcing healthy eating habits for an older adult client to ensure adequate dietary intake. Which of the following is an appropriate recommendation?

Snack between meals.

A nurse in a provider’s office is collecting data from a client who was discharged from the hospital 7 days ago following treatment for deep-vein thrombosis. Which of the following findings is the nurse’s priority? (Click on the “Exhibit” button for additional information about the client. There are three tabs that contain separate categories of data.)

The client takes ibuprofen daily to treat musculoskeletal pain.

A nurse is preparing to administer an IM injection to a client. To reduce the risk of a needlestick injury, the nurse should take which of the following actions?

Dispose of the used needle immediately in a puncture-proof sharps container.

A nurse is reviewing the laboratory results for a client who is at 32 weeks of gestation. For which following results should the nurse notify the provider?

Platelet count 90,000/mm3

A nurse is reinforcing teaching with a client who has a new prescription for levothyroxine. Which of the following statements indicates understanding of the instructions?

“I need to take this medication for the rest of my life.”

A nurse is preparing to administer purified protein derivative (PPD) to a client. The nurse should use which of the following routes?

Intradermal

A nurse in a clinic is reinforcing teaching with a client who has a new prescription for a combination contraceptive transdermal patch. Which of the following should the nurse include in the teaching?

Apply the patch to the lower abdomen,”

A nurse is reinforcing teaching with a parent about appropriate snacks for a toddler. Which of the following foods should the nurse include?

Graham crackers

A nurse is caring for a client who has COPD. The client tells the nurse, “I get short of breath during meals. It is too much trouble to eat.” Which of the following instructions should the nurse suggest?

“Have several small meals during the day.”

A nurse is caring for a school-age child whose family adheres to a vegan diet in the home. The nurse should recognize the child is at risk for deficiency of which of the following?

Vitamin D

A nurse working in a long-term care facility is assisting with the plan of care for a client who has Alzheimer’s disease. The nurse notes that the client wanders the halls at night and says she is looking for the bathroom. Which of the following interventions should the nurse recommend to include in the plan of care?

Label the client’s bathroom door.

A nurse is reinforcing discharge teaching with an older adult client’s family about safety precautions when administering a tap water enema to the client. Which of the following should the nurse include in the instructions?

Instruct the client to bear down during rectal tube insertion.

A client who is prescribed metoprolol for hypertension tells the nurse, “I don’t want to take this medication because makes me tired all the time.” Which of the following is the appropriate response?

“Let’s talk with your doctor about other options.”

A nurse is preparing to catheterize a client’s bladder to check for residual urine. The nurse should schedule this procedure at which of the following times?

Right after the client voids

A nurse on a mental health unit is discussing client rights with a group of coworkers. Which of the following statements should the nurse include?

“A client must withdraw consent for treatment in writing if he is competent to do so.”

A nurse is collecting data from a client who has diabetes mellitus. Which of the following findings indicates the client might be experiencing diabetic ketoacidosis?

Anuria

A nurse is reinforcing teaching with the parent of a toddler who has difficulty falling asleep. Which of the following instructions should the nurse include?

Establish a bedtime routine.

A nurse is caring for a client who is receiving warfarin 5 mg PO daily. Which of the following laboratory values should the nurse check prior to administering the medication? (Select all that apply.)

PT Serum potassium INR Serum sodium BUN

A nurse is caring for a client who had abdominal surgery 24 hours ago. Which of the following actions is the priority?

A. Assess fluid intake every 24 hours
B. Ambulate three times a day
C. Assist with deep breathing and coughing
D. Monitor the incision site for findings of infection
C

The priority action the nurse should take when using the airway, breathing, circulation approach to client care is to assist the client with deep breathing and coughing, which reduces the risk for postoperative pneumonia.

A nurse is talking with a client who has stage IV breast cancer. The nurse should recognize which of the following statements by the client as a constructive use of a defense mechanism?

A. I have experienced physical discomfort when intimate with my partner since my diagnosis
B. I wish other women would stop socializing with my partner
C. I told my doctor that I would like to start a support group for other women who are sick in my community
D. I used to mistrust my doctor, but now I know that she is the best one to care for me during my illness
C

This statement indicates that the client is using the constructive defense mechanism sublimation by devising a socially acceptable alternative to facing a reality that she does not wish to accept.

A nurse is caring for a client who has immunosuppression and a continuous IV infusion. Which of the following actions should the nurse take?

A. Assess the clients IV site every 8 hours
B. Check the clients WBC count every 48 hours
C. Monitor the clients mouth every 8 hours
D. Change the clients IV tubing every 48 hours
C

A nurse is caring for a 2-month-old infant who has Hirschsprung disease (HD). Which of the following areas should the nurse assess for manifestations of HD?

A. Eyes area
B. Chest area
C. Lower abdominal area
C

Hirschsprung disease is a condition that affects the large intestine (colon) and causes problems with passing stool. This is present at birth (congenital) as a result of missing nerve cells in the muscle of the baby’s colon

A nurse at a mental health clinic is caring for four clients. The nurse should recognize that which of the following clients is using dissociation as a defense mechanism?

A. A client forgets to buy their partner a birthday gift after a disagreement
B. A client who was abused as a child describes the abuse as if it happened to someone else.
C. A client who is shorter than average is verbally assertive with coworkers
D. A client states that they did not get a job promotion because the boss did not like them
B

A nurse is providing teaching to a client who has a new diagnosis of type 1 diabetes mellitus. The nurse should instruct the client to monitor for which of the following findings as a manifestation of hypoglycemia?

A. irritability
B. increased urination
C. vomiting
D. facial flushing
A

A nurse in an outpatient mental health clinic is caring for four clients. The nurse should recognize that which of the following clients is effectively using sublimation as a defense mechanism?

A. A client who transfers their anger about their job onto their family and then apologizes
B. A client who misses provider appointments because they say they are too busy
C. A client who channels their energy into a new hobby following the loss of their job
D. A client whose partner died 4 years ago sets a place for him at dinner each night
C

The nurse should identify that this client is using the defense mechanism of sublimation by channeling negative feelings over the loss of their job into a new hobby.

A hospice nurse is consulting with a client and her family about receiving home services. Which of the following statements should the nurse identify as an indication that the family understands home hospice care?

A. “We can expect the hospice nurse to provide support for us after our mother’s death.”
B. A hospice nurse will come to the house each time our mother needs pain medication
C. Now that my mother is receiving hospice services, we will not be able to get respite care
D. Hospice care focuses on arranging treatment that will prolong our mother’s life
A

Hospice care includes bereavement services after a family member’s death.

A nurse is caring for a client who has active pulmonary tuberculosis. Which of the following actions should the nurse take?

A. Wear a surgical mask when providing client care
B. Have visitors maintain a distance of 1.8m (6 feet) from the client
C. Restrict fresh flowers from the clients room
D. Assign the client to a private room with negative air pressure
D

A nurse is providing teaching to a client who is at 24 weeks of gestation and is scheduled for a 3-hr oral glucose tolerance test. Which of the following instructions should the nurse include in the teaching?

A. Limit your fat intake for 72 hours before the test
B. You will need to fast the night before the test
C. We will collect a urine sample the day after testing
D. A blood sample will be collected every 15 minutes during the test
B

A nurse on a pediatric unit has received change-of-shift report for four children. Which of the following children should the nurse assess first?

A. A 6month old infant who has croup and an O2 saturation of 92% on room air
B. A 15 year old adolescent who is 2 hour postop following an open reduction and internal fixation of the left ankle and is requesting pain medication
C. A 3 year old toddler who has gastroenteritis, moderate dehydration, and had 2 loose bowel movements over the past 24 hours
D. A 10-year-old child who is awaiting surgery for an appendectomy and experienced sudden relief from pain.
D

A nurse in a community center is providing an educational session to a group of clients about ovarian cancer. Which of the following manifestations of ovarian cancer should the nurse include in the teaching?

A. Diarrhea
B. Urinary retention
C. Purulent discharge
D. Abdominal bloating
D

A nurse is caring for a client who is postoperative after receiving moderate (conscious) sedation. The client suddenly becomes restless and reports feeling lightheaded. Which of the following actions should the nurse take?

A. Check the clients temperature
B. Prepare to administer acetylcysteine to the client
C. Place the client in the Trendelenburg position
D. Check the client’s oxygen saturation level
D

Restlessness and lightheadedness are indications of hypoxia.

A nurse working in an emergency department is triaging four clients. Which of the following clients should the nurse recommend for treatment first?

A. An older adult client who reports constipation of 4 days
B. A preschooler who has a skin rash
C. An adolescent who has a closed fracture
D. A nurse working in an emergency department is triaging four clients. Which of the following clients should the nurse recommend for treatment first?
D

A nurse is providing teaching for a client who has a fracture of the right fibula with a short leg cast in place and a new prescription for crutches. The client is non-weight-bearing for 6 weeks. Which of the following instructions should the nurse include in the teaching?

A. Adjust the crutches for comfort as needed
B. Use a three-point gait.
C. Wear leather soled shoes
D. Advance the affected leg first when walking upstairs
B

A nurse is preparing to initiate IV access for an older adult client. Which of the following sites should the nurse select when initiating the IV for this client?

A. Radial vein of the inner arm
B. Great saphenous vein of the leg
C. Dorsal plexus vein of the foot
D. Basilic vein of the hand
A

A nurse is planning to delegate client care tasks to an assistive personnel. Which of the following tasks should the nurse plan to delegate to the AP?

A. Perform gastrostomy feedings through a clients established gastrostomy tube
B. Administer glycerin suppository to a client who is constipated
C. Provide instructions about client care to a family member over the telephone
D. Teach a client how to measure their own blood pressure
A

A nurse is caring for a newborn immediately after delivery. Which of the following interventions should the nurse implement to prevent heat loss by conduction?

A. Dry the newborn immediately after birth
B. Maintain an ambient room temp of 24 celcius
C. Use a protective cover on the scale when weighing the infant
D. Place the newborns bassinet away from outside windows
C

Conduction is the process of losing heat through physical contact with another object or body. For example, if you were to sit on a metal chair, the heat from your body would transfer to the cold metal chair.

Convection is the process of losing heat through the movement of air or water molecules across the skin

A nurse is caring for a client who is receiving a continuous heparin infusion. Which of the following lab tests should the nurse review prior to adjusting the client’s heparin?

A. aPTT
B. PT
C. INR
D. WBC count
A

A nurse administers an incorrect dose of medication to a client. The nurse recognizes the error immediately and completes an incident report. Which of the following facts related to the incident should the nurse document in the client’s medical record?

A. Completion of the incident report
B. Time the medication was given
C. Reason for the medication error
D. Notification of the pharmacist
B

A nurse is providing education to the parent of a school-age child who has asthma. Which of the following statements by the parent indicates an understanding of the teaching?

A. “I will administer aspirin to my child to treat pain or fever”
B. “I will record an average of three readings from my child’s peak expiratory flow meter”
C. “I will place carpet in my child’s bedroom to control allergens”
D. “I will make sure my child receives a yearly influenza immunization.”
D

— Children who have asthma should be immunized and protected from infections.

A nurse is caring for a client who has type 1 diabetes mellitus and reports severe ankle pain after falling off a stepstool at home. Which of the following prescriptions should the nurse clarify with the provider?

A. Obtain capillary blood glucose level every 2 hours
B. Check the neurovascular status of the client’s lower extremities every hour
C. Apply a cold pack to the client’s ankle for 30 min every hour.
D. Maintain the affected ankle elevated and immobilized
C

A nurse is assessing a client who has major depressive disorder and is taking amitriptyline. Which of the following findings should the nurse identify as an adverse effect of the medication?

A. Diarrhea
B. Frequent urination
C. Excessive salivation
D. Blurred vision
D

The nurse should identify blurred vision as an adverse effect of amitriptyline and notify the provider. Other adverse effects include constipation, urinary retention, and dry mouth.

A nurse is caring for a client who has signed an informed consent form to receive electroconvulsive therapy (ECT). The client states to the nurse, “I’m not sure about this now. I’m afraid it’s too risky.” Which of the following responses should the nurse make?

A. “Perhaps you think the ECT is dangerous, but I’ve seen it have good results”
B. “You have the right to change your mind about this procedure at any time.”
C. “Everyone gets a little nervous about this procedure as the time for it approaches”
D. “Your doctor wouldn’t have suggested ECT if they didn’t think it would help you”
B

A nurse is preparing to administer mannitol 0.2 g/kg IV bolus over 5 min as a test dose to a client who has severe oliguria. The client weighs 198 lbs. What is the amount in grams the nurse should administer?
18g

A nurse is preparing to administer an IM injection to a client who is obese. Which of the following actions should the nurse plan to take?

A. Select a 1 inch needle
B. Use a 45 degree angle when inserting the needle
C. Use the ventrogluteal site
D. Pinch the skin up during injection
C

A nurse is providing discharge teaching to a client who is to receive home oxygen therapy. Which of the following instructions should the nurse include in the teaching

A. Check the functioning of oxygen equipment once each week
B. Wear clothing made with cotton fabrics while oxygen is in use
C. Apply petroleum-based lubricant to the nares as needed
D. Store full oxygen tanks on their side
B

A nurse is providing teaching to the guardians of a newborn about measures to prevent SIDS. Which of the following guardian statements indicates an understanding of the teaching?

A. “I will not allow anyone to smoke near my baby.”
B. “I will place bumper pads in my baby’s crib”
C. “My baby’s head should be placed on a pillow for sleeping”
D. “My baby should sleep in a side-lying position”
A

— This statement by the guardian indicates an understanding of the nurse’s instructions. Research indicates a strong correlation between exposure to cigarettes smoke and the occurrence of SIDS.

A nurse is assessing a client following a vaginal delivery and notes heavy loch and a boggy fundus. Which of the following medications should the nurse expect to administer?

A. Nalbuphine
B. Terbutaline
C. Oxytocin
D. Magnesium sulfate
C

A nurse on a medical-surgical unit is caring for a client who has a new diagnosis of terminal cancer. The client tells the nurse that they would like to go home to be with family and loved ones. Which of the following actions should the nurse take?

A. Contact the facility chaplain to visit with the client
B. Explain the process of leaving the facility against medical advice
C. Make a referral for social services
D. Encourage the client to continue with inpatient care
C

A nurse is caring for a client who has a clogged percutaneous gastrostomy feeding tube. Which of the following actions should the nurse take first?

A. Obtain a prescription for the client to receive an enzyme product
B. Aspirate the client’s tube
C. Flush the client’s tube with 30 mL of water
D. Change the position of the client
D

A nurse is preparing to assist with a thoracentesis for a client who has pleurisy. The nurse should plan to perform which of the following actions?

A. Administer a bowel preparation the night before the procedure
B. Place the client on bed rest for 24 hours after the procedure
C. Perform pulmonary function tests following the procedure
D. Instruct the client to avoid coughing during the procedure
D

A charge nurse is providing an educational session about infection control for a group of staff nurses. Which of the following statements by one of the staff nurses indicates an understanding of isolation precautions?

A. “Droplet precautions should be initiated for client who tests positive for measles”
B. “A client who requires airborne precautions should be placed in a negative pressure airflow room”
C. “Airborne precautions should be initiated for a client who has Clostridium Difficile”
D. “A client who is immunocompromised should be placed in a negative pressure airflow room”
B

A nurse is providing dietary teaching to a client who has a new prescription for phenelzine. Which of the following food recommendations should the nurse make? (Select all that apply.)

A.Broccoli
B. Yogurt
C. Pepperoni pizza
D. Cream cheese
E. Bologna sandwich
A, B, D

A nurse is assessing an older adult client who has pneumonia. Which of the following findings should the nurse expect?

A. Paradoxical chest movement
B. Subcutaneous emphysema
C. Acute confusion
D. Distended neck veins
C

A nurse is providing teaching about home care to the parents of a child who has autism spectrum disorder. Which of the following instructions should the nurse include?

A. Maintain a flexible daily schedule for the client
B. Use a reward system to modify the child’s behavior
C. Provide a variety of family members to care for the child
D. Administer alprazolam as needed to reduce the child’s anxiety
B

A nurse is preparing to administer lactated Ringer’s 1,500 mL IV to infuse at 50 mL/hr. The drop factor of the manual IV tubing is 15 gtt/mL. The nurse should set the manual IV infusion to deliver how many gtt/min?
12.5 gtt/min (or 13 if rounding to nearest whole number)

A nurse is caring for a client following a vacuum-assisted birth. The nurse should monitor the client for which of the following complications related to vacuum-assisted birth?

A. Constipation
B. Urinary urgency
C. Cervical laceration
D. Retained placenta
C

A nurse is assessing a client who has COPD. Which of the following findings should the nurse expect?

A. Weight gain
B. Decrease in anteroposterior diameter of the chest
C. HCO3- 24 mEq/L
D. pH 7.31
D

COPD causes hypoventilation, which leads to retained CO2. High amounts of CO2 lead to acidosis (pH below 7.35)

A nurse is providing discharge teaching for the parents of a preschool-age child who has a new prescription for amoxicillin/clavulanate suspension. Which of the following instructions should the nurse include in the teaching? (select all that apply)

A. “You will give the medication every 4 hours”
B. “Shake the medication bottle feel before each dose is given.”
C. “Store the medication in the refrigerator.”
D. “Report diarrhea to the provider immediately.”
E. “Discard the unused portion of medication after 21 days”
B, C, D

A nurse is preparing a client for a paracentesis. Which of the following actions should the nurse take?

A. Instruct the client to void
B. Position the client on their left side
C. Insert an IV catheter
D. Prepare the client for moderate (conscious) sedation
A

A nurse is caring for a client who has active TB. Which of the following actions should the nurse plan to take to prevent the transmission of the disease?

A. Initiate contract precautions for the client upon admission
B. Restrict visitors from entering the client’s room during hospitalization
C. Wear a surgical mask while providing care for the client
D. Have the client wear a surgical mask while being transported outside the room
D

A nurse is caring for a client who has deep-vein thrombosis. Which of the following actions should the nurse take?

A. Teach the client to massage the affected extremity
B. Instruct the client to elevate the affected extremity when sitting
C. Assess pulses proximal to the affected area
D. Apply a cold compress to the affected extremity
B

A nurse is assessing a client who has a chest tube. Which of the following findings should the nurse expect?

A. Drainage system located above the clients chest wall
B. Continuous bubbling in the water seal chamber
C. Occlusive dressing on the insertion site
D. Drainage of 125 mL/hr
C

All chest tube dressings should be an occlusive, air tight dressing to prevent air leaks. In order to keep the dressing occlusive and to avoid an air leak, tape all the connections from the insertion site of the patient to the chest drainage unit.

If the water seal is continuously bubbling, you should suspect an air leak. Think of the lungs as wrapped in plastic. An air leak occurs when there is a hole in the plastic wrap allowing air to escape from the lung tissue into to the pleural cavity.

A nurse on an inpatient mental health unit is monitoring a visit between a client who has a history of aggressive behavior and the client’s partner. Which of the following observations should the nurse identify as an indication for potential violence?

A. The client is taking numerous deep, measured breaths
B. The client is calmly telling their partner that “the staff here is so controlling”
C. The client is sitting with their head in their hands and appears to be crying
D. The client is pacing around the chair in which their partner is sitting
D

Hyperactivity and pacing indicates that this client is at risk for violent behavior. The nurse should assess the situation further and attempt to de-escalate the situation by speaking to the client in a low, calm voice using short sentences.

A nurse is caring for four clients. Which of the following tasks should the nurse delegate to an assistive personnel (AP)?

A. Evaluate dietary intake for a client who has anorexia
B. Measure the vital signs of a client who just returned from the PACU
C. Arrange the lunch tray for a client who has a hip fracture
D. Assess I&O for a client who is relieving dialysis
C

A nurse is caring for a client who has a prescription for chlorpromazine. Which of the following findings should the nurse identify as an indication that the medication is effective?

A. Decreased blood pressure
B. Decreased hallucinations
C. Decreased cholesterol
D. Decreased esophageal reflux
B

The nurse should recognize that chlorpromazine is an antipsychotic medication administered to decrease hallucinations and other manifestations of schizophrenia.

A nurse is performing an abdominal assessment on a client. Identify the sequence of actions the nurse should take.

Palpatation
Inspection
Percussion
Auscultation

  1. Inspection
  2. Auscultation
  3. Percussion
  4. Palpitation

Go from least invasive to most invasive. Touch the abdomen last, you do not want to move around any organs

A nurse is updating the plan of care for a client who is 48 hr postoperative following a laryngectomy and is unable to speak. Which of the following actions should the nurse plan to take first?

A. Determine the client’s reading skills
B. Instruct the client on the technique for esophageal speech
C. Provide the client with an alphabet board
D. Show the client how to use artificial larynx
A

The first action the nurse should take when using the nursing process is to assess the client. By determining the client’s level of reading skills and cognition, the nurse can best provide the client with a variety of customized techniques to practice and use after verbal skills are lost.

A nurse is planning care for a client who has rheumatoid arthritis. Which of the following interventions should the nurse include in the plan?

A. Encourage the client to take a cool sponge bath each morning
B. Administer opioid analgesia
C. Increase the client’s dietary iron intake
D. Restrict the client’s intake of foods high in purines
C

A nurse is caring for a client who has acute blood loss following a trauma. The client refuses a blood transfusion that might potentially save their life. Which of the following actions should the nurse take first?

A. Document the client’s refusal in the medication record
B. Honor the client’s decision to refuse the blood transfusion
C. Explore the client’s reasons for refusing the treatment
D. Discuss the client’s refusal with the provider
C

A nurse is caring for a group of clients. Which of the following clients should the nurse attend to first?

A. An older adult client who is anxious and attempting to pull out an IV line
B. A middle adult client who is reporting nausea after receiving pain medication
C. An older adult client who has kidney failure and returned from dialysis 4 hr ago
D. A middle adult client who has a terminal illness and is requesting a visit from the chaplain
A

A nurse is preparing to teach about dietary management to a client who has Crohn’s disease and an enteroenteric fistula. Which of the following nutrients should the nurse instruct the client to decrease in their diet?

A. Calories
B. Protein
C. Potassium
D. Fiber
D

The nurse should instruct the client who has Crohn’s disease and an enteroenteric fistula to consume a low-fiber diet to reduce diarrhea and inflammation.

A nurse manager is preparing an educational session for nursing staff about how to provide cost-effective care. Which of the following method should the nurse include in the teaching?

A. Delegate non-nursing tasks to ancillary staff
B. Stock client rooms with extra supplies
C. Assign dedicated equipment to each client’s room
D. Change continuous IV infusion tubing every 24 hours
A

A nurse is assessing a newborn following a vaginal delivery. Which of the following findings should the nurse report to the provider?

A. Heart rate 136/min
B. Nasal flaring
C. Transient strabismus
D. Overlapping of sutures
B

–A newborn heart rate is normally 120-160bpm
–Nasal flaring indicates respiratory distress in a newborn.
–Transient misalignment/ strabismus of a baby’s eyes is very common up to the age of four months. The eyes may be intermittently esodeviated or exodeviated, but by three months of age, the eyes should be straight. Any strabismus that is apparent after that time is a source of concern
–In an infant only a few minutes old, the pressure from delivery compresses the head. This makes the bony plates overlap at the sutures and creates a small ridge. In the next few days, the head expands and the overlapping disappears

A nurse is assessing a client who has a decreased visual acuity due to cataracts. The nurse should identify that which of the following physiological changes is the cause for the client’s visual loss?

A. An increase in the intra-ocular pressure
B. Deterioration of the macula
C. Increased opacity of the lens
D. Vitreous hemorrhage
C

A nurse must recommend clients for discharge in order to make room for several critically injured clients from a local disaster. Which of the following clients should the nurse recommend for discharge?

A. A client who has cellulitis and is receiving oral antibiotics every 8 hr
B. A client who is postoperative following an upper endoscopy procedure and is alert but does not have a gag reflex
C. A mother and their newborn 12 hr postdelivery
D. A client who has lower extremity weakness and is newly admitted for observation
A

Oral antibiotics may be taken at home, IV antibiotics would require admission.
A patient without a gag reflex is at risk for aspiration and should not go home.
A patient with weakness is a fall risk and requires observation in a hospital

A nurse in an emergency department is caring for a client who is unconscious and requires emergency medical procedures. The nurse is unable to locate members of the client’s family to obtain consent. Which of the following actions should the nurse take?

A. Contact the facility’s ethics committee
B. Obtain consent from the client’s employer
C. Limit care to comfort measures
D. Proceed with provision of medical care
D

A nurse is creating a plan of care for a newly admitted child. Which of the following actions should the nurse include the plan? (see exhibit)

History and physical:
8 year old male admitted with cystic fibrosis
Reports shortness of breath
Wheezing throughout lung fields
Productive cough with thick sputum

Graphic Record:
Heart rate 108/min
Respiratory rate 26/min
Temperature 37.2 (98.9)
Blood pressure 100/62
Oxygen saturation 92%

Diagnostic results:
Sputum culture: Burkholderia cepacia

A. Initiate droplet isolation precautions
B. Keep the child on NPO status for 12 hours
C. Maintain the child on bed rest for 24 hours
D. Administer high-dose antibiotic therapy
D

This bacteria is spread through contact, not droplet

A nurse is providing teaching about lithium to a client who has bipolar disorder. Which of the following statements should the nurse include in the teaching?

A. “Expect to have blurred vision while taking this medication”
B. “Notify your provider if you experience increased thirst”
C. “You might be unable to have an orgasm while taking this medication”
D. “You should take this medication on an empty stomach”
B

A nurse is reviewing the ABG values of a client. The client has a pH of 7.2, PaCO2 of 60 mmHg, and HCO3- of 25 mEq/L. The nurse should identify that the client has which of the following acid-base imbalances?

A. Respiratory Alkalosis
B. Metabolic Alkalosis
C. Respiratory Acidosis
D. Metabolic Acidosis
C

pH 7.2= low
PaCO2 60= high

Opposites= respiratory
Low pH= acidosis

A nurse is providing teaching about advance directives to a middle adult client. Which of the following client responses indicates an understanding of the teaching?

A. “I can designate my partner as my health care surrogate”
B. “I am only 40 years old, so I don’t need to worry about this yet”
C. “I will need a lawyer’s help to draw up the documents”
D. “I understand that my family can alter my advance directives if I become incapacitated”
A

A charge nurse notices that one of the nurses on the shift frequently violates unit policies by taking an extended amount of time for break. Which of the following statements should the charge nurse make to address this conflict?

A. “I would like to talk to you about the unit policies regarding break time.”
B. “If you continue to take a long lunch break, I will have to report this to the nurse manager”
C. “Have you thought about how your extended lunch breaks affect the other members of our team?”
D. “Did you inform the other members of your team about when you left and returned from break?”
A

— The charge nurse is dealing with the conflict in a cooperative, positive manner by using this statement to open the conversation in a nonthreatening way. The focus is on the length of the break time and is not a personal affront.

A nurse manager is preparing to teach a group of newly licensed nurses about effective time management. Which of the following steps of the time management process should the nurse manager include as the priority?

A. Organizing the work environment
B. Delegating assigned tasks appropriately
C. Making a list of activities to complete
D. Rewarding yourself for accomplishing goals
C

A nurse is teaching about adverse effects with a client who is starting to take captopril. Which of the following findings should the nurse identify as an adverse effect of the medication to report to the provider?

A. Tinnitus
B. Cough
C. Polyuria
D. Blurred vision
B

The client can develop a cough due to a buildup of bradykinin in the lungs.

A home health nurse is caring for a group of older adult clients. The nurse should initiate a referral to the Program of All-Inclusive Care for the Elderly (PACE) for which of the following clients?

A. A client whose family requests hospital based hospice care
B. A client who requires transfer to a skilled care facility
C. A client who qualifies for telehealth for pacemaker diagnostics
D. A client whose caregiver requests adult day care services
D

A nurse is assessing a client for compartment syndrome. Which of the following findings should the nurse expect?

A. Fever
B. Shortened femoral neck
C. Edema
D. Dark brown urine
C

A nurse in an emergency department is caring for a child who has a fever and fluid-filled vesicles on the trunk and extremities. Which of the following interventions should the nurse identify as the priority?

A. Encourage oral fluids
B. Apply topical calamine lotion
C. Administer acetaminophen as an antipyretic
D. Initiate transmission based precautions
D

A nurse in an outpatient mental health facility is assessing a child who has autism spectrum disorder. Which of the following manifestations should the nurse expect?

A. Strict adherence to routines
B. Difficulty paying attention to tasks
C. Disobedience to authority figures
D. Excessive anxiety when separated from parents
A

A nurse is caring for a client who recently signed an informed consent form to donate a kidney to her sibling who has end-stage kidney disease. The donor states to the nurse, “I don’t want my brother to die, but what if I need this kidney one day?” Which of the following responses should the nurse make?

A. “I understand your hesitation, but I’m very proud of you for making the right decision”
B. “Organ donation from a first degree relative is your brother’s best chance of survival”
C. “You’re afraid that your other kidney will fail at some point after the organ donation”
D. “I know this process won’t be easy, but you should focus on saving your brother’s life”
C

A nurse is preparing to administer a blood transfusion to a client. Which of the following procedures should the nurse follow to ensure proper client identification?

A. Check the client’s blood type and crossmatch it against the providers orders
B. Ask the client to state their blood type prior to beginning blood administration
C. Compare information on the blood product to the informed consent form
D. Verify the client and blood product information with another licensed nurse.
D

A nurse is preparing to transfer a client from the ICU to the medical floor. The client was recently weaned from mechanical ventilation following a pneumonectomy. Which of the following information should the nurse include in the change-of-shift report?

A. The last time the provider evaluated the client
B. The client’s most recent ventilator settings
C. The time of the client’s last dose of pain medication
D. The frequency in which the client presses the call buttom
C

A nurse is reviewing the lab report of a client who has end-stage kidney disease and received hemodialysis 24 hr ago. Which of the following lab values should the nurse report to the provider?

A. Platelets 268,000/mm3
B. Calcium 9.2 mg/dL
C. WBC 5,200/mm3
D. Sodium 148 mEq/L
D

Sodium is elevated and could indicate issues with renal

A nurse is caring for a client who has fluid volume overload. Which of the following tasks should the nurse delegate to an assistive personnel (AP)?

A. Palpate the degree of edema
B. Regulate IV pump fluid rate
C. Measure the client’s daily weight
D. Assess the client’s vital signs
C

A nurse is caring for an older adult client who is experiencing chronic anorexia and is receiving enteral tube feedings. Which of the following laboratory values indicates that the client needs additional nutrients added to the feeding?

A. Creatinine 1.1 mg/dL
B. Albumin 2.8 g/dL
C. Triglycerides 100 mg/dL
D. Alkaline phosphatase 118 units/L
B

A home health nurse is evaluating a school-age child who has cystic fibrosis. The nurse should initiate a request for a high-frequency chest compression vest in response to which of the following parent statements?

A. “My child doesn’t like to sit still for nebulizer treatments”
B. “I think that my child has been running a fever over the last couple of days”
C. “My child has only a small amount of mucus after percussion therapy”
D. “I am concerned about my child’s future participation in team sports”
C

A nurse is assessing for correct placement of a client’s NG feeding tube prior to administering a bolus feeding. Which of the following actions should the nurse take?

A. Insert air in the tube and listen for gurgling sounds in the epigastric area
B. Aspirate contents from the tube and verify the pH level
C. Review the medical record for previous x-ray verification of placement
D. Auscultate the lungs for adventitious breath sounds
B

A nurse is reviewing the lab results of a toddler who has hemophilia A. Which of the following aPTT values should the nurse expect?

A. 11 seconds
B. 22 seconds
C. 30 seconds
D. 45 seconds
D

A nurse is assessing a client whose partner recently died. The client states, “I don’t know what to do without my partner. Life is just not worth living.” Which of the following responses should the nurse make?

A. “It’s natural for you to feel this way now, but things will get better with time”
B. “You seem to be having a difficult time right now”
C. “Why do you feel like your life isn’t worth living?”
D. “You’d be surprised how many people experience these feelings”
B

An RN is observing a licensed practical nurse (LPN) and an assistive personnel (AP) move a client up in bed. For which of the following situations should the nurse intervene?

A. The LPN and AP lower the side rails before lifting the client up in bed
B. Prior to lifting the client, the LPN and AP raise the bed to waist level
C. The LPN and the AP grasp the client under this arms to life him up in bed
D. The LPN and the AP ask the client to flex his knees and push his heels into the bed as they lift
C

A nurse is caring for a client who is in the resuscitation phase of burn injury. Which of the following findings should the nurse expect?

A. Decreased hematocrit
B. Hypokalemia
C. Hyponatremia
D. Increased albumin
C

the initial resuscitation period (between 0 and 36 h). characterized by hyponatremia and hyperkalemia d/t sodium loss in burn tissue and tissue necrosis.
Hyponatremia is due to extracellular sodium depletion following changes in cellular permeability.

A nurse on a mental health unit is conducting a mental status examination (MSE) on a new admitted client. Which of the following components of the MSE is the priority for the nurse to assess?

A. Mood
B. Speech
C. Ideas of self harm
D. Memory loss
C

A nurse manager is planning to use a democratic leadership style with the nurses on the unit. Which of the following actions by the nurse manager demonstrates a democratic leadership style?

A. Avoids initiating change
B. Seeks input from the other nurses
C. Makes decisions quickly
D. Limits the amount of feedback to the staff
B

A home health care nurse is developing a teaching plan for a client who has a new ileostomy. Which of the following instructions should the nurse include?

A. Limit intake of fluids to 1,000 mL daily
B. Take a laxative if no stool has passed after 12 hr
C. Empty the appliance when it is one-third to one-half full
D. Change the entire pouch system every 1-2 days
C

A nurse manager is preparing an educational session about advocacy to a group of nurses. The nurse manager should include which of the following information in the teaching?

A. Advocacy is a leadership role that helps others to self- actualize
B. Subordinates are advocates for the nurse manager
C. Advocacy encourages clients to rely on health care staff for decision making
D. Nurse managers should distrust people who advocate against inappropriate professional practices
A

A nurse is caring for a client who has hyperthyroidism. Which of the following findings should the nurse expect?

A. Dry, coarse hair
B. Bradycardia
C. Tremors
D. Periorbital edema
C

Tremors are a manifestation of hyperthyroidism, along with tachycardia, diaphoresis, weight loss despite increased hunger, insomnia, and exophthalmia.

A nurse is assessing a client after administering epinephrine for an anaphylactic reaction. Which of the following findings should the nurse identify as an adverse effect of this medication?

A. Hypotension
B. Report of tinnitus
C. Report of chest pain
D. Ecchymosis
C

A nurse manager is reviewing clients’ rights with the nurses on the unit. The nurse manager should tell the nurses that informed consent promotes which of the following ethical principles?

A. Autonomy
B. Nonmaleficience
C. Justice
D. Fidelity
A

A nurse is assessing a school-age child who has bacterial meningitis. Which of the following findings should the nurse expect?

A. Nuchal rigidity
B. Weight gain
C. Tinnitus
D. Positive Trendelenburg sign
A

A nurse in a mental health clinic is assessing a client who has a history of seeking counseling for relationship problems. The client shows the nurse multiple superficial self-inflicted lacerations on their forearm The nurse should identify these behaviors as characteristics of which of the following personality disorders?

A. Borderline
B. Antisocial
C. Histrionic
D. Paranoid
A

A nurse is caring for a client who has a fecal impaction. Which of the following actions should the nurse take when digitally evacuating the stool?

A. Place the client in the lithotomy position
B. Elicit a vagal response by performing gentle rectal stimulation
C. Administer oral bisacodyl 30 min prior to the procedure
D. Insert a lubricated gloved finger and advance along the rectal wall
D

Lithotomy position: a supine position of the body with the legs separated, flexed, and supported in raised stirrups, originally used for lithotomy and later also for childbirth. For fecal impaction, place pt in sims position (having a patient lie on their left side, left hip and lower extremity straight, and right hip and knee bent. It is also called lateral recumbent position.)

A nurse is caring for a client who has cancer and is deciding between two treatment plans. The client asks the nurse for assistance in making the decision. Which of the following responses should the nurse make?

A. “I understand this is a difficult decision”
B. “Tell me more about your understanding of the options”
C. “You will make the right choice”
D. “I will ask your provider to talk with you further”
B

A nurse is assigning task roles for a group of clients in a community mental health clinic. Which of the following tasks should the nurse assign to the member of the group functioning as the orienteer?

A. Measuring the group’s work against the assigned objectives
B. Noting the progress of the group toward assigned goals
C. Sharing experiences as an authority figure
D. Offering new and fresh ideas on an issue
B

A nurse is caring for a client who has a pulmonary embolism. The client is receiving heparin via continuous IV infusion at 1,200 units/hr and warfarin 5 mg PO daily. The morning laboratory values for the client are aPTT 98 seconds and INR 1.8. Which of the following actions should the nurse take?

A. Prepare to administer vitamin K1
B. Prepare to administer alteplase
C. Withhold the heparin infusion
D. Withhold the next dose of warfarin
C

A nurse on an inpatient unit is caring for a client who has schizophrenia and recently started taking risperidone. Which of the following actions should the nurse take?

A. Implement fall precautions for the client
B. Monitor the client’s thyroid function
C. Place the client on a fluid restriction
D. Discontinue the medication if hallucinations occur
A

A nurse is admitting a client to the psychiatric unit after attempting suicide. The client states, “My family does not care whether I live or die.” Which of the following responses should the nurse make?

A. “I’m sure your family does not want you to die”
B. “Why would you believe such things”
C. “How does this make you feel”
D. “You should talk to your family about your feelings”
C

A nurse in an emergency department is assessing a school-age child who was brought in by their parents and has scald burns to both hands and wrists. The nurse suspects physical abuse. Which of the following actions should the nurse take?

A. Discuss the suspicion of physical abuse with the provider
B. Confront the parents with the suspicion of physical abuse
C. Ask the hospital security to detain and question the parents
D. Contact Child Protective Services
D

A nurse is caring for a client who has a new prescription for clonidine. The nurse should inform the client that which of the following findings is an adverse effect of this medication?

A. Diarrhea
B. Dry mouth
C. Photophobia
D. Brusiing
B

A nurse is caring for a client who has a magnesium level of 2.7 mEq/L. Which of the following interventions should the nurse plan to take?

A. Initiate continuous cardiac monitoring
B. Administer 40 mEq/L potassium chloride PO with orange juice
C. Provide a diet rich in legumes, nuts, and green vegetables
D. Monitor the client for tetany
A

normal mg level is 1.5-2.5. Elevated magnesium can cause cardiac arrhythmias

A rural community health nurse is developing a plan to improve health care delivery for migrant farmworkers. To identify health services data for this minority group, the nurse should gather information from which of the following sources?

A. Agency for Healthcare Research and Quality
B. National Institutes of Health
C. Department of Agriculture
D. World Health Organization
A.

A nurse is planning care for a client who has rheumatoid arthritis and has moderate to severe pain in multiple joints. Which of the following actions should the nurse plan to take?

A. Perform ADLs for the client to promote rest
B. Allow for frequent rest periods throughout the day
C. Use heat to reduce joint inflammation
D. Develop a daily schedule for acetaminophen up to 6g/day that covers peak periods of pain
B

The nurse should encourage clients who have rheumatoid arthritis to balance rest with exercise to maintain muscle strength, joint function, and range of motion.

A nurse manager in a long-term care facility is having difficulty with staffing for weekend shifts and is planning to implement some changes to the scheduling procedure. Which of the following actions should the nurse manager take first?

A. Form a committee of staff members to investigate current staffing issues.
B. Provide support to staff members who are resistant to staffing changes
C. Schedule a staff meeting to present the different options to staff members
D. Give the staff members advance written notice of staffing changes
A

A nurse is assessing a client following a colonoscopy. Which of the following findings should indicate to the nurse that the client is hemorrhaging?

A. Sudden drop in heart rate
B. Rapid decrease in blood pressure
C. Client reports a feeling of abdominal fullness
D. Client reports pain as an 8 on a scale of 0 to 10
B

A nurse is caring for a client who is in the fourth stage of labor and is receiving oxytocin via continuous IV infusion. Which of the following assessments is the nurse’s priority?

A. Amount of vaginal bleeding
B. Amount of urinary output
C. Pain level
D. Fundal height
A

Oxytocin increases uterine contractions, which increases the risk of hemorrhaging

A nurse is caring for a school-age child who has dehydration and is receiving an oral rehydration solution. Which of the following laboratory results indicates that the treatment regimen is effective?

A. Hematocrit 45%
B. Urine specific gravity 1.035
C. Serum sodium 138 mEq/L
D. BUN 19 mg/dL
C

Serum of 138 is within normal range (135-145), which indicates a balanced hydration status

A nurse is assessing a newborn’s heart rate. Which of the following actions should the nurse take?

A. Assess the apical pulse while the newborn is crying
B. Palpate the radial pulse for 30 seconds
C. Listen to the apical pulse while palpating the radial pulse
D. Auscultate the apical pulse at least 1 min
D

We assess a brachial pulse and not a radial pulse for a newborn, too

A nurse is caring for a client who is taking valproic acid for seizure control. For which of the following adverse effects should the nurse monitor and report?

A. Weight loss
B. Jaundice
C. Bradycardia
D. Polyuria
B

Valproic acid can cause valproate hepatotoxicity

A nurse is providing information to a client immediately before his scheduled Romberg test. Which of the following statements should the nurse make?

A. “You will be standing with your feet 1 foot apart”
B. “You will place and hold your hands on your hips”
C. “I will be standing across the room from you to evaluate your sense of balance”
D. “I will be checking you once with your eyes open and once with them closed”
D

The Romberg test is a test of the body’s sense of positioning (proprioception), which requires healthy functioning of the dorsal columns of the spinal cord. The Romberg test is used to investigate the cause of loss of motor coordination (ataxia)

The test is performed as follows:
The patient is asked to remove his shoes and stand with his two feet together. …
The clinician asks the patient to first stand quietly with eyes open, and subsequently with eyes closed. …
The Romberg test is scored by counting the seconds the patient is able to stand with eyes closed.

A nurse is assessing a client who is at 11 weeks of gestation and reports drinking ginger tea. Which of the following findings indicates the client’s use of ginger tea is effective?

A. The client reports a decrease in episodes of nausea
B. The client reports a decrease in breast tenderness
C. The client reports a decrease in headaches
D. The client reports a decrease in urinary frequency
A

A nurse is assessing an infant who has hydrocephalus and is 6 hr postoperative following placenta of a VP shunt. Which of the following findings should the nurse report to the provider?

A. Heart rate 122/min
B. Irritability when being held
C. Hypoactive bowel sounds
D. Urine specific gravity 1.018
B

A client is receiving IV fluids at 150 mL/hr. Which of the following findings indicates that the client is experiencing fluid overload?

A. Oliguria
B. Bradycardia
C. Dyspnea
D. Poor skin turgor
C

Fluid overload would present as increased urinary output (oliguria= small urine output), dyspnea/ shortness of breath caused by extra fluid entering your lungs and reducing your ability to breathe normally, we could see a high or low HR (usually a bounding pulse), and edema (poor skin turgor is seen with dehydration/ hypovolemia)

A nurse in an emergency department is caring for a client who is at 9 weeks of gestation and reports nausea and vomiting for the past 2 days. Which of the following findings should the nurse expect?

A. Hgb 15 g/dL
B. Urine specific gravity 1.052
C. Urine osmolarity 300 mOsm/ kg
D. Hct 44%
B

The nurse should recognize this urine specific gravity is significantly elevated above the expected reference range of 1.005 to 1.030. An increased urine specific gravity indicates dehydration from vomiting.

A nurse is developing a client education program about osteoporosis for older adult clients. The nurse should include which of the following variables as a risk factor for osteroporosis?

A. Obesity
B. Acromegaly
C. Estrogen replacement therapy
D. Sedentary lifestyle
D

When women lose estrogen during menopause, that puts them at greater risk for osteoporosis.

A nurse in the emergency department is assessing a preschooler who has a facial laceration. The nurse should identify which of the following findings as a potential indication of child sexual abuse?

A. The child exhibits discomfort while walking
B. The child has thin extremities
C. The child has bruises on the upper back
D. The child is wearing a stained shirt
A

A nurse in a providers office is assessing an adolescent who has been taking ibuprofen for 6 months to treat juvenile idiopathic arthritis. Which of the following questions should the nurse ask to assess for an adverse effect of his medication

A. “have you experienced muscle stiffness?”
B. “have you had any stomach pain or bloody stools?”
C. “have you experienced a dry cough?”
D. “have you noticed an increase in urine output?”
B

A nurse is teaching a client who is at 20 weeks of gestation about common discomforts associated with pregnancy. Which of the following statements by the client indicates an understanding of the teaching?

A. “I will decrease my intake of high fiber foods”
B. “I will apply hydrocortisone cream if I develop a rash on my face”
C. “I will sleep flat on my stomach if I develop back pain”
D. “I will wear a supportive bra overnight”
D

This helps to decrease pain associated with engorgement or swelling

A nurse at an urgent care clinic is assessing a client who reports impaired vision in one eye. Which of the following reports by the client should indicate to the nurse that the client has a detached retina?

A. Halos around lights
B. Floating dark spots
C. Pain in the affected eye
D. Blurred vision
B

A nurse is preparing a sterile field to perform a sterile dressing change. Which of the following interventions should the nurse use to maintain surgical aseptic technique?

A. Hold hands folded below the waist after donning sterile gloves
B. Pick up and pour solutions with the palm of the hand covering bottle labels
C. Keep sterile items within a 1.3cm (0.5 in) boarder of the sterile drape
D. Maintain sterile objects within the line of vision
D

A nurse is admitting a client who has pneumonia. The nurse should initiate which of the following isolation precautions for the client?

A. Droplet
B. Airborne
C. Contact
D. Protective environment
A

A nurse is providing discharge teaching about disease management for a client who has a new diagnosis of type 1 diabetes mellitus. Which of the following activities is the nurse’s priority

A. Instruct the client about the importance of regular medical appointments
B. Encourage the client to participate in daily exercise
C. Explain proper foot care techniques to the client
D. Ensure that the client understands the medication regimen
D

Key to preventing hyper/hypoglycemic episodes!

A nurse is caring for a client who is receiving positive end expiratory pressure (PEEP) via mechanical ventilation. The nurse should monitor the client for which of the following adverse effects of PEEP?

A. Hypoxemia
B. Tension pneumothorax
C. Malignant hypertension
D. Atelectasis
B
PEEP can be associated with a high tidal volume, which could lead to an alveolar rupture and pneumothorax

A, PEEP improves oxygenation
C, PEEP can cause hypotension
D, PEEP keeps the lungs inflated at the end of expiration to prevent lung from collapsing (atelectasis)

During a change-of-shift report, a night shift nurse informs the day shift nurse that a newly admitted client was disoriented and combative during the night. Which of the following actions should the day shift nurse take?

A. Keep the client’s television on with the volume low
B. Insert an indwelling catheter to minimize interaction with the client
C. Consult the provider regarding administering a mild sedative on schedule
D. Move the client to a room near the nurses’ station
D

A nurse is providing teaching to a school-age child who has asthma about using an albuterol metered-dose inhaler. Which of the following instructions should the nurse include?

A. Clean the mouthpiece with warm water every 2 weeks
B. Wait 10 seconds between inhalations
C. Take a quick inhalation when pressing the dispenser
D. Take the medication 15 min before playing sports
D

A nurse is providing client teaching about the basal body temperature method of birth control. Which of the following information should the nurse include in the teaching?

A. “your body temperature will drop approximately 1 degree 1 week after ovulation”
B. “you should take your body temperature each evening prior to going to sleep”
C. “your body temperature might decrease slightly just prior to ovulation”
D. “your body temperature is at its highest during mensturation”
C

A nurse is preparing to administer insulin to a client via a pen device. Which of the following actions should the nurse take?

A. Hold the insulin pen device perpendicular to the client’s skin to inject the medication
B. Shake the insulin pen device prior to injecting the medication
C. Withdraw the insulin from the pen device into an insulin syringe
D. Hold the pen device in place for 3 seconds after injecting the insulin
A

A nurse is caring for a client who is receiving continuous bladder irrigation following a transurethral resection of the prostate. The client reports bladder spasms, and the nurse observes a decreased urinary output. Which of the following actions should the nurse take?

A. Increase tension on the urinary catheter
B. Irrigate the catheter with 0.9% sodium chloride irrigation
C. Assist the client to ambulate
D. Remove the urinary catheter immediately
B

A nurse in a provider’s office is caring for an 18 month old toddler who has a blood lead level of 3 mcg/dL. Which of the following actions should the nurse take?

A. Schedule chelation therapy
B. Contact the poison control center
C. Recommend re-screening in 1 year
D. Refer the family to social services
C

<5 mcg/dL is normal

A nurse preceptor is evaluating the performance of a newly licensed nurse. Which of the following actions by the newly licensed nurse requires intervention by the preceptor

A. Documents client tasks upon completion
B. Starts a task then determines what supplies are needed
C. Completes a client assessment while infusing an IV antibiotic over 30 minutes
D. Returns to the nurses’ station after completing several tasks in the same location
B

A nurse is assessing a client who is taking propranolol. Which of the following findings should indicate to the nurse that this client is experiencing an adverse reaction to propranolol

A. Weight loss
B. Wheezing
C. Blood pressure 146/92
D. Heart rate 110/min
B

A dry cough is an expected finding of a beta blocker, but wheezing would indicate an adverse reaction

C, we expect the BP to decrease
D, we expect the HR to decrease with beta blockers

A nurse is assessing a client who has pulmonary edema. Which of the following findings should the nurse expect?

A. Pink, frothy sputum
B. Bradycardia
C. Flushed, dry skin
D. Wheezing
A

A, this is a common finding
B, most patients have a rapid, irregular heart rate/ palpitations
C, most patients with have cold clammy skin
D, patients have shortness of breath especially when laying down

A nurse is assessing a client who received 2 units of packed RBCs 48 hr ago. Which of the following findings should indicate to the nurse that the therapy has been effective?

A. Hemoglobin 14.9 g/dL
B. WBC count 12,000
C. Potassium 4.8
D. BUN 18
A

PRBC will increase the hemoglobin (normal hemoglobin for women is 12-16 and males is 14-18, so 14.9 indicates the therapy successfully got the patient within range)

A nurse has received change-of-shift report on four assigned clients. For which of the following clients should the nurse intervene to prevent a potential food and medication interaction?

A. A client who is receiving verapamil and has a continuous infusion of parenteral nutrition (TPN)
B. A client who is taking phenytoin and is requesting a milkshake
C. A client who is receiving a diet high in potassium rich foods and furosemide by mouth
D. A client who is receiving an MAOI and is requesting a cheeseburger for dinner
D

Patients on MAOI need to avoid tyramine: beefs, liver, fermented sausage (pepperoni, salami), bacon, hot dogs, lunch meat. Eating these foods can cause a hypertensive crisis!

B, phenytoin can be taken with milk products
C, furosemide is potassium wasting (loop diuretic) so we do want to replace K in the diet

A nurse is caring for a client who had a stroke 6 hr ago. Which of the following interventions should the nurse implement to reduce the risk of increased intracranial pressure (ICP)?

A. Flex the clients neck forward
B. Group several nursing activities to be completed at once
C. Limit suctioning the client’s airway to 30 seconds at a time
D. Place the client in a quiet environment
D

A, keep the head in a neutral position
B, do not over stimulate the patient at one time
C, limit suctioning to only 10 seconds if you absolutely have to suction (limit suctioning as much as possible)

A nurse is planning care for a client who is receiving chemotherapy and has neutropenia. Which of the following interventions should the nurse include in the plan?

A. Avoid including raw fruits in the client’s diet
B. Restrict visits form young children to 2 hr per day
C. Measure the client’s temperature once per shift
D. Use disposable gloves from a box outside the clients room
A

A school nurse is notified of an emergency in which several children were injured following the collapse of playground equipment. Upon arrival at the playground, which of the following actions should the nurse take first?

A. Instruct a staff member to maintain a log of emergency care provided
B. Apply cervical spine collars to children who have suspected neck trauma
C. Notify guardians of the emergency and injuries to their children
D. Survey the scene for potential hazards to staff and children
D

A nurse is caring for a school aged child who is taking valproic acid. The nurse should expect the provider to order which of the following diagnostic tests

A. Chest x ray
B. Serum liver enzyme levels
C. ABG’s
D. Urine culture and sensitivity
B

Med can cause hepatotoxicity

A nurse is preparing to transfer a client who has had a stroke to a rehabilitation facility. The client’s family tells the nurse they are concerned about the level of care the client will receive. Which of the following actions should the nurse take?

A. Facilitate an interdisciplinary conference at the new facility for the family
B. Refer the client and family to a social worker for assistance and a follow-up meeting
C. Reassure the client’s family that the same provider will provide care at the new facility
D. Tell the family that the rehabilitation facility has an excellent client care record
A

A charge nurse is speaking with the partner of a client. The partner states that the client is not receiving adequate care. Which of the following actions should the charge nurse take first to resolve the situation?

A. Evaluate the changes the partner requests
B. Review the client’s plan of care
C. Analyze other reports of poor care to look for trends
D. Ask the partner to list specific concerns
D

A nurse is caring for a client who has a prescription for a continuous passive motion (CPM) machine following a total knee arthroplasty. Which of the following actions should the nurse take?

A. Turn off the CPM machine during mealtime
B. Maintain the client’s affected hip in an externally rotated position
C. Instruct the client how to adjust the CPM settings for comfort
D. Store the CPM machine under the client’s bed when not in use
A
Allow for adequate rest so that the patient can fully eat

A nurse is proving discharge instructions about newborn care to a client who is postpartum. Which of the following statements indicates to the nurse that the client understands the teaching? (select all that apply)

A. “I will breastfeed my baby on a schedule of every 4 hours”
B. “I will bathe my baby daily”
C. “I will place my baby on her stomach for sleeping”
D. “I will cover my baby’s body when I wash her hair”
E. “I will use my bulb syringe first in her mouth and then in her nose”
D, E

A nurse has just received change-of-shift report on four clients. Which of the following clients should the nurse assess first?

A. A client who is postoperative with abdominal distention and no bowel sounds
B. A client who has diabetes mellitus and a blood glucose of 105 mg/dL
C. A client who has heart failure and 2+ pitting edema
D. A client who is receiving maintenance IV fluids and needs a new IV catheter
A
This is an adverse effect/ abnormal findings that is threatening, they are the priority

B, 105 is within normal range
C, pitting edema is an expected finding in heart failure d/t fluid retention. Will want to see this patient soon, but not the priority
D, maintenance IV fluids is not a priority

A nurse is preparing to replace a client’s transdermal fentanyl patch after 72 hr of use. After the nurse opens the packet containing the new pouch, the client declines to accept it. Which of the following actions should the nurse take?

A. Withhold pain medications for 24 hr after the old patch is removed
B. Ask another nurse to witness the disposal of the new patch
C. Seal the patches in a plastic bag and place in the client’s trash basket
D. Stick the two patches to each other and place them in the sharps bin
B

A nurse is teaching a client who has a new prescription for total parenteral nutrition through a central line. Which of the following information should the nurse include in the teaching?

A. “I will change your IV tubing once every 48 hr”
B. “Abdominal distention is an expected effect of this therapy”
C. “I will need to check your gastric residual before administering feedings”
D. “I will need to measure your weight daily”
D
This is the best way to measure fluid volume

B, this would be an adverse finding
C, gastric residuals are checked for NG tubes, not central lines

A nurse is performing an admission assessment on a client who had a recent positive pregnancy test. The first day of her last menstrual period (LMP) was May 8. According to Nagele’s rule, which of the following dates should the nurse document as the client’s estimated date of birth (EDB)?

A. February 1
B. February 8
C. February 15
D. February 22
C

Calculates the due date for a pregnancy when assuming a gestational age of 280 days at childbirth.

(EDD)= adding a year, subtracting three months, and adding seven days to the origin of gestational age

A nurse on a medical surgical unit is assessing a client who has had a stroke. For which of the following findings should the nurse initiate a referral for occupational therapy?

A. Difficulty performing ADLs
B. Inability to swallow clear liquids
C. Elevated blood glucose levels
D. Unsteady gait when ambulatin
A

B, contact speech pathology
C, contact diabetes educator
D, contact physical therapy

A nurse is providing teaching to a client who is scheduled for electroconvulsive therapy (ECT). The nurse should inform the client that which of the following findings is an adverse effect of ECT

A. agitation
B. Short term memory loss
C. Post treatment seizures
D. Incontinence of the bowel and bladder
B

An antepartum nurse is caring for four clients. For which of the following clients should the nurse initiate seizure precautions?

A. A client who is at 33 weeks of gestation and has severe gestational hypertension
B. A client who is at 16 weeks of gestation and has a hydatidiform mole
C. A client who is at 26 weeks of gestation and is experiencing vaginal bleeding
D. A client who is at 36 weeks of gestation and has a positive group B streptococcal culture
A

hypertension is a major indicator of seizures!

B, A noncancerous tumor that develops in the uterus as a result of a nonviable pregnancy. Can cause dark bleeding, nausea and vomiting
C, this is a concern, but not a risk for seizures
D, not a risk for seizures

a nurse is planning care for a client who is receiving heparin to treat a deep vein thrombosis of the left lower leg. which of the following interventions should the nurse include in the plan of care?

elevate the affected leg

ambulation should be encouraged

a nurse is teaching a group of guardians about child safety measures. which of the following statements by a guardian indicates an understanding of the teaching?

I should have my child avoid sun exposure between 10 am and 2 pm

a home health nurse is providing teaching about infection prevention to a client who has cancer and is receiving chemotherapy. Which of the following statements by the client indicates an understanding of the teaching?

I will walk for short distances throughout the day

a nurse is providing dietary teaching to the parents of a 6 month old infant. which of the following instructions should the nurse include?

Introduce new foods one at a time over 5 to 7 days

a charge nurse is planning care for a client who has mechanical restraints in place. which of the following interventions should the nurse include in the plan?

Provide a staff member to stay with the client continuously

can be physically or using an audiovisual device

need to document every 15-30 minutes, a new prescription should be in every 4 hrs for adults, every 3 hrs for 9-17 years and every hour for those younger

a community health nurse is assisting with the development of a disaster management plan. the nurse should include which of the following nursing responsibilities in the disaster response stage of the plan?

performing a rapid needs assessment

a nurse manager is assisting with the orientation of a newly licensed nurse. which of the following actions by the nurse requires the nurse manager to intervene?

tells the hospital chaplain a clients diagnosis

a nurse is caring for a client who is in labor at 39 weeks of gestation. during the second stage of labor, the nurse observes early deceleration on the monitor tracing. which of the following actions should the nurse take?

continue observing the fetal heart rate

these are an expected finding

a nurse manager is planning to make changes to the current scheduling system on the unit. to facilitate the staff’s acceptance of this change, which of the following actions should the nurse manager take first?

provide information about scheduling issues to the staff

unfreezing stage – increase understanding of why change is needed

a nurse is planning teaching about allowable foods for a client who has a history of uric acid based urinary calculi formation. which of the following foods should the nurse include in the teaching?

oranges

can eat citrus fruits

no chicken, no organ meats like liver, no red wine (avoiding purine)

a charge nurse is teaching a newly licensed nurse how to identify true labor. which of the following should the nurse include in the teaching?

the cervix transitions to an anterior position

contractions will be felt in the lower abdomen and back, intensity increases with ambulation, cervix shortens and thins

a nurse on a medical surgical unit is caring for a client prior to a surgical procedure. which of the following findings should indicate to the nurse that the client has the ability to sign the informed consent?

the client is able to accurately describe the upcoming procedure

a charge nurse assigns a newly licensed nurse to care for a client who has a chest tube. The nurse expresses concern about having limited experience with monitoring chest tube drainage. which of the following actions should the charge nurse take first to provide teaching about chest tubes?

ask the nurse about their knowledge of the procedure

a nurse is teaching a client who has a new prescription for estradiol. for which of the following adverse effects of this medication should the nurse instruct the client to monitor and report to the provider?

headaches – can be indicative of a thromboembolic stroke

a nurse is preparing to administer diazepam 0.3mg/kg IV bolus to a toddler who weighs 22 lb and is experiencing a grand maul seizure. availabale is diazepam solution for injection 5mg/mL. how many mL should the nurse administer? 0

0.6 mL

a nurse is caring for a client who is 12 hours postoperative, is receiving PCA for pain control, and requires a blood pressure check in 10 min. which of the following staff members should the nurse assign to collect the information?

An assistive personnel (AP) who is assisting a client to return to bed

a nurse is creating a plan of care for a child who has acute lymphoid leukemia and an absolute neutrophil count of 400/mm^3. which of the following interventions should the nurse include in the plan?

withhold administering the varicella vaccine to the child – it is a live vaccine and should not be given

a nurse is providing teaching about improving nutrition for a client who has multiple sclerosis. which of the following instructions should the nurse include? select all

A speech pathologist will be performing a swallowing study for you
you should rest before eating meal
thicken your beverage before drinking

a nurse is assessing a client who has schizophrenia and is taking chlorpromazine. which of the following findings is the priority for the nurse to report to the provider?

temperature 39.4 (102.9)

a client is receiving lorazepam IV for panic attacks and develops a respiratory rate of 6/min and a blood pressure of 90/44 mm Hg. Which of the following medications should the nurse anticipate administering?

flumazenil

a nurse is caring for an older adult client in the PACU following general anesthesia. which of the following findings should the nurse report to the provider?

audible stridor

a nurse is preparing a sterile field in order to insert an indwelling urinary catheter for a male client. which of the following techniques should the nurse use to maintain surgical aseptic technique?

set the catheter tray on the overbed table at waist height

a nurse is planning care for a client who has a deficit with cranial nerve II. which of the following actions should the nurse plan to take?

clear objects from the clients walking area

a nurse is providing discharge instructions to a client following a total hip arthroplasty. which of the following instructions should the nurse include?

install a raised toilet seat at home

a nurse is assessing a client who is receiving a blood transfusion. which of the following findings should indicate to the nurse that the client is having a hemolytic transfusion reaction?

low back pain

other signs include hypotension, tachycardia

a nurse is caring for a child who is experiencing a tonic clinic seizure. which of the following actions should the nurse take?

place the child in a side lying position

a nurse is administering medications to a client who has a percutaneous gastrostomy tube for enteral feedings. which of the following actions should the nurse take to prevent clogging of the tube?

flush the client’s gastrostomy tube with 30 mL of water before administering the medication

a nurse is assessing a client who has macular degeneration. which of the following findings should the nurse expect?

decreased central vision

a nurse is assessing a client who has schizophrenia. the nurse should identify the following alteration in speech as which of the following?

clang association – uses words based on their sound rather than their meaning (this one had a lot of b words)

a nurse in a clinic receives a call form a guardian whose child has varicella. the guardian asks when the child can return to school. which of the following responses should the nurse make?

when the crusts have formed on every lesion

a nurse is providing discharge instructions to a client who has a new prescription for warfarin. which of the following client statements should the nurse identify as an indication that the client understands the teaching?

I should report a change in the color of my stools – red, black, tarry, etc.

a nurse is assessing a school age child who has cystic fibrosis. which of the following findings is the priority for the nurse to report to the provider?

hemoptysis 275 mL/24 hr

when caring for a child, a nurse plans to use nonpharmacological interventions to enhance the effectiveness of pain medication. which of the following strategies incorporates visualization techniques to help decrease the child’s discomfort?

blowing bubbles with liquid soap to blow the hurt away

an assistive personnel and a nurse are turning a client onto the right side. which of the following actions by the AP requires the nurse to intervene?

Places a pillow under the client’s right arm

the AP should place a pillow under the client’s left arm to prevent internal rotation of the left shoulder

a nurse on a mental health unit is caring for a client who tells the nurse that she does not want to receive a scheduled dose of lorazepam IM. which of the following actions should the nurse take?

document the client’s refusal of the medication

a nurse is caring for four clients., which of the following clients should the nurse assign to an assistive personnel to assist with meals

a client who has Alzheimer’s disease and is demonstrating aphasia

a mental health nurse is conducting the first of several meetings with a client whose partner recently died. the nurse should perform which of the following actions to establish trust during the orientation phase of the nurse client relationship

establish the termination date of therapy

a nurse is caring for a client who is at 28 weeks of gestation. the client asks the nurse to explain what causes her to have constipation. which of the following responses should the nurse make?

the enlarged uterus compresses the intestines and causes constipation

a nurse manager is reviewing unit records and discovers that client falls occur most frequently during the hours of 0540 and 0730. Which of the following actions should the nurse take when conducting a root cause analysis

investigate environmental factors that might be contributing to client injury during these hours

a nurse is caring for a client who is immediately postoperative following a total vaginal hysterectomy. which of the following actions should the nurse take first?

measure the clients vital signs (every 15 minutes until stable)

a charge nurse is planning an educational session for staff nurses about working with parents whose terminally ill children are candidates for donating their organs. which of the following information should the nurse plan to include?

the family can have the child in an open casket without fearing that the organ donation might disfigure the child’s body

a nurse is teaching about total parenteral nutrition TPN and IV lipid emulsions with a client who has an extensive burn injury. Which of the following information should the nurse include?

you will receive fingersticks for blood glucose testing

a nurse is caring for a client who has a fractured femur and has had a fiberglass leg cylinder cast for 24 hours. which of the following assessment findings should the nurse identify as the priority?

the client’s heel is reddened and tender (possible pressure injury)

a home health nurse is providing teaching to a client who has hepatitis A. which of the following instructions should the nurse include?

use hydrogen peroxide to clean kitchen surfaces

a nurse is preparing to perform an intermittent urinary catheterization for a client who has unrainy retention. which of the following images indicates the catheter the nurse should use?

It’s a picture of a catheter that is pink in color with no bulbs or anything just one single tube

a community health nurse is preparing a health education program for a local rural community. which of the following actions should the nurse plan to take first?

identify health related issues within the community

a nurse is caring for a client who has generalized anxiety disorder and is to begin taking alprazolam. which of the following actions should the nurse take?

initiate fall precautions for the client (common adverse effects are orthostatic hypotension, dizziness, confusion, and lethargy)

a nurse in the delivery room is caring for a newborn immediately after birth. which of the following actions should the nurse take first?

dry the newborn

a nurse in an acute mental health facility is planning care for a client who has anorexia nervosa. which of the following interventions should the nurse include in the clients plan of care?

supervise the client during and after eating

everything to do with food should be structured, the client has minimal choices

a nurse is talking with the partner of a client who attempted suicide. which of the following statements by the client’s partner should the nurse identify as the priority?

“my husband doesn’t know that I’ve already moved out of the house and filed for a divorce” (lack of social and emotional support)

a nurse is assessing a client who has multiple sclerosis. which of the following manifestations should the nurse expect?

nystagmus (involuntary eye movement)

abdominal striae is found in cushings disease

masklike face is found in parkinsons disease

ptosis is found in myasthenia gravis

a nurse is interviewing a client who is now without a home due to a natural disaster. after ensuring the client’s safety, which of the following actions should the nurse take first

determine the client’s perception of the personal impact of the crisis

a nurse is preparing to administer enoxaparin to a client. Identify the area the nurse should use to administer the injection.

Subcutaneous tissue in the periumbilical area

a nurse is assessing a client who has skeletal traction for a femur fracture. which of the following findings should the nurse identify as the priority?

upper chest petechiae (fat embolism syndrome)

a nurse is assessing a client who has sickle cell anemia. the nurse should identify which of the following findings as a manifestation of vaso-occlusive crisis?

hematuria

also painful swelling of the hand and feet and visual disturbances

a nurse is assessing a client who has been taking lithium carbonate for the past month to treat bipolar disorder. which of the following assessment findings should the nurse identify as the priority?

confusion (lithium toxicity)

hand tremors are an adverse effect of lithium

a nurse is providing teaching to a parent of a child who has a permanent tracheostomy tube. Identify the sequence of steps the parent should follow to perform tracheostomy care.

Remove the inner cannula
remove the soiled dressing
clean the stoma with 0.9% sodium chloride irrigation
change the tracheostomy collar

a nurse in an emergency department is assessing a client who reports taking methylenedioxymethamphetamin MDMA. which of the following findings should the nurse expect

diaphoresis

increases tactile sensitivity, lowered inhibition, chills, muscle cramping, teeth clenching, and mild hallucinogenic effects

a nurse is teaching a client who is to start taking misoprostol and currently is on long term therapy with NSAIDs for arthritis. the nurse should provide the client with which of the following information?

complete a serum pregnancy test before taking the medication (can induce uterine contractions)

a nurse is caring for a client who has a potassium level of 3 mEq/L. For which of the following manifestations should the nurse monitor?

decreased deep tendon reflexes

muscle weakness as well

a nurse in an emergency department is preparing to discharge a client who has experienced intimate partner violence. which of the following actions should the nurse take first?

develop a safety plan with the client

a nurse is caring for a client who is at 37 weeks of gestation and is experiencing abruptio placentae. which of the following findings should the nurse expect?

persistent uterine contractions

board like abdomen, and dark red vaginal bleeding

a nurse is caring for a client who is in the manic phase of bipolar disorder. which of the following manifestations should the nurse expect?

grandiose delusions

a nurse is caring for a client who states, “my boss accused me of stealing yesterday. I was so angry I went to the gym and worked out” the nurse should recognize the client is demonstrating which of the following defense mechanisms?

sublimation

a nurse is teaching a client about foods high in vitamin A. which of the following foods should the nurse recommend as having the highest amount of vitamin A?

1 medium raw carrot

a nurse is teaching the parent of a school aged child about administering ear drops. which of the following responses by the parent indicates an understanding of the teaching?

“I should pull the top of the ear upward and back while instilling the medication”

a nurse is caring for a client who requires physical therapy following discharge. which of the following actions should the nurse take?

Involve the client in selection of a physical therapy provider

a nurse is planning care for a client who is receiving hemodialysis via an established arteriovenous (AV) fistula in the right arm. which of the following interventions should the nurse include in the client’s plan of care?

auscultate the affected extremity for a bruit

a nurse is providing teaching to a client about newborn safety. which of the following statements should the nurse include in the teaching?

“set your hot water heater temperature at or below 120 degrees Fahrenheit”

a nurse is caring for a newborn who has herpes simplex virus HSV. which of the following isolation precautions should the nurse initiate?

contact

a home health nurse is planning care for an older adult client who has impaired vision. which of the following interventions should the nurse include in the plan of care?

mark the edges of stairs for contrast

should have vision testing done every year

a nurse is initiating discharge planning for a client who had a stroke and is experiencing right sided weakness. which of the following actions should the nurse take first?

request a referral for the client to receive physical therapy

a nurse is reviewing the laboratory findings of a client who is experiencing chest pain. The nurse should identify that an elevation in which of the following laboratory values indicates cellular injury of myocardial tissue?

troponin T

a nurse is preparing to administer heparin 5,000 units subcutaneously, available is heparin injection 10,000 units/mL. how many mL should the nurse administer per dose? Round to the nearest tenth.

0.5mL

a nurse in an outpatient mental health clinic is working with a client who has post traumatic stress disorder PTSD and asks the nurse to recommend a nonpharmacological therapy to use to provide relief of the manifestations. which of the following complementary therapies should the nurse teach the client to use to help alleviate the distress?

guided imagery

a nurse manager is preparing a newly licensed nurse’s performance appraisal. which of the following methods should the nurse manager use to evaluate the nurse’s time management skills?

maintain regular notes about the nurse’s time management skills

a nurse is caring for four clients at the beginning of a shift. after receiving change of shift report, which of the following clients should the nurse attend to first?

a client who is confused and has been attempting to get out of bed (risk for injury)

a night shift nurse is giving change of shift report to the day sift nurse on a client who is ready for discharge. which of the following information is the priority for the nurse to communicate to the oncoming nurse?

the client needs assistance when transferring from the bed to a wheelchair

a nurse is assessing a client who has a stage II pressure injury. Which of the following wound characteristics should the nurse expect?

partial thickness skin loss

a nurse is providing discharge instructions to a client who has a new prescription for amitriptyline to treat depression. the nurse should identify that which of the following client statements indicates an understanding of the teaching?

“I should watch for common reactions like dry mouth and constipation”

a charge nurse is preparing to administer 0900 medications and is told by the pharmacy staff that the medications are not available. medication availability has been an ongoing problem, and the charge nurse has previously discussed this issue with the pharmacy staff. which of the following actions should the charge nurse take first?

inform the nurse manager of the issue

a nurse is caring for a client who vomits on a reusable BP cuff. which of the following actions should the nurse take?

place the BP cuff in a labeled bag to send it for decontamination

chlorine bleach is used for blood

a nurse is assessing a newborn who is 2 hours old. which of the following findings should the nurse report to the provider?

axillary temperature of 36.2 (97.2)

should be 36.5-37.5

a nurse manager is on a planning committee to develop an emergency preparedness plan. The nurse should recommend that which of the following actions take place first when implementing an emergency preparedness plan?

notify the incident commander

a nurse is preparing to administer 15 units of regular insulin along with 20 units of NPH insulin. which of the following actions should the nurse plan to take?

inject 20 units of air into the NPH insulin vial

draw the short acting regular insulin into the vial first, which prevents the longer acting insulin from contaminating the short acting insulin

a charge nurse overhears two staff nurses in the hallway discussing the nutritional status of a client who has anorexia nervosa. which of the following actions should the charge nurse take?

tell the nurses to stop the discussion

a nurse is assessing a client who is 2 hours postoperative following cardiac catheterization. which of the following information should the nurse report to the provider?

neurologic status – responds to verbal stimuli, speech slurred, hand grasps weak and unequal

a nurse is providing client education to a postpartum client who has decided to bottle feed the newborn. which of the following instructions should the nurse include in the teaching to help prevent the discomfort of engorgement?

place ice packs on the breasts for 15 min several times per day

a case manager is reviewing the medical records of several clients. for which of the following clients should the nurse request an interprofessional care conference?

A client who has diabetes mellitus and has had repeated hospitalizations for diabetic ketoacidosis

a charge nurse is observing a newly licensed nurse administer enteral feedings via NG tube. Which of the following actions by the newly licensed nurse indicates an understanding of the procedure?

keeps the head of the bed elevated to 45 degrees for 1 hour after feedings

a nurse is assessing a client during the immediate postpartum period. which of the following findings requires immediate intervention by the nurse?

boggy uterus

a nurse is caring for a newborn whose parent asks why the baby is receiving vitamin K. the nurse should explain to the parent that the newborn should receive vitamin K to prevent which of the following?

bleeding

a nurse is planning care for a client who has thrombocytopenia. which of the following instructions should the nurse include in the client’s plan of care?

avoid venipunctures when possible

a nurse is assessing a 2 month old infant during a well baby examination. which of the following actions should the nurse take to assess the infants rooting reflex?

stroke the infants cheek

a nurse is teaching a client who has a new prescription for digoxin about manifestations of toxicity. which of the following findings should the nurse include in the teaching?

nausea

anorexia, abdominal pain, bradycardia, visual changes

a nurse is reviewing the urinalysis report of a client who has acute glomerulonephritis. which of the following findings should the nurse expect?

protein

a nurse working on a medical surgical unit receives a telephone call requesting the status of a client from an individual who identifies themselves as the client’s parent. which of the following actions should the nurse take?

ask the caller for verification of their identity

a nurse is caring for a client who has end stage alzheimer’s disease. the adult child of the client says to the nurse “I don’t know why I bother to visit my mother anymore” which of the following responses should the nurse make?

it seems like you feel your visits are a waste of time

a nurse is assessing a client who has antisocial personality disorder. which of the following manifestations should the nurse expect?

lack of remorse

a community health nurse is performing triage tagging following a mass casualty incident. on which of the following clients should the nurse place a black tag?

a client who has significant head trauma and agonal respirations

a nurse is caring for a client who has had nausea and vomiting for the past 2 days. The nurse should identify which of the following findings as an indication the client is experiencing fluid volume deficit?

orthostatic hypotension

increased BUN

A nurse is providing discharge teaching to a new parent about car seat safety. which of the following statements should the nurse include in the teaching?

secure the retainer clip at the level of your baby’s armpits

a nurse is caring for a toddler who has infectious gastroenteritis. which of the following actions should the nurse take?

initiate oral rehydration therapy for the toddler

chx and beef broth contain a lot of sodium and very few carbs so should not be used

a nurse is preparing to insert an indwelling urinary catheter for a client. The nurse should assess the client for which of the following conditions prior to starting the procedure?

latex allergy

a nurse is caring for an older adult client. which of the following findings should the nurse recognize as a physical change associated with aging?

decreased lung expansion

a charge nurse observes a staff nurse document a dressing change in a client’s chart that was not performed. which of the following actions should the charge nurse take first?

gather more information about the staff nurse’s actions

a nurse is providing discharge teaching to a client following a cataract extraction. which of the following statements by the client indicates an understanding of the teaching?

“I will bend at the knees when picking an object up off the floor”

a nurse is caring for a client who has methicillin-resistant staphylococcus aureus MRSA in an abdominal wound. which of the following precautions should the nurse implement?

contact

a nurse is caring for a client who is 4 hr postpartum and has a boggy uterus with heavy lochia. which of the following actions should the nurse take first?

massage the uterus to expel clots

a home health nurse is assessing a 2 week old newborn who has a birth weight of 3.64 kg (8lb) and is being breastfed. which of the following findings indicates effective breastfeeding.

the newborn has six to eight wet diapers per day

a nurse is caring for a client who has a closed head injury and is receiving mechanical ventilation. the nurse should expect to administer which of the following medications to reduce intracranial pressure?

mannitol

it is an osmotic diuretic to reduce intracranial pressure

a nurse is teaching a newly admitted client who has heart failure about advance directives. which of the following statements should the nurse make?

you should complete advance directives in the event you cannot express your own wishes

a nurse is teaching the parents of a preschooler about sleep promotion. the parents report that their child is demonstrating reluctance in going to bed at night and states, “I am not tired”. Which of the following statements by the parents indicates an understanding of the teaching?

we should read a story together every night before bedtime

an RN is planning care for a group of clients and is working with a licenced practical nurse LPN and an assistive personnel AP. which of the following tasks should the RN delegate to the LPN

insertion of a nasogastric tube

a nurse in an emergency department is admitting a client who has cardiac tamponade. which of the following assessment findings should the nurse expect?

pulsus paradoxus

a nurse is assessing a client who has delirium. which of the following manifestations should the nurse expect?

rapid speech

a nurse working in a long term care facility is assessing an adult client. which of the following findings places the client at risk for development of a pressure injury

recent weight loss(inadequate nutrition)

a nurse is caring for a client who had a recent stroke. prior to transferring the client to the bedside commode, which of the following actions should the nurse take first?

assess the client for functional limitations

a community health nurse is reviewing the medical records of four newly diagnosed clients. The nurse should identify which of the following clients as having a nationally notifiable infectious condition?

an adolescent client who has foodborne botulism

a nurse is performing tracheostomy care for a client who is postoperative following a laryngectomy. which of the following actions should the nurse take when suctioning the clients airway?

apply suction for 10 seconds

a nurse is caring for a client who has an STI that must be reported to the state health department. which of the following actions should the nurse take?

explain to the client why this information will be shared

a nurse is caring for an adolescent client who has a diagnosis of terminal cancer. when discussing the client’s prognosis with the parents, the nurse should recognize which of the following responses by the parents as an example of rationalization?

“Maybe this is better for our child because we don’t want any suffering through chemotherapy treatments”

a nurse is assessing a client who has obstructive sleep apnea. for which of the following complications should the nurse monitor?

hypertension

a nurse is conducting visual acuity testing using the snellen letter chart for a school age child who has eyeglasses. which of the following instructions should the nurse give to the child?

you should keep both eyes open during the testing

a nurse is preparing to administer a long acting insulin to a client who has diabetes mellitus. which of the following actions should the nurse plan to take first?

check the insulin dose with another licensed nurse

a clinic nurse is caring for a client who is in the first trimester of pregnancy. the client reports using acupressure bands on both wrists. which of the following statements by the client indicates that this therapy is having the desired effect?

“I have not vomited as much recently”

a nurse is caring for a client who has hypertension and is taking captopril. which of the following tasks should the nurse delegate to an assistive personnel?

obtain the client’s blood pressure before the nurse administers medication

a nurse is caring for a client who is receiving total parental nutrition TPN solution by continuous IV infusion at 60 mL/hr. The nurse discovers the infusion pump has stopped working. Which of the following actions should the nurse take while waiting for a new infusion pump?

provide dextrose 10% in water solution using manual drip tubing at 60 mL/hr

a nurse is performing gastric lavage for a client who has gastrointestinal bleeding and an NG tube in place. which of the following actions should the nurse take?

use 0.9% sodium chloride for irrigation of the NG tube

a nurse is reviewing the medical record of a client who has schizophrenia and is to start taking clozapine. which of the following findings should the nurse identify as a contraindication for the client to receive clozapine?

WBC count 2,800/mm^3

a nurse is performing an admission assessment of a preschooler who is in the acute phase of Kawasaki disease. Which of the following findings should the nurse expect?

fever unresponsive to antipyretics

a nurse is caring for a group of clients. for which of the following events should the nurse complete an incident report?

A client’s IV pump delivers an inadequate dose of medication

a nurse is caring for a client who has bipolar disorder. the nurse observes that the client is becoming increasingly restless. the client is pacing the unit and speaking rapidly, frequently using profanities and sexual references. which of the following actions should the nurse take first?

move the client to a quiet place away from others

a nurse is assessing a preschooler who has cystic fibrosis and has been receiving oxygen therapy for the past 36 hr. which of the following findings should the nurse identify is an indication that the client has developed oxygen toxicity?

substernal pain

increased work of breathing

a charge nurse is observing a newly licensed nurse performing a physical assessment on a client. which of the following actions by the nurse indicates that the charge nurse should intervene?

the newly licensed nurse writes detailed notes while performing the head to toe assessment

a nurse is providing discharge teaching to a client who has colorectal cancer and a new colostomy. the client states, “I’m worried about being discharged because I live alone, and my insurance doesn’t cover ostomy supplies” which of the following actions should the nurse take? (select all that apply)

refer the client to a community based social worker
initiate a consult with a home health care provider
give the client information about local support groups

a nurse is administering cyclophosphamide orally to a school age child who has neuroblastoma. which of the following actions should the nurse take when administering this medication?

maintain hydration with liberal fluid intake

a nurse is teaching home wound care to the family of a child who has a large wound. which of the following interventions should the nurse recommend?

double bad soiled dressings in plastic bags for disposal

a client who is 24 hours postoperative following abdominal surgery refuses to ambulate. which of the following actions should the nurse take first?

ask the client to rate their pain level

a nurse is providing colostomy care for a client using a two piece pouching system. which of the following actions should the nurse take?

place the skin barrier over the stoma and hold it for 30 seconds

-opening should be no more than 0.3cm larger than the stoma

a nurse is preparing to administer 2 units of fresh frozen plasma to a client. which of the following actions should the nurse plan to take?

enter the plasma product number into the clients medical record

infuse over 30-60 min immediately after getting it from the blood bank

a nurse is assessing a client who is experiencing autonomic dysreflexia. which of the following findings should the nurse expect?

facial flushing
nasal congestion
headache

a nurse is teaching a client who has opioid use disorder about methadone. which of the following information should the nurse include in the teaching?

“sedation is a common adverse effect of this medication”

a nurse is providing teaching to an adolescent following insertion of a tunneled central venous catheter without a pressure sensitive valve. which of the following information should the nurse include in the teaching?

“you should keep the catheter clamped when not in use”

flush with heparin
change dressing every 5-7 days

a nurse is reviewing the ABG results of a client who has COPD. The results include a pH of 7.3, PaO2 56 mm Hg, PaCO2 54 mm Hg. HCO3 26 mEq/L, SaO2 87%. Which of the following is the correct interpretation of these values

uncompensated respiratory acidosis

a nurse is assessing a client who has Raynaud’s disease. Which of the following findings should the nurse expect?

Blanching of the fingers and toes

a nurse is caring for a client who has a terminal illness and requests no lifesaving measures if a cardiac arrest occurs. which of the following statements should the nurse make?

“I will provide you with information about medical treatment to include in your living will”

A nurse is teaching the parents of a toddler about snacks. Which of the following foods should the nurse recommend?

diced steamed carrots

a nurse is caring for a toddler who is admitted to the pediatric unit for surgery. which of the following should the nurse include in the toddler’s plan of care?

encourage the parents to bring toys from home

a nurse receives a request from a client to review the information in his medical record. which of the following responses should the nurse give?

“There’s a protocol for reviewing your medical record and I can initiate the process”

A nurse is caring for a client who states, “My boss accused me of stealing yesterday. I was so angry I went to the gym and worked out.” The nurse should recognize the client is demonstrating which of the following defense mechanisms?

Sublimation

Rationale: The client is exhibiting behaviors consistent with sublimation, which is displayed when a client substitutes socially unacceptable behavior for acceptable behavior.

A nurse is caring for a client who has generalized anxiety disorder and is to begin taking alprazolam. Which of the following actions should the nurse take?

Initiate fall precautions for the client

Rationale: The nurse should initiate fall precautions for a client who has a new prescription for alprazolam because common adverse effects associated with this medication are orthostatic hypotension, dizziness, confusion, and lethargy.

A nurse on a med surg unit is caring for a client prior to a surgical procedure. Which of the following findings should indicate to the nurse that the client has the ability to sign the informed consent?

The client is able to accurately describe the upcoming procedure

Rationale: The ability of the client to accurately describe the upcoming procedure indicates that the provider adequately informed the client and that the client is able to sign the informed consent

An assistive personnel (AP) and a nurse are turning a client onto the right side. Which of the following actions by the AP requires the nurse to intervene?

Places a pillow under the client’s right arm.

Rationale: The AP should place a pillow under the client’s left arm to prevent internal rotation of the left shoulder.

A nurse is providing dietary teaching to the parents of a 6-month-old infant. Which of the following instructions should the nurse include?

Introduce new foods one at a time over 5 to 7 days.

A nurse is caring for a client who has MRSA in an abdominal wound. Which of the following precautions should the nurse implement?

Contact

Rationale: The nurse should implement contact precautions for a client who has an infection spread by direct contact, such as MRSA.

A nurse is caring for a client who is 4 hr postpartum and has a boggy uterus with heavy lochia. Which of the following actions should the nurse take first

Massage the uterus to expel clots

Rationale: Using the EBP approach to client care, the nurse should identify that the priority action is massaging the client’s uterus. Uterine massage will expel clots and increase uterine firmness, resulting in decreased bleeding.

A nurse is providing discharge teaching to a new parent about car seat safety. Which of the following statements should the nurse include in the teaching?

“Secure the retainer clip at the level of your baby’s armpits”

A nurse is providing discharge teaching to a client who has colorectal cancer and a new colostomy. The client states, “I’m worried about being discharged because I live alone, and my insurance doesn’t cover ostomy supplies. “Which of the following actions should the nurse take? (SATA)

-Refer the client to a community based social workers
-Initiate a consult with a home health care provider
-Give the client information about local support groups

Rationale:
-A social worker is necessary to help a client with self-care, as well as assist in locating agencies who can help the client face challenges with self-care and paying for necessary ostomy supplies
-A home health nurse can assist the client in learning to care for the colostomy as well as provide medication management and emotional support
-A client who has cancer and a new colostomy can get help with coping from a support group and possibly receive assistance obtaining supplies from local agencies

A nurse manager is reviewing unit records and discovers that client falls occur most frequently during the hours of 0530 and 0730. Which of the following actions should the nurse take when conducting a root cause analysis?

Investigate environmental factors that might be contributing to client injury during these hours.

Rationale: When conducting a root cause analysis, the nurse should look at the factors that could possibly lead to the clients’ falls. This can include environmental factors that might be causing the problem.

A nurse is caring for a client who has terminal illness and requests lifesaving measures if a cardiac arrest occurs. Which of the following statements should the nurse make?

“I will provide you with information about medical treatment to include in your living will”

Rationale: The nurses’ responsibility is to provide the client with information about specific instructions for addressing medical treatment in a living will. The nurse should assist the client while they are able to make decisions for themself by providing information about what end-of-life preferences to document.

A nurse is assessing a client who has delirium. Which of the following manifestations should the nurse expect?

Rapid speech

Rationale: Clients who have delirium exhibit rapid, inappropriate, incoherent, and rambling speech patterns

A night shift nurse is giving a change of shift report to the day shift nurse on a client who is ready for discharge. Which of the following information is the priority for the nurse to communicate to the oncoming nurse?

The client needs assistance when transferring from the bed to a wheelchair.

Rationale: The greatest risk to this client is injury due to a fall. Therefore, the priority information for the nurse to communicate is that the client requires assistance during transfers.

A nurse is assessing a client during the immediate postpartum period. Which of the following findings requires immediate intervention by the nurse?

Boggy uterus

Rationale: When using urgent vs. nonurgent approach to client care, the nurse should determine that the priority finding is a boggy uterus, which can indicate uterine hemorrhage. The nurse should immediately intervene to stimulate uterine contractions and prevent blood loss. If the uterus becomes relaxed during the postpartum period, the client will rapidly lose blood because no permanent thrombi have formed at the placenta.

A nurse in an emergency department is preparing to discharge a client who has experienced intimate partner violence. Which of the following actions should the nurse take first?

Develop a safety plan with the client

Rationale: The greatest risk to this client is injury from violence. Therefore, the first action the nurse should take is to develop a safety plan with the client.

A client is receiving lorazepam IV for panic attacks and develops a respiratory rate of 6/min and a blood pressure of 90/44 mm Hg. Which of the following medications should the nurse anticipate administering.

Flumazenil

Rationale: The nurse should anticipate administering flumazenil, a competitive benzodiazepine receptor antagonist, to reverse the sedative effects of lorazepam. In addition, the nurse should continue to support the client’s respirations with a bag valve mask.

A home health nurse is planning care for an older adult client who has impaired vision. Which of the following interventions should the nurse include in the plant of care to prevent injury in the home?

Mark the edges of the stairs for contrast

Rationale: Marking the edges of stairs with paint or colored tape for contrast can help older adult clients who have impaired vision prevent injury by decreasing the risk of falls.

A nurse manager is planning to make changes to the current scheduling system on the unit. To facilitate the staff’s acceptance of this change, which of the following actions should the nurse manager take first?

Provide information about scheduling issues to the staff.

Rationale: The first stage of the change process is the unfreezing stage, when the nurse should inform the staff about the current staffing issues. This can increase their understanding of why changes are necessary.

A nurse is teaching a group of guardians about child safety measures. Which of the following statements by guardian indicates an understanding of the teaching?

“I should have my child avoid sun exposure between 10 am and 2 pm”

Rationale: To prevent sunburns, guardians should apply sunscreen, dress their child in protective clothing, and avoid sun exposure between 1000 and 1400.

An RN is planning care for a group of clients and is working with a licensed practical nurse (LPN) and an assistive personnel (AP). Which of the following tasks should the RN delegate to the LPN?

Insertion of a nasogastric tube

Rationale: The nurse should delegate the insertion of a nasogastric tube to the LPN because this task is within the LPN’s scope of practice.

A nurse is assessing a newborn who is 2 hr old. Which of the following findings should the nurse report to the provider?

Axillary temperature 36.2 C (97.2 F)

Rationale: The expected reference range for the axillary temperature of newborn is between 36.5 C to 37.5 C (97.7 F to 99.5 F). An axillary temperature of 36.2 C (97.2 F) or below in a newborn who is 2 hr old indicates cold stress and should be reported to the provider.

A nurse is caring for a newborn whose parent asks why the baby is receiving vitamin K. The nurse should explain to the parent that the newborn should receive vitamin K to prevent which of the following?

Bleeding

The nurse should explain to the parent that newborns are deficient in vitamin K and should receive it following birth because this deficiency can lead to bleeding.

A nurse is caring for a client who requires physical therapy following discharge. Which of the following actions should the nurse take?

Involve the client in selection of a physical therapy provider/

Rationale: The nurse should involve the client in the referral process, including selection of the physical therapist and the location.

A nurse in an emergency department is assessing a client who reports taking MDMA. Which of the following should the nurse expect?

Diaphoresis

Rationale: Diaphoresis is an expected finding of MDMA use. Additionally, the client might experience increased tactile sensitivity, lowered inhibition, chills, muscle cramping, teeth clenching, and mild hallucinogenic effects.

A nurse is caring for a client who vomits on a reusable BP cuff. Which of the following actions should the nurse take?

Place the BP cuff in a labeled bag to send it for decontamination.

Rationale: The nurse should place the BP cuff in a labeled bag before removing it from the client’s room and sending it to the proper facility location for decontamination.

A nurse is reviewing the medical record of a client who has schizophrenia and is to start taking clozapine. Which of the following findings should the nurse identify as a contraindication for the client to receive clozapine?

WBC count 2,800/mm3

Rationale: Clozapine can cause agranulocytosis, which can be life-threatening. Therefore, a WBC count of less than 3,000/mm3 is a contraindication for the client to receive clozapine. The nurse should withhold the medication and notify the provider of the client’s WBC count.

A nurse is providing teaching to an adolescent following insertion of a tunneled central venous catheter without a pressure sensitive valve. Which of the following information should the nurse include in the teaching?

“You should keep the catheter clamped when not in use”

Rationale: The adolescent should keep the catheter clamped to prevent blood backflow. Not all tunneled catheters have a pressure-sensitive valve that prevents blood reflux.

A nurse is conducting visual acuity testing when using the Snellen letter chart for a school age child who has eyeglasses. Which of the following instructions should the nurse give to the child?

“You should keep both eyes open during the testing”

Rationale: The nurse should instruct the child to keep both eyes open during visual acuity testing.

When caring for a child, a nurse plans to use non-pharmacological interventions to enhance the effectiveness of pain medication. Which of the following strategies incorporates visualization techniques to help decrease the child’s discomfort?

Blowing bubbles with liquid soap to “blow the hurt away”

Rationale: Having the child blow bubbles is a visualization technique that can help to decrease the child’s discomfort. The child can visualize the pain as the bubble that they blow away from themself and into the air.

A nurse is preparing to administer heparin 5,000 units SQ. Available is heparin injection 10,000 units/mL. How many mL should the nurse administer per dose?

0.5 mL

5,000 units/ 10,000 units = 0.5 mL

A charge nurse is observing a newly licensed nurse performing a physical assessment on a client. Which of the following actions by the nurse indicates that the charge nurse should intervene?

The newly licensed nurse writes detailed notes while performing the head-to-toe assessment.

Rationale: The newly licensed nurse should record brief notes during the assessment to avoid delays and write more detailed notes after completing the assessment.

A nurse is assessing a client who has schizophrenia. The nurse should identify the following alteration in speech as which of the following? (Audio)

Clang association

Rationale: Clang association is an alteration in speech in which the client uses words based on their sound, rather than their meaning. Clients who have neurological disorders can also have this alteration in speech.

A nurse is assessing a school age-child who has cystic fibrosis. Which of the following findings is the priority for the nurse to report to the provider?

Hemoptysis 275 mL/24 hr

Rationale: Hemoptysis greater than 250 mL/24 hr indicates that this child is at greatest risk for hemorrhage. Therefore, this is the priority finding for the nurse to report.
Fever

A nurse is caring for a client who ha bipolar disorder. The nurse observes that the client is becoming increasingly restless. The client is pacing the unit and speaking rapidly, frequently using profanities and sexual references. Which of the following actions should the nurse take first?

Move the client to a quiet place away from others.

Rationale: The client’s behavior indicates the greatest risk is injury to others. Therefore, the first action the nurse should take is to prevent harm to other clients by moving the client to a quiet place away from others.

A nurse is providing colostomy care for a client using a two-piece pouching system. Which of the following actions should the nurse take?

Place the skin barrier over the stoma and hold it for 30 seconds.

Rationale: The nurse should activate the adhesive in the skin barrier by holding it in place over the stoma for 30 seconds.

A nurse is teaching the parent of a school-age about administering ear drops. Which of the following response by the parent indicates an understanding of the teaching?

“I should pull the top of the ear upward and back while instilling the medication.”

Rationale: The nurse should instruct the parent to pull the pinna upward and back in children older than 3 years of age to straighten the ear canal and allow the medication to reach the entire canal. For children younger than 3 years of age, the parent should gently pull the pinna downward and back.

A nurse is assessing a client who is 2 hr postoperative following a cardiac catheterization. Which of the following information should the nurse report to the provider?

Neurologic status

Rationale: This client is experiencing slurred speech and extremity weakness, which are indications of a stroke, a potential complication of cardiac catheterization. The nurse should report these findings to the provider.

A nurse is caring for a client who is receiving total parenteral nutrition (TPN) solution by continuous IV infusion at 60 mL/hr. The nurse discovers the infusion pump has stopped working. Which of the following actions should the nurse take while waiting for a new infusion pump?

Provide dextrose 10% in water solution using manual drip tubing at 60 mL/hr.

Rationale: The nurse should use an infusion pump when administering TPN solution to ensure accurate dosage and should taper the infusion rate before discontinuing the solution to prevent hypoglycemia. If the nurse is unable to continue the TPN infusion by infusion pump, the nurse should use manual drip tubing to infuse dextrose 10% in water at the same rate as the TPN solution.

A nurse is caring for a client who has an STI that must be reported to the state health department. Which of the following actions should the nurse take?

Explain to the client why this information will be shared.

Rationale: It is the responsibility of the nurse to advocate for the client, provide confidential information, and explain legal requirements. Reporting communicable disease occurrences helps with identifying outbreaks and overall disease trends.

A nurse is caring for a group of clients. For which of the following events should the nurse complete an incident report?

A client’s IV pump delivers an inadequate dose of medication.

Rationale: The nurse should complete an incident report to record occurrences which resulted in a medication error, such as a failure of the IV pump, as part of the quality improvement process. Other situations requiring an incident report include significant complaints about care quality and visitor or client injury.

A nurse is caring for a client who has hypertension and is taking captopril. Which of the following tasks should the nurse delegate to an assistive personnel (AP)?

Obtain the client’s blood pressure before the nurse administers medication.

Rationale: The nurse can delegate obtaining blood pressure before and after medication administration because this task is within the range of function for an AP.

A nurse is assessing a client who is receiving a blood transfusion. Which of the following findings should indicate to the nurse that the client is having a hemolytic transfusion reaction?

Low back pain

Rationale: The nurse should expect low back pain in a client who is having a hemolytic transfusion reaction.

A nurse is caring for a toddler who has infectious gastroenteritis. Which of the following actions should the nurse take?

Initiate oral rehydration therapy for the toddler.

Rationale: Infectious gastroenteritis can lead to dehydration. The nurse should treat the toddler with oral rehydration therapy to replace fluids lost by diarrhea. Soft or pureed foods can be given along with the oral rehydration therapy. After adequate rehydration has occurred, a regular diet can be resumed.

A nurse is administering medications to a client who has percutaneous gastrostomy tube for enteral feedings. Which of the following actions should the nurse take to prevent clogging of. the tube?

Flush the client’s gastrostomy tube with 30 mL of water before administering the medication.

Rationale: The nurse should flush the gastrotomy tube with at least 30 mL of water before and after medication administration to clear the tube of any residuals and to ensure patency.

A nurse is teaching home wound care to the family of a child who has a large wound. Which of the following interventions should the nurse recommend?

Double-bag soiled dressings in plastic bags for disposal.

Rationale: The client should double-bag soiled dressings in plastic bags to prevent the spread of micro-organisms to other household members.

A nurse is teaching the parents of a toddler about snacks. Which of the following foods should the nurse recommend?

Diced steamed carrots

Rationale: Diced steamed carrots are a safe food choice for toddlers because they are soft and do not present a choking hazard.

A nurse is assessing a preschooler who has cystic fibrosis and has been receiving oxygen therapy for the past 36 hr. Which of the following findings should the nurse identify is an indication that the client has developed oxygen toxicity?

Substernal pain

Rationale: The nurse should identify substernal pain as a manifestation of oxygen toxicity due to the increased work of breathing, such as in a preschooler who has cystic fibrosis.

A charge nurse is observing a newly licensed nurse administer enteral feedings via NG tube. Which of the following actions by the newly licensed nurse indicates an understanding of the procedure?

Keeps the head of the bed elevated to 45° for 1 hr after feedings

Rationale: The nurse should keep the client’s head elevated to 30° to 45° for 1 to 2 hr after feedings to decrease the risk for aspiration.

A nurse is assessing a client who has Raynaud’s disease. Which of the following findings should the nurse expect?

Blanching of the fingers and toes

Rationale: A client who has Raynaud’s disease can have blanching of the fingers and toes in response to exposure to cold or emotional stress. Pallor develops first, then cyanosis, followed by redness or heat as the vessels reperfuse, before the skin returns to the client’s baseline tone

A nurse is talking with the partner of a client who attempted suicide. Which of the following statements by the client’s partner should the nurse identify as the priority?

“My husband doesn’t know that I’ve already moved out of the house and filed for a divorce.”

Rationale: A lack of social support and isolation indicates the client is at greatest risk for another suicide attempt. Therefore, this is the priority concern that the nurse should report to the provider.

A nurse in a clinic receives a call from a guardian whose child has varicella. The guardian asks when the child can return to school. Which of the following responses should the nurse make?

“When crusts have formed on every lesion.”

Rationale: The child should return to school once all the lesions have crusted over. Varicella is no longer contagious after crusts have formed on all lesions.

A nurse is caring for a toddler who is admitted to the pediatric unit for surgery. Which of the following should the nurse include in the toddler’s plan of care?

Encourage the parents to bring toys from home.

Rationale: To help decrease the toddler’s anxiety, the nurse should encourage the family to bring familiar objects from home, such as toys, blankets, and feeding utensils.

A nurse is caring for an older adult client in the PACU following general anesthesia. Which of the following findings should the nurse report to the provider?

Audible stridor

Rationale: Audible stridor, or a high-pitched sound heard in the client’s airway indicates edema, laryngeal spasm, secretions, or some type of airway obstruction that could become life-threatening. The nurse should report this finding to the provider.

A nurse is preparing to insert an indwelling urinary catheter for a client. The nurse should assess the client for which of the following conditions prior to starting the procedure?

Latex allergy

Rationale: The nurse should assess the client for a latex allergy prior to the insertion of an indwelling urinary catheter due to the risk of an allergic reaction

A home health nurse is providing teaching about infection prevention to a client who has cancer and is receiving chemotherapy. Which of the following statements by the client indicates an understanding of the teaching?

“I will walk for short distances throughout the day.”

Rationale: The client should ambulate for short distances as tolerated throughout the day. This will help to reduce pulmonary stasis and prevent the development of respiratory infections

A nurse is caring for a client who has end-stage Alzheimer’s disease. The adult child of the client says to the nurse, “I don’t know why I bother to visit my mother anymore.” Which of the following responses should the nurse make?

“It seems like you feel your visits are a waste of time.”

Rationale: The nurse is using a clarifying technique that facilitates the nurse’s understanding of the adult child’s feelings

A charge nurse assigns a newly licensed nurse to care for a client who has a chest tube. The nurse expresses concern about having limited experience with monitoring chest tube drainage. Which of the following actions should the charge nurse take first to provide teaching about chest tubes?

Ask the nurse about their knowledge of the procedure.

Rationale: The first action the charge nurse should take using the nursing process is to assess the newly licensed nurse’s knowledge about the procedure. By assessing the nurse’s knowledge, the charge nurse can identify the nurse’s learning needs.

A nurse is caring for an adolescent client who has a new diagnosis of terminal cancer. When discussing the client’s prognosis with the parents, the nurse should recognize which of the following responses by the parents as an example of rationalization?

“Maybe this is better for our child because we don’t want any suffering through chemotherapy treatments.”

Rationale: By justifying the adolescent’s prognosis by searching for a more personally acceptable explanation for the impending loss, the parent is using the defense mechanism of rationalization.

A nurse is caring for a client who is at 28 weeks of gestation. The client asks the nurse to explain what causes the nurse to explain what causes her to have constipation. Which of the following responses should the nurse make?

“The enlarged uterus compresses the intestines and causes constipation.”

Rationale: During the second and third trimesters, the size and weight of the growing uterus cause both displacement and compression of the intestines. These changes cause a decrease in motility, leading to constipation.

A nurse is teaching a client who has opioid use disorder about methadone. Which of the following information should the nurse include in the teaching?

“Sedation is a common adverse effect of this medication.”

Rationale: Sedation and drowsiness are common adverse effects of methadone. Sedation most frequently occurs at the beginning of treatment or during dosage increases.

A community health nurse is reviewing the medical records of four newly diagnosed clients. The nurse should identify which of the following clients as having a nationally notifiable infectious condition?

An adolescent client who has foodborne botulism

Rationale: The nurse should report botulism to the CDC because this information is necessary for the prevention and control of this disease. Clients who ingest the botulism toxin can develop dysphasia, drooping eyelids, and vision changes, and in 12 to 36 hr can develop neurologic symptoms such as symmetric, flaccid paralysis and cranial nerve impairment.

A nurse is assessing a client who is experiencing autonomic dysreflexia. which of the following findings should the nurse expect? (SATA)

-Facial flushing is correct. The nurse should expect a client who has autonomic dysreflexia to have facial flushing. Flushing occurs from the point of the lesion upward.
-Nasal congestion is correct. The nurse should expect a client who has autonomic dysreflexia to have nasal congestion.
-Headache is correct. The nurse should expect a client who has autonomic dysreflexia to have a severe headache.

A nurse is caring for a client who is 12 hr postoperative, is receiving PCA for pain control, and requires a blood pressure check every 10 min. Which of the following staff members should the nurse assign to collect this information?

An assistive personnel (AP) who is assisting a client to return to bed

Rationale: Performing a blood pressure check is within the range of function of an AP, and the AP should be available to obtain a blood pressure within the specified time.

A charge nurse observes a staff nurse document a dressing change in a client’s chart that was not performed. Which of the following actions should the charge nurse take first?

Gather more information about the staff nurse’s actions.

Rationale: The first action the nurse should take when using the nursing process is to assess the reasons for the staff nurse’s negligent actions. Therefore, the charge nurse should gather additional information and discuss the issue with the staff nurse before deciding on the next course of action.

A home health nurse is providing teaching to a client who has hepatitis A. Which of the following instructions should the nurse include?

Use hydrogen peroxide to clean kitchen surfaces.

Rationale: The client should clean kitchen surfaces with hydrogen peroxide to kill the virus and prevent transmission.

A nurse manager is on a planning committee to develop an emergency preparedness plan. The nurse should recommend that which of the following actions takes place first when implementing an emergency preparedness plan?

Notify the incident commander.

Rationale: The first action to take when implementing an emergency preparedness plan is to notify the incident commander to initiate the command hierarchy and maintain order.

A nurse is performing an admission assessment of a preschooler who is in the acute phase of Kawasaki disease. Which of the following findings should the nurse expect?

Fever unresponsive to antipyretics

Rationale: The nurse should expect a child who has acute Kawasaki disease to have a high fever that is unresponsive to antibiotics or antipyretics.

A nurse is caring for an older adult client. Which of the following findings should the nurse recognize as a physiological change associated with aging?

Decreased lung expansion

Rationale: Older adult clients are more likely to have decreased lung expansion due to decreased mobility of the ribs.

A nurse is providing teaching about improving nutrition for a client who has multiple sclerosis. Which of the following instructions should the nurse include? (SATA)

-“A speech pathologist will be performing a swallowing study for you.” is correct. The nurse should instruct the client that a swallowing study will be performed to determine the client’s risk for aspiration due to difficulty swallowing, which is a manifestation of multiple sclerosis.
-“You should rest before eating a meal.” is correct. The nurse should encourage the client to rest before each meal. Clients who have multiple sclerosis often report weakness and are easily fatigued.
-“Thicken your beverages before drinking.” is correct. The nurse should instruct the client that liquids should be thickened to reduce the risk of aspiration due to difficulty swallowing, which is a manifestation of multiple sclerosis.

A nurse is assessing a client who has obstructive sleep apnea. For which of the following complications should the nurse monitor?

Hypertension

Rationale: The nurse should assess the client for hypertension, a complication of obstructive sleep apnea from hypoxia. Other complications include heart failure and cardiac dysrhythmias.

A charge nurse is teaching a newly licensed nurse how to identify true labor. Which of the following should the nurse include in the teaching?

The cervix transitions to an anterior position.

Rationale: In true labor, the cervix transitions to an anterior position and begins to dilate in preparation for birth.

A nurse is planning care for a client who is receiving hemodialysis via an established arteriovenous (AV) fistula in the right arm. Which of the following interventions should the nurse include in the client’s plan of care?

Auscultate the affected extremity for a bruit.

Rationale: The nurse should auscultate the AV fistula every 4 hr to ensure a bruit is present, which indicates patency.

A nurse is planning teaching about allowable foods for a client who has a history of uric acid-based urinary calculi formation. Which of the following foods should the nurse include in the teaching?

Oranges

Rationale: A client who is prone to uric acid calculi formation can eat citrus fruits.

A nurse is assessing a client who has multiple sclerosis. Which of the following manifestations should the nurse expect?

Nystagmus

Rationale: Nystagmus is involuntary eye movements and muscle spasticity, which are manifestations of multiple sclerosis.

A nurse is preparing to administer a long-acting insulin to a client who has diabetes mellitus. Which of the following actions should the nurse plan to take first?

Check the insulin dose with another licensed nurse.

Rationale: The greatest risk to the client is injury due to a medication error. Therefore, the priority action is for the nurse to validate the correct dose of insulin with another licensed nurse prior to administration. Insulin is a high-alert medication and incorrect dosages can be fatal for the client

A nurse is caring for a client who is in the manic phase of bipolar disorder. Which of the following manifestations should the nurse expect?

Grandiose delusions

Rationale: Clients who are in the manic phase of bipolar disorder typically exhibit behaviors that appear to be euphoric. Clients can also have abrupt mood changes, expansiveness, unlimited energy, poor impulse control, and grandiose delusions.

A case manager is reviewing the medical records of several clients. For which of the following clients should the nurse request an interprofessional care conference?

A client who has diabetes mellitus and has had repeated hospitalizations for diabetic ketoacidosis

Rationale: A client who is having repeated episodes of a life-threatening complication requires an interprofessional care conference so team members can address the client’s needs to provide care and support.

A nurse working on a medical-surgical unit receives a telephone call requesting the status of a client from an individual who identifies themself as the client’s parent. Which of the following actions should the nurse take?

Ask the caller for verification of their identity.

Rationale: According to HIPAA, if someone requests information about a client it is the nurse’s duty to protect that information. Therefore, the nurse should inform the caller that nurses cannot release any client information over the phone without the permission of the client. The nurse should ask for verification of the caller’s identity to determine if they have been authorized by the client to receive information.

A nurse is caring for a client who is at 37 weeks of gestation and is experiencing abruptio placentae. Which of the following findings should the nurse expect?

Persistent uterine contractions

Rationale: The nurse should expect a client who has abruptio placentae to experience persistent uterine contractions, board-like abdomen, and dark red vaginal bleeding.

A nurse is providing discharge instructions to a client following a total hip arthroplasty. Which of the following instructions should the nurse include?

Install a raised toilet seat at home.

Rationale: The client should use a raised toilet seat at home to minimize hip flexion and prevent hip dislocation.

A nurse is preparing to administer enoxaparin to a client. Identify the area the nurse should use to administer the injection.

A is correct. The nurse should recognize that enoxaparin is administered into the subcutaneous tissue, specifically in the periumbilical area

A nurse in an outpatient mental health clinic is working with a client who has post-traumatic stress disorder (PTSD) and asks the nurse to recommend a nonpharmacological therapy to use to provide relief of the manifestations. Which of the following complementary therapies should the nurse teach the client to use to help alleviate the distress?

Guided imagery

Rationale: Helping clients imagine themselves as strong and capable and in settings that are positive and therapeutic can assist clients who have PTSD by relieving anxiety and pain.

A nurse is caring for four clients. Which of the following clients should the nurse assign to an assistive personnel (AP) to assist with meals?

A client who has Alzheimer’s disease and is demonstrating aphasia

Rationale: Aphasia impairs the client’s ability to communicate but does not interfere with nutritional intake or place the client at an increased risk for aspiration while eating. Therefore, assisting the client with meals is within the AP’s range of function.

A community health nurse is assisting with the development of a disaster management plan. The nurse should include which of the following nursing responsibilities in the disaster response stage of the plan?

Performing a rapid needs assessment

Rationale: Disaster management includes prevention, preparedness, response, and recovery stages. The nurse should perform a rapid needs assessment during the response phase of the disaster cycle. A rapid needs assessment allows the nurse to identify the severity of the incident, the health needs of the community, and the priority actions needed during the response stage.

A community health nurse is preparing a health education program for a local rural community. Which of the following actions should the nurse plan to take first?

Identify health-related issues within the community.

Rationale: The first action the nurse should take when using the nursing process is to assess the clients living in the community to identify the prevalent health problems.

A charge nurse is planning an educational session for staff nurses about working with parents whose terminally ill children are candidates for donating their organs. Which of the following information should the nurse plan to include?

The family can have the child in an open casket without fearing that the organ donation might disfigure the child’s body.

Rationale: Removal of organs does not damage or violate the child’s body in a way that would prevent an open casket funeral.

A nurse is assessing a client who has schizophrenia and is taking chlorpromazine. Which of the following findings is the priority for the nurse to report the provider?

Temperature 39.4° C (102.9° F)

Rationale: The greatest risk to this client is injury from neuroleptic malignant syndrome, a potentially life-threatening adverse effect of chlorpromazine that can cause the client to have a high temperature, dysrhythmia, decreased level of consciousness, and a labile blood pressure. Therefore, the priority finding for the nurse report to the provider is a fever.

A nurse is providing discharge teaching to a client following a cataract extraction. Which of the following statements by the client indicates an understanding of the teaching?

“I will bend at my knees when picking an object up off the floor.”

Rationale: The client should avoid bending at the waist, because this movement increases intraocular pressure. The nurse should instruct the client to bend at the knees when picking up an object.

A nurse is assessing a client who has macular degeneration. Which of the following findings should the nurse expect?

Decreased central vision

Rationale: The nurse should expect a client who has macular degeneration to have a decrease or loss of central vision due to bleeding into the macula or yellow spots under the retina.

A nurse is planning care for a client who is receiving heparin to treat a deep-vein-thrombosis of the left lower leg. Which of the following interventions should the nurse include in the plan of care?

Elevate the affected leg.

Rationale: The nurse should elevate the client’s affected extremity to reduce edema and decrease the risk of chronic venous insufficiency.

A nurse is providing teaching to a client about newborn safety. Which of the following statements should the nurse include in the teaching?

“Set your hot water heater temperature at or below 120 degrees Fahrenheit.”

Rationale: The nurse should instruct the client to set the maximum hot water temperature to no more than 49° C (120° F). The nurse should also instruct the client to test the temperature of the bath water with her elbow prior to bathing the newborn.

A nurse manager is assisting the orientation of a newly licensed nurse. Which of the following actions by the nurse requires the nurse manager to intervene?

Tells the hospital chaplain a client’s diagnosis

Rationale: Discussing a client’s diagnosis with the hospital chaplain is a breach of client confidentiality and a violation of HIPAA.

A nurse is assessing a 2-month-old infant during a well-baby examination. Which of the following actions should the nurse take to assess the infant’s rooting reflex?

Stroke the infant’s cheek.

Rationale: The nurse should stroke the infant’s cheek to assess the rooting reflex, which should cause the infant to turn towards that side and suck.

A nurse is providing client education to a postpartum client who has decided to bottle feed the newborn. Which of the following instructions should the nurse include in the teaching to help prevent the discomfort of engorgement?

Place ice packs on the breasts for 15 min several times per day.
The client should place ice packs on the breasts to reduce swelling and relieve the pain caused by engorgement.

A nurse receives a request from a client to review the information in his medical record. Which of the following responses should the nurse give?

“There’s a protocol for reviewing your medical record, and I can initiate the process.”

Rationale: The client’s record is the legal property of the facility, but the client has a right to access the record, obtain a copy of the record, and request corrections to the document if there are discrepancies. According to HIPAA, the nurse is responsible for following the facility’s policy when providing the client with access to the medical record.

A nurse is administering the cyclophosphamide orally to a school-age child who has a neuroblastoma. Which of the following actions should the nurse take when administering this medication?

Maintain hydration with liberal fluid intake.

Rationale: The nurse should offer fluids frequently to maintain hydration and prevent hemorrhagic cystitis, which is an adverse effect of this medication.

A nurse in the delivery room is caring for a newborn immediately after birth. Which of the following actions should the nurse take first?

Dry the newborn.

Rationale: The greatest risk to the newborn is cold stress. Therefore, the first action the nurse should take is to dry the newborn.

A nurse is reviewing the urinalysis report of a client who has acute glomerulonephritis. Which of the following findings should the nurse expect?

Protein

Rationale: A client who has glomerulonephritis has increased glomerular permeability, which allows protein to filter into the urine. Therefore, the nurse should expect proteinuria on the urinalysis report.

A nurse is initiating discharge planning for a client who had a stroke and is experiencing right-sided weakness. Which of the following actions should the nurse take first?

Request a referral for the client to receive physical therapy.

Rationale: The greatest risk to this client is injury from falls. Therefore, the first action the nurse should take is to request a referral for physical therapy.

A nurse is teaching the parents of a preschooler about sleep promotion. The parents report that their child is demonstrating reluctance in going to bed at night and states, “I am not tired.” Which of the following statements by the parents indicate an understanding of the teaching?

“We should read a story together every night before bedtime.”

Rationale: Preschoolers respond to rituals that prepare them for bed, such as hearing a story or taking a bath.

A clinic nurse is caring for a client who is in the first trimester of pregnancy. The client reports using acupressure bands on both wrists. which of the followings statements by the client indicates that this therapy is having the desired effect?

“I have not vomited as much recently.”

Rationale: Using an acupressure band on the wrists is a type of complementary and alternative therapy that applies pressure to a specific part of the body and can be used to alleviate nausea and vomiting.

A nurse is planning care for a client who has thrombocytopenia. Which of the following instructions should the nurse include in the client’s plan of care?

Avoid venipunctures when possible.

Rationale: Clients who have thrombocytopenia have a decreased platelet count and are at risk for bleeding. To reduce the risk for bleeding, the nurse should avoid venipunctures when possible.

A nurse is preparing to administer 15 units of regular insulin along with 20 units of NPH insulin. Which of the following actions should the nurse plan to take?

Inject 20 units of air into the NPH insulin vial.

Rationale: The nurse should inject 20 units of air into the NPH insulin vial and withdraw the needle without touching the insulin, then proceed to inject 15 units of air into the regular insulin vial.

A nurse is caring for a client who is immediately postoperative following a total vaginal hysterectomy. Which of the following actions should the nurse take first?

Measure the client’s vital signs.

Rationale: The first action the nurse should take when using the nursing process is to assess the client. The nurse should monitor the client’s vital signs every 15 min until stable and then every 4 hr for the next 48 hr.

A nurse is providing discharge instructions to a client who has a new prescription for amitriptyline to treat depression. The nurse should identify that which of the following client statements indicates an understanding of the teaching?

“I should watch for common reactions like dry mouth and constipation.”

Rationale: The nurse should reinforce that increasing dietary fiber, fluid intake, and chewing sugar-free gum can alleviate the anticholinergic effects of dry mouth and constipation.

A nurse is teaching a client who has a new prescription for estradiol. For which of the following adverse effects of this medication should the nurse instruct the client to monitor and report to the provider>

Headaches

Rationale: The nurse should instruct the client to monitor for and report headaches. Headaches can be an indication of a thromboembolic stroke because estradiol increases the risk for adverse cardiovascular events.

A nurse is providing teaching to a parent of a child who has a permanent tracheostomy tube. Identify the sequence of steps the parent should follow to perform tracheostomy care.

When teaching the parent to provide tracheostomy care, the nurse should instruct the parent to first remove the inner cannula. Next, the nurse should instruct the parent to remove the soiled dressing and then clean the stoma with 0.9% sodium chloride irrigation. Finally, the nurse should instruct the parent to change the tracheostomy collar.

A nurse is caring for a newborn who has herpes simplex virus (HSV). Which of the following isolation precautions should the nurse initiate?

Contact

Rationale: The nurse should initiate contact precautions because clients transmit HSV by direct and indirect contact with others and the environment. The nurse should wear gloves when in close contact with the newborn.

A nurse is assessing a client who has antisocial personality disorder. Which of the following manifestations should the nurse expect?

Lack of remorse

Rationale: A client who has antisocial personality disorder is more likely to show a lack of remorse.

A nurse is caring for a client who had a recent stroke. Prior to transferring the client to the bedside commode, which of the following actions should the nurse take first?

Assess the client for functional limitations.

Rationale: When using the nursing process, the first action the nurse should take is to assess the client’s functional limitations to determine how much the client can assist with the transfer.

A nurse is preparing to administer diazepam 0.3 mg/kg IV bolus to a toddler who weighs 22 lb and is experiencing a grand mal seizure. Available is diazepam solution for injection 5 mg/mL. How many mL should the nurse administer?

0.6 mL

A nurse is caring for a client who is in labor at 39 weeks of gestation. During the second stage of labor, the nurse observes early decelerations on the monitor tracing. Which of the following actions should the nurse take?

Continue observing the fetal heart rate.

Rationale: Early decelerations indicate the progression of labor and are an expected finding. The nurse should continue to monitor the fetus by observing the fetal heart rate and tracing.

A nurse is interviewing a client who is now without a home due to a natural disaster. After ensuring the client’s safety, which of the following actions should the nurse take first?

Determine the client’s perception of the personal impact of the crisis.

Rationale: The first action the nurse should take using the nursing process is to assess the client. Therefore, the first action the nurse should take is to determine the client’s feelings and understanding of the natural disaster and its personal impact.

A charge nurse is planning care for a client who has mechanical restraints in place. Which of the following interventions should the nurse include in the plan?

Provide a staff member to stay with the client continuously.

Rationale: A staff member must remain continuously with a client who is in restraints or view the client via audiovisual equipment, if necessary, due to the risk of injury.

A nurse is performing tracheostomy care for a client who is postoperative following a laryngectomy. Which of the following actions should the nurse take when suctioning the client’s airway?

Apply suction for 10 seconds.

Rationale: The nurse should apply suction for only 5 to 15 seconds to minimize oxygen loss.

A nurse is caring for a client who has a closed-head injury and is receiving mechanical ventilation. The nurse should expect to administer which of the following medications to reduce intracranial pressure?

Mannitol

Rationale: The client should receive mannitol, an osmotic diuretic, to reduce intracranial pressure caused by cerebral edema.

A charge nurse overhears two staff nurses in the hallway discussing the nutritional status of a client who has anorexia nervosa. Which of the following actions should the charge nurse take?

Tell the nurses to stop the discussion.

Rationale: The nurses are violating client confidentiality by having the discussion in a public hallway. The charge nurse should tell the nurses to stop the discussion to prevent any further breach of confidentiality.

A community health nurse is performing triage tagging following a mass casualty incident. On which of the following clients should the nurse place a black tag?

A client who has significant head trauma and agonal respirations

Rationale: The nurse should place a black tag on a client who has significant head trauma and agonal respirations because this client is not likely to recover or will require extensive resources for care.

A nurse is planning care for a client who has a deficit with cranial nerve II. Which of the following actions should the nurse plan to take?

Clear objects from the client’s walking area.

Rationale: The nurse should plan to clear objects from the client’s walking area because CN II is the optic nerve and a deficit can result in visual impairment which can lead to falls.

A nurse is teaching a client who is to start taking misoprostol and currently is on long-term therapy with NSAIDs for arthritis. The nurse should provide the client with which of the following information?

Complete a serum pregnancy test before taking the medication.

Rationale: Misoprostol can induce uterine contractions. Clients of childbearing age must rule out pregnancy before taking misoprostol.

A nurse is creating a plan of care for a child who has acute lymphoid leukemia and an absolute neutrophil count of 400/mm3. Which of the following interventions should the nurse include in the plan?

Withhold administering the varicella vaccine to the child.

Rationale: A child who has severe immunodeficiency should not receive a live vaccine due to the risk of developing the disease. Inactivated vaccines can be administered to children who are immunosuppressed.

A nurse is assessing a client who has a stage II pressure injury. Which of the following wound characteristics should the nurse expect?

Partial-thickness skin loss

Rationale: The nurse should expect to see partial-thickness skin loss or blister formation in a client who has a stage II pressure injury.

A nurse in an emergency department is admitting a client who has cardiac tamponade. Which of the following assessment findings should the nurse expect?

Pulsus paradoxus

Rationale: The nurse should identify pulsus paradoxus, a finding in which the systolic BP is 10 mm Hg or greater on expiration than inspiration, as an expected finding of cardiac tamponade, along with jugular vein distention, bradycardia, and hypotension.

A nurse is caring for a client who has had nausea and vomiting for the past 2 days. The nurse should identify which of the following findings as an indication the client is experiencing fluid volume deficit?

Orthostatic hypotension

Rationale: Clients who have a fluid volume deficit can experience orthostatic hypotension, which is a result of the body’s inability to maintain adequate blood pressure following position changes.

A nurse is assessing a client who has sickle cell anemia. The nurse should identify which of the following findings as a manifestation of vasoocclusive crisis?

Hematuria

Rationale: The nurse should identify hematuria as a manifestation of vaso-occlusive sickle cell crisis resulting from ischemia of the kidneys

A nurse is caring for a client who has a potassium level of 3 mEq/L. For which of the following manifestations should the nurse monitor?

Decreased deep tendon reflexes

Rationale: A client who has hypokalemia can have muscle weakness and decreased deep tendon reflexes.

A nurse is caring for four clients at the beginning of a shift. After receiving change-of-shift report, which of the following clients should the nurse attend to first?

A client who is confused and has been attempting to get out of bed

Rationale: The nurse should recognize that a client who is confused and has been attempting to get out of bed is at greatest risk for injury from a fall. Therefore, the nurse should attend to this client first.

A client who is 24 hr postoperative following abdominal surgery refuses to ambulate. Which of the following actions should the nurse take first?

Ask the client to rate their pain level.

Rationale: Using the nursing process, the first action the nurse should take is to assess the client’s level of pain. If indicated, the nurse should administer an analgesic, then wait 30 to 45 min to allow the analgesic to take effect before encouraging the client to ambulate. Management of the client’s pain is a priority for encouraging postoperative activity.

A nurse is teaching a client about foods high in vitamin A. Which of the following foods should the nurse recommend as having the highest amount of vitamin A?

1 medium raw carrot

Rationale: The nurse should identify that 1 medium raw carrot contains 2,025 mcg/dL of vitamin A and is therefore the best food to recommend to the client.

A nurse working in a long-term care facility is assessing an adult client. Which of the following findings places the client at risk for development of a pressure injurty?

Recent weight loss

Rationale: Weight loss can increase the risk for pressure injury. Inadequate nutrition will cause decreased nutrients for the skin and tissues and increases the chance for shearing against the bony prominences.

A nurse is teaching a client who has a new prescription for digoxin about manifestations of toxicity. Which of the following findings should the nurse include in the teaching?

Nausea

Rationale: The nurse should instruct the client to monitor for and report manifestations of digoxin toxicity, such as nausea, anorexia, abdominal pain, bradycardia, and visual changes.

A home health nurse is assessing a 2-week-old newborn who had a birth weight of 3.64 kg (8 lb) and is being breastfed. Which of the following findings indicates effective breastfeeding?

The newborn has six to eight wet diapers per day.

Rationale: Measuring the number of wet diapers per day is an effective measurement of adequate intake. Six to eight wet diapers each day after the fourth day of life indicates effective breastfeeding.

A nurse is reviewing the laboratory findings of a client who is experiencing chest pain. The nurse should identify that an elevation in which of the following laboratory values indicates cellular injury of myocardial tissue?

Troponin T

Rationale: Troponin T is a myocardial muscle protein that is released into circulation after cardiac injury. The nurse should expect increases in the client’s troponin level within 2 to 3 hr following a myocardial injury.

A nurse is teaching a newly admitted client who has heart failure about advance directives. Which of the following statements should the nurse make?

“You should complete advance directives in the event you cannot express your own wishes.”

Rationale: The client should prepare advance directives to make their wishes known should they be unable to communicate them in the future.

A nurse on a mental health unit is caring for a client who tells the nurse that she does not want to receive a scheduled dose of lorazepam IM. Which of the following actions should the nurse take?

Document the client’s refusal of the medication.

Rationale: The client has the right to refuse medication. The nurse should document the refusal in the client’s medical record.

A nurse is preparing to administer 2 units of fresh frozen plasma to a client. Which of the following actions should the nurse plan to take ?

Enter the plasma product number into the client’s medical record.

Rationale: The nurse should complete documentation following blood product therapy, which includes recording the type of product, amount administered, product number, infusion time, and client response.

A nurse is providing discharge instructions to a client who has a new prescription of warfarin. Which of the following client statements should the nurse identify as an indication that the client understands the teaching?

“I should report a change in the color of my stools.”

Rationale: The nurse should inform the client that red, black, or tarry stools can indicate bleeding, an adverse effect of warfarin, and the client should report these findings to the provider.

A charge nurse is preparing to administer 0900 medications and is told by the pharmacy staff that the medications are not available. Medications availability has been ongoing problem, and the charge nurse has previously discussed this issue with the pharmacy staff. Which of the following actions should the charge nurse take first?

Inform the nurse manager of the issue.

Rationale: The greatest risk to clients is injury from not receiving medications on time and developing a medical complication. Therefore, the priority intervention the charge nurse should take is to follow the chain of command and contact the nurse manager.

A nurse is teaching about total parenteral nutrition (TPN) and IV lipid emulsions with a client who has an extensive burn injury. Which of the following information should the nurse include?

“You will receive fingersticks for blood glucose testing.”

Rationale: A client who is receiving TPN is at risk for hyperglycemia due to the dextrose in the TPN solution. Therefore, the client will require blood glucose monitoring

A nurse is caring for a client who has a fractured femur and has had a fiberglass leg cylinder cast for 24 hr. Which of the following assessment findings should the nurse identify as the priority?

The client’s heel is reddened and tender.

Rationale: The greatest risk to this client is injury from a pressure injury. Therefore, the priority assessment finding the nurse should identify is a reddened and tender heel.

A mental health nurse is conducting the first of several meetings with a client whose partner recently died. The nurse should perform which of the following actions to establish trust during the orientation phase of the nurse-client relationship?

Establish the termination date of therapy.

Rationale: This task occurs in the orientation phase of a therapeutic relationship.

A nurse is performing gastric lavage for a client who has gastrointestinal bleeding an an NG tube in place. Which of the following actions should the nurse take?

Use 0.9% sodium chloride for irrigation of the NG tube.

Rationale: The nurse should use 0.9% sodium chloride, sterile water, or tap water for irrigation of the client’s NG tube.

A nurse is assessing a client who has been taking lithium carbonate for the past month to treat bipolar disorder. Which of the following assessment findings should the nurse identify as the priority?

Confusion

Rationale: When using the urgent vs. nonurgent approach to client care, the nurse should determine that the priority finding is confusion because it is an early manifestation of lithium toxicity. The nurse should monitor the client for additional indications of lithium toxicity, including coarse hand tremors, incoordination, ECG changes, and sedation.

A nurse is preparing to perform an intermittent urinary catheterization for a client who has urinary retention. Which of the following images indicates the catheter the nurse should use?

A straight urinary catheter, which should be used to perform an intermittent catheterization for a client who has urinary retention.

A nurse is reviewing the ABG results of a client who has COPD. The results include a pH of 7.3, PaO2 56 mm Hg, PaCO2 54 mm Hg, HCO 26 mEq/L, SaO2 87%. Which of the following is the correct interpretation of these values?

Uncompensated respiratory acidosis

Rationale: A pH of 7.3 is below the expected reference range and indicates the client has acidosis. The PaCO2 of 54 mm Hg is above the expected reference range, which, when combined with the low pH, indicates that the acidosis has a respiratory origin. The HCO3- of 26 mEq/L is within the expected reference range, indicating that the acidosis is not metabolic in origin and the body has not yet corrected the imbalance through compensation.

A nurse is assessing a client who has skeletal traction for a femur fracture. Which of the following findings should the nurse identify as the priority?

Upper chest petechiae

Rationale: The greatest risk to this client is organ damage from fat embolism syndrome, a life-threatening complication of fractures. In fat embolism syndrome, a fat embolus enters the blood stream and can obstruct blood vessels of a major organ, such as the lung, kidney, or brain. Manifestations include petechiae on the upper torso, dyspnea, hypoxia, headache, lethargy, and confusion. Therefore, the nurse should identify this as the priority finding.

A nurse in an acute mental health facility is planning care for a client who has anorexia nervosa. Which of the following interventions should the nurse include in the client’s plan of care?

Supervise the client during and after eating.

Rationale: The nurse should monitor the client during and for 1 hr after meals to prevent the client from hiding food or purging.

A nurse is preparing a sterile field in order to insert an indwelling urinary catheter for a male client. Which of the following techniques should the nurse use to maintain surgical aseptic technique?

Set the catheter tray on the overbed table at waist height.

Rationale: To maintain sterility, the nurse should place the catheter tray on a work surface at or above waist level.

A nurse is caring for a child who is experiencing a tonic clonic seizure. Which of the following actions should the nurse take?

Place the child in a side-lying position.

Rationale: The nurse should place the child in a side-lying position during a seizure to maintain a patent airway, decrease the risk of aspiration, and facilitate drainage of oral secretions.

A nurse manager is preparing a newly licensed nurse’s performance appraisal. Which of the following methods should the nurse manager use to evaluate the nurse’s time management skills?

Maintain regular notes about the nurse’s time management skills.

Rationale: Maintaining notes over a period of time provides a comprehensive view of the nurse’s abilities so the manager can identify trends in the nurse’s overall performance.

Leave a Comment

Scroll to Top